0% found this document useful (0 votes)
767 views376 pages

ابراهيم

Uploaded by

Karren Fernandez
Copyright
© © All Rights Reserved
We take content rights seriously. If you suspect this is your content, claim it here.
Available Formats
Download as PDF, TXT or read online on Scribd
0% found this document useful (0 votes)
767 views376 pages

ابراهيم

Uploaded by

Karren Fernandez
Copyright
© © All Rights Reserved
We take content rights seriously. If you suspect this is your content, claim it here.
Available Formats
Download as PDF, TXT or read online on Scribd
You are on page 1/ 376

PrometricMCQ.

com 11/03/2015 10:45

A one month old boy present with the head


tilted towards the left side and the chin
rotated to the right side. There is a
palpable mass of soft tissue on the right
side of the neck near the clavicle:

Passive muscle stretching

Surgical release of the muscle


(Correct Answer)

Surgical removal of the mass

It’s a normal mass in infants

Report an issue

A patient with bowlegs due to abnormal


bone formations and deformities has
calcium level of 7.5 mg/100ml.
Which of the following foods would the
nurse most likely instruct the patient to add
to a diet?

Organ meats

Whole grains (Correct Answer)

Egg yolks

Chicken meat

Report an issue

file:///Users/mohammad/Desktop/PrometricMCQ.com.webarchive Page 2 of 377


PrometricMCQ.com 11/03/2015 10:45

A patient is one day post-operative repair


of a large umbilical hernia. The patient
complaints of abdominal pain and describe
feeling the sutures give way. Upon
assessment of the abdomen the nurse
observes an evisceration. The nurse’s
IMMEDIATE response should be to:

Medicate the patient for pain

Instruct the patient to cough hard

Have the patient perform the


Valsalva maneuver

Cover the abdomen with a sterile


saline spiked dressing (Correct
Answer)

Report an issue

While reviewing stress management


techniques with a patient diagnosed with
multiple sclerosis, what would the nurse
identify as MOST appropriate?

Relaxing in a warm bubble bath

Yoga in a cool room (Correct


Answer)

Sunbathing

Cross-country running

file:///Users/mohammad/Desktop/PrometricMCQ.com.webarchive Page 3 of 377


PrometricMCQ.com 11/03/2015 10:45

Report an issue

A four year – old boy presented with a firm


and non- tender mass that his mother had
discovered in the right upper abdominal
quadrant He was admitted to the hospital
and underwent complete surgical excision
of a renal tumor that had extended beyond
the kidney. The child was then scheduled
for radiation and chemotherapy treatment.
Which stage was the tumor?

II (Correct Answer)

III

IV

Report an issue

When caring for a patient with an ostomy


the nurse knows that extra skin protection
for the personal skin is MOST important for
those patients with a (an):

Ileostomy (Correct Answer)

Ascending colostomy

Transverse colostomy

Sigmoid colostomy

file:///Users/mohammad/Desktop/PrometricMCQ.com.webarchive Page 4 of 377


PrometricMCQ.com 11/03/2015 10:45

Report an issue

The nurse is assessing the growth and


development of a healthy three year-old
child. The nurse should expect the child to
be able to:

Ride a bicycle

Jump rope

Throw a ball overhead (Correct


Answer)

Hop on one foot

Report an issue

The nurse is monitoring a patient


recovering parenteral nutrition via a central
line catheter for the 24 hours. The patient
has polyuria and complaint thirst and
headache.
Blood pressure 120/76 mmHg
Heart rate 88/min
Respiratory rate 16/mm
Temperature 37.1°C
Oxygen Saturation 99% an room air
What will the nurse most likely
administers?

Insulin (Correct Answer)

Dextrose 5% in water

file:///Users/mohammad/Desktop/PrometricMCQ.com.webarchive Page 5 of 377


PrometricMCQ.com 11/03/2015 10:45

Normal saline

Calcium gluconate

Report an issue

A 22 year-old man suffered major injuries


to the right leg in a road traffic accident
that resulted in an above –the- knee
amputation with a rigid dressing. Twelve
hours later the patient complains of
increasing pressure from the dressing and
the feeling that something is crawling
around the stump.

Notify the physician (Correct


Answer)

Remove and reapply the dressing

Remind patient phantom limb pain is


normal

Elevate and apply ice to the residual


limb

Report an issue

A patient with congestive heart failure and


severe peripheral edema has a nursing
diagnosis of fluid volume excess
What are the two MOST important
interventions for the nurse to initiate?

file:///Users/mohammad/Desktop/PrometricMCQ.com.webarchive Page 6 of 377


PrometricMCQ.com 11/03/2015 10:45

Diuretic therapy and intake and


output (Correct Answer)

Nutritional education and low-


sodium diet

Daily weights and intake and output

Low-sodium diet and elevate legs


when in bed

Report an issue

A nurse is providing care to a patient with a


new skin graft on the leg. The patient is
upset and the nurse notes copious red
drainage oozing around the dressing the
nurse should immediately:

Lift the dressing to assess the area

Ask if the patient is having any pain

Apply firm pressure for 10 to 15


minutes (Correct Answer)

Assess the apical pulse

Report an issue

A 25 years-old male patient suffered a


spinal cord injury at the T-4 level and is
being cared for in hospital. The nurse
enters the patient’s room and finds the
patient sitting upright and looking anxious

file:///Users/mohammad/Desktop/PrometricMCQ.com.webarchive Page 7 of 377


PrometricMCQ.com 11/03/2015 10:45

and restless. He complains of sudden


headache and nausea. Sweat forms on his
forehead yet his feet are cool to touch
Blood pressure 150/100 mmhg Heart rate
55/min Respiratory rate 28/min
Temperature 37.1C What nursing
intervention is initially most appropriate?

Assess for a full bladder (Correct


Answer)

Lower the head of the bed 30


degrees

Loosen clothing and bed sheets

Apply heating pad to lower


extremities

Report an issue

To minimize a toddler from scratching and


picking at healing skin graft, the nurse
should utilize:

Mild sedatives

Hand mittens (Correct Answer)

Punishment for picking

Distractions

Report an issue

file:///Users/mohammad/Desktop/PrometricMCQ.com.webarchive Page 8 of 377


PrometricMCQ.com 11/03/2015 10:45

A 30 year-old diabetic woman complains of


blood in the urine and dull pain over the left
lower abdomen, middle back and above
the pubic bone. She feels the urge to
urinate frequently and urgently. For the
past two days, she has had nausea and
vomiting. Abdominal assessment reveals
tenderness over the pubic bone; no
guarding and bowel sounds are active in all
quadrants.
Which home intervention would be most
effective?

Oral antibiotics (Correct Answer)

Increased fluid intake

Increased dietary protein

Cold application to left hand

Report an issue

The nurse calls together an interdisciplinary


team with members from medicine, social
service, the clergy, and nutritional services
to care for a patient with a terminal illness.
Which of the following types of care would
the team MOST likely be providing?

Palliative (Correct Answer)

Curative

Respite

Preventive

file:///Users/mohammad/Desktop/PrometricMCQ.com.webarchive Page 9 of 377


PrometricMCQ.com 11/03/2015 10:45

Report an issue

Pulse Volume Scale


Scale
Description of pulse
0
Absent pulse
1
Weak and Thready pulse
2
Normal
3
Bounding pulse
A 26 year- old female patient’s radial pulse
is assessed for a full minute. The nurse
notes the pulse is difficult to palpate and
count.
What should the nurse chart? Blood
pressure 82/48 mmHg Respiratory rate
20/min Temperature 38.8°C

1 (Correct Answer)

Report an issue

A patient recently underwent coronary


artery graft (CABG) surgery.
Which of the following nursing diagnose
PRIORITY?

file:///Users/mohammad/Desktop/PrometricMCQ.com.webarchive Page 10 of 377


PrometricMCQ.com 11/03/2015 10:45

Anxiety

Impaired gas exchange

Acute pain (Correct Answer)

Sleep deprivation

Report an issue

A child with asthma has an order for


albuterol, before administration of the
medication the nurse MUST.

Pre-oxygenate the patient

Assess the patient’s heart rate


(Correct Answer)

Obtain venous Access

Feed the patient a snack

Report an issue

A 52 year- old woman is scheduled to


undergo an abdomino- perineal resection
in three days for removal of a cancer of the
rectum. The nurse reviews the care plan
with the patient. The patient will receive
prophylactic antibiotics and will be given a
mechanical bowel preparation the day
before.
Which additional preparation should the
patient undertake at this time?

file:///Users/mohammad/Desktop/PrometricMCQ.com.webarchive Page 11 of 377


PrometricMCQ.com 11/03/2015 10:45

Wear pressure stockings

Perform leg strengthening exercises

Maintain high- protein, low- residue


diet (Correct Answer)

Take daily ferrous iron tablets

Report an issue

A 32 year-old female has an adrenocortical


hormone disorder. The nurse notes the
patient has thin scalp hair, a large trunk
and thin extremities. Blood pressure
152/84 mmHg Heart rate 64/min
Respiratory rate 16/min Temperature
37.2°C Oxygen Saturation 98% on room
air
Which clinical findings are most likely?

Decreased bowel sounds and


muscle soreness

Cardiac arrhythmias and abdominal


cramping

Headache, confusion and muscle


twitching (Correct Answer)

Hunger, trembling and nervousness

Report an issue

file:///Users/mohammad/Desktop/PrometricMCQ.com.webarchive Page 12 of 377


PrometricMCQ.com 11/03/2015 10:45

A 62 year-old man is diagnosed with


localized non cell tumor of the lung without
metastasis and under lobectomy. After the
procedure, he is brought to Postoperative
Anesthesia Care Unit for recovery with
chest tube drainage system in place and
administered by nasal cannula.
Which nursing action is initially most
important?

Assess level of consciousness

Attach chest tubes to the bed


sheets

Perform deep breathing and


coughing

Apply elastic stockings to the legs


(Correct Answer)

Report an issue

A 67 year-old man is admitted to the Post-


anesthesia Recovery unit following chest
surgery. The patient has a right chest tube
that is attached to low suction. Three hours
after admission to the unit, the nurse
observes the drainage output from the
chest tube is 300 milliliters.
What is the most appropriate initial
intervention?

Notify the doctor (Correct Answer)

Reduce IV infusion rate

Strip tube with roller device

Re-position in left lateral decubitus

file:///Users/mohammad/Desktop/PrometricMCQ.com.webarchive Page 13 of 377


PrometricMCQ.com 11/03/2015 10:45

Report an issue

An elderly patient with severe degenerative


joint comes to the clinic for routine follow
up of management. The patient reports
that over the month, the pain has begun to
increase in severity patient requests an
increase in dosage of the medication. The
nurse recognizes that this is most likely
due to?

Drug addiction

Drug tolerance (Correct Answer)

An improvement in condition

Lack of efficacy of the current


medication

Report an issue

A community health nurse visits a patient


who has suffered a stroke. The patient’s
spouse explains to the nurse that the
patient chokes while eating some times.
Which of the following referral orders
would the nurse anticipate needing for this
patient?

Speech therapist

Dietician

file:///Users/mohammad/Desktop/PrometricMCQ.com.webarchive Page 14 of 377


PrometricMCQ.com 11/03/2015 10:45

Physical therapist (Correct


Answer)

Neurologist

Report an issue

A patient is transferred to the Intensive


Care following a craniotomy. The patient is
difficult to arouse and the pupils are
pinpoint and non-reactive.
Blood pressure 118/70 mmHg
Heart rate 58/min
Respiratory rate 11/min
Temperature 37.2°C
Which medication should the nurse
prepare administer?

Adrenaline

Thiamine

Naloxone (Correct Answer)

Dextrose 50%

Report an issue

The nurse has been assigned to care for a


60 year old critically ill patient with a triple-
lumen central venous line. The doctor’s
orders include daily care of the insertion
site and catheter device. The nurse creates

file:///Users/mohammad/Desktop/PrometricMCQ.com.webarchive Page 15 of 377


PrometricMCQ.com 11/03/2015 10:45

care plane based on the nursing diagnosis,


Risk for infection related to insertion of a
venous catheter.
Which intervention is most likely to prevent
infection?

Re-cap access hub after drawing


blood

Maintain clean technique

Wash hands before performing


catheter care (Correct Answer)

Clean catheter tubing with isopropyl


alcohol

Report an issue

A patient who underwent a right knee


arthroplasty two days ago has a nursing
diagnosis of impaired mobile. The patient
refuses to get out of bed and ambulate due
chest pain.
Which of the following action would the
nurse MOST likely implement?

Medicate the patient prior to


ambulation

Add a nursing diagnosis of non-


compliance

Let the patient rest now and then try


to ambulate later

Assess to determine the cause of


the chest pain (Correct Answer)

file:///Users/mohammad/Desktop/PrometricMCQ.com.webarchive Page 16 of 377


PrometricMCQ.com 11/03/2015 10:45

Report an issue

Prior to providing care for a hospitalized


infant the nurse who focuses on preventive
measures must.

Introduce self-parent

Perform hand hygiene (Correct


Answer)

Have a witness present

Assess the child’s


developmental level

Report an issue

The nurse is inserting a nasogastric (NG)


tube into patient as prescribed. The nurse
has advanced the into the patient’s
posterior pharynx. The nurse show now
ask the patient to?

Hold the breath

Stare upwards with the eyes


towards the ceiling

Perform the Valsalva maneuver

Lower the chin towards the chest


(Correct Answer)

file:///Users/mohammad/Desktop/PrometricMCQ.com.webarchive Page 17 of 377


PrometricMCQ.com 11/03/2015 10:45

Report an issue

What would be the long-term management


goal for a patient with diabetes mellitus,
type II?

Quickly lower blood glucose level by


administering insulin

Achieve hypoglycemic state

Maintain daily exercise (Correct


Answer)

Understand how to self-administer


msulax

Report an issue

The nurse administered a dose of


morphine sulfate, as prescribed to a
patient who is in the post-anesthesia care
unit (PACU). The patient appears to be
resting comfortably; the respiratory rate is
8 and the ð​‘¶ð​Ÿ​saturation on 2L of oxygen
via nasal cannula is 86%. The nurse should
IMMEDIETLY administer.

Flumazenil (Romazicon)

Midazolam (Versed)

Naloxone (Narcan) (Correct


Answer)

file:///Users/mohammad/Desktop/PrometricMCQ.com.webarchive Page 18 of 377


PrometricMCQ.com 11/03/2015 10:45

Ondansetron (Zofran)

Report an issue

A patient is 90-minutes post-abdominal


hysterectomy and complains of pain in the
post anesthesia care unit (PACU). The
nurse administers the prescribed dose unit
intravenous morphine. Five minutes later,
the patient is difficult to arouse, with the
following vital signs.
What should the nurse do NEXT?
Respiratory rate 6/min Saturation 86% on
room air

Document the efficacy of analgesic


therapy

Administer naloxone (Narcan)


(Correct Answer)

Place in supine position

Check the surgical site for bleeding

Report an issue

The stage of dying as identified by Dr.


Elizabeth Kubler-Ross occurs in what
order?

file:///Users/mohammad/Desktop/PrometricMCQ.com.webarchive Page 19 of 377


PrometricMCQ.com 11/03/2015 10:45

Anger, depression, bargaining,


denial, acceptance

Bargaining, denial, Anger,


acceptance, depression

Denial, Anger, bargaining,


depression, acceptance (Correct
Answer)

Depression, denial, Anger,


bargaining, acceptance

Report an issue

Which of the following statements is most


accurate?

Girls have more ADHD than asthma

Boys are more frequently diagnosed


with chronic illnesses

file:///Users/mohammad/Desktop/PrometricMCQ.com.webarchive Page 20 of 377


PrometricMCQ.com 11/03/2015 10:45

Boys are less frequently diagnosed


with chronic illnesses

Girls have more other chronic


illnesses than ADHD (Correct
Answer)

Report an issue

A 42 year- old patient is in a lower body


cast following a motor vehicle accident. In
order to minimize muscle strength loss
while in the cast, the nurse will instruct the
patient in the performance of.

Isometric exercises (Correct


Answer)

Passive range of motion exercises

Active-assistive range of motion


exercises

Resistive range of motion exercises

Report an issue

A patient has a history of severe,


uncontrolled epistaxis. The patient’s blood
pressure and platelet count are normal. To
minimize the occurrence of bleeding
episodes the nurse should teach the
patient to.

file:///Users/mohammad/Desktop/PrometricMCQ.com.webarchive Page 21 of 377


PrometricMCQ.com 11/03/2015 10:45

Sleep with the head elevated on at


least two to three pillows

Apply firm pressure to the nostrils


four times a day

Apply a water- soluble lubricant to


the nasal septum twice daily
(Correct Answer)

Minimize the intake of caffeine and


increase fluids intake

Report an issue

A six year-old patient has been diagnosed


with acute rheumatic fever. The nurse
knows that the antibiotic of choice for this
illness is:

Benzathine penicillin (Megacillin)


(Correct Answer)

Amoxicillin (Amoxil)

Erythromycin (Erythrosine)

Vancomycin (Vancocin)

Report an issue

20 year- old woman takes cyclosporine


100 mg by mouth twice per day. She had a
heart transplant two months ago and
complains of dizziness, shortness of

file:///Users/mohammad/Desktop/PrometricMCQ.com.webarchive Page 22 of 377


PrometricMCQ.com 11/03/2015 10:45

breath, light headedness and fatigue.


Capillary refill >3. Blood pressure 90/50
mmHg Heart rate, irregular 100/min
Respiratory rate 28/mm Temperature
38.0°C
What is the most likely underlying cause of
the patient’s problem?

Cyclosporine toxicity

Hyperkalemia

Organ rejection (Correct Answer)

Infection

Report an issue

A patient is transferred to the Intensive


Care Unit following a craniotomy. The
patient is difficult to arouse and the pupils
are pinpoint and non-reactive.
Blood pressure 118/70 mmHg
Heart rate 58/min
Respiratory rate 11/mm
Temperature 37.2°C
Which medication should the nurse
prepare to administer?

Adrenaline

Thiamine

Naloxone (Correct Answer)

Dextrose 50%

file:///Users/mohammad/Desktop/PrometricMCQ.com.webarchive Page 23 of 377


PrometricMCQ.com 11/03/2015 10:45

Report an issue

A 23 year- old male comes to the


Emergency Department in a sickle cell
crisis. He reports that his pain level is a
10/10 in all extremities. During the
assessment, he cannot lie still because of
the pain. There is no cyanosis or clubbing
in the extremities and all examination
findings are normal. The vital signs
recorded were
Blood pressure 132/82 mmHg Heart rate
110/min
Respiratory rate 18/mm
Temperature 38.4°C
Oxygen Saturation 94% an room air
Which nursing diagnosis is first priority?

Acute pain (Correct Answer)

Fluid volume deficit

Ineffective tissue perfusion

Ineffective airway clearance

Report an issue

A patient who underwent a right knee


arthroplasty two days ago has a nursing
diagnosis of impaired mobility. The patient
refuses to get out of bed and ambulate due
to chest pain.
Which of the following actions would the
nurse MOST likely implement?

file:///Users/mohammad/Desktop/PrometricMCQ.com.webarchive Page 24 of 377


PrometricMCQ.com 11/03/2015 10:45

Medicate the patient prior to


ambulation

Add a nursing diagnosis of non-


compliance

Let the patient rest now and then try


to ambulate later.

Assess to determine the cause of


the chest pain (Correct Answer)

Report an issue

Upon assessment of an obese patient’s


back, the nurse observes a forward
curvature of the lumbar spine. These
findings are indicative of.

Kyphosis

List

Lordosis (Correct Answer)

Scoliosis

Report an issue

A newborn was delivered pre-term


weighing 2700 grams with. Apgar scores of
4 and 6, respectively. When the mother
had presented to the Obstetrical Triage

file:///Users/mohammad/Desktop/PrometricMCQ.com.webarchive Page 25 of 377


PrometricMCQ.com 11/03/2015 10:45

Unit, she was already 7 centimeters dilated


and fully effaced. Her due date was
unknown as she had no parental care. The
infant showed signs of fetal distress and
was finally delivered by Cesarean section.
At birth a large, thin, membranous sac was
protruding from the umbilical base.
What is the priority nursing intervention at
birth?

Maintain cardio respiratory


stability (Correct Answer)

Protect the herniated viscera

Manage fluid intake and output

Establish vascular access

Report an issue

A surgeon instructs a nurse to serve as a


witness to an elderly patient’s informed
consent for surgery. During the surgeon’s
explanation to the patient, it becomes clear
that the patient is confused and does not
understand the procedure, but reluctantly
signs the consent from. The nurse should.

Sign the form as a witness, making


a notation that the patient did not
appear to understand (Correct
Answer)

Not sign the form as a witness and


notify the nurse supervisor

Not sign the form and answer the


patient’s questions after the
surgeon leaves the room

file:///Users/mohammad/Desktop/PrometricMCQ.com.webarchive Page 26 of 377


PrometricMCQ.com 11/03/2015 10:45

Report an issue

A child is treated for superficial (first-


degree) thermal burns to the thigh. The
child is in great discomfort and does not
eat.
Which of the following diagnoses should
receive PRIORITY?

Altered nutrition

Impaired skin integrity

Risk for infection

Acute pain (Correct Answer)

Report an issue

A patient is being prepared for a right


breast biopsy under general anesthesia.
The patient asks the nurse about the
surgical scar and possible postoperative
complications.
Which of the following actions would be
appropriate for the nurse to take?

Review the postoperative risks with


the patient

Notify the surgeon about the


patient’s questions (Correct
Answer)

file:///Users/mohammad/Desktop/PrometricMCQ.com.webarchive Page 27 of 377


PrometricMCQ.com 11/03/2015 10:45

Compete the patient’s


preoperative check list

Show the patient photos of breast


surgical scar

Report an issue

A patient who is 4 days postoperative after


a total hip replacement surgery, is obese
and has not been able to ambulate since
the surgery. The patient is now diaphoretic,
has chills, and complains of pain in the
thigh. There is tenderness over the
anteromedial surface of the thigh. The
MOST likely cause is.

Wound infection (Correct Answer)

Deep vein thrombosis (DVT)

Pulmonary edema

Dehydration

Report an issue

A nurse is communicating with a


moderately autistic child during a
developmental and behavioral assessment.
The nurse uses non-verbal techniques to
engage the child’s cooperation. She uses
facial expressions when responding and
simple body language to communicate
commends.
What is the most likely age of the child?
file:///Users/mohammad/Desktop/PrometricMCQ.com.webarchive Page 28 of 377
PrometricMCQ.com 11/03/2015 10:45

One

Three (Correct Answer)

Six

Nine

Report an issue

A 27 year-old woman presents with


stomach cramping with alternating
constipation and diarrhea. She had been
diagnosed with irritable bowel syndrome
two years before and has so far managed
the illness with lifestyle changes, including
diet and exercise.
What is the most appropriate advice/

Increase dairy intake

Use antacids to relive pain

Increase dietary fiber

Avoid emotional stress triggers


(Correct Answer)

Report an issue

A nine year-old child has been diagnosed


with external otitis. In addition to assessing
severity of symptoms and need for pain
relief, the nurse should assess.
file:///Users/mohammad/Desktop/PrometricMCQ.com.webarchive Page 29 of 377
PrometricMCQ.com 11/03/2015 10:45

The hearing of the child

The tympanic temperature

Speech and language development

Ear hygiene and the need for


earplugs (Correct Answer)

Report an issue

What is the most likely percentage rate per


liter for oxygen administration via nasal
cannula for this patient?

0.5-1

1.5-2

2.5-3

5-6 (Correct Answer)

Report an issue

Which of the following questions by the


nurse should receive priority?

Do you experience sudden bruises


anywhere in your body? (Correct
Answer)

file:///Users/mohammad/Desktop/PrometricMCQ.com.webarchive Page 30 of 377


PrometricMCQ.com 11/03/2015 10:45

Has your weight changed?

Report an issue

Which position is best for this patient?

Supine

Reverse Trendelenburg’s

Prone

High fowler’s (Correct Answer)

Report an issue

Which additional nursing action is most


important prior to administering the
medicine?

Request the patient urinate

Perform blood typing and cross


matching

Ensure the consent form has been


signed (Correct Answer)

Clarify contact numbers of her


family members

Report an issue

file:///Users/mohammad/Desktop/PrometricMCQ.com.webarchive Page 31 of 377


PrometricMCQ.com 11/03/2015 10:45

A 24 year-old man was admitted to the


hospital after receiving a closed head injury
in a motor vehicle accident. He had a
Glasgow Coma Score of 6 on admission
and was being mechanically ventilated.
The nurse observes the intracranial
pressure has slowly and steadily increased
over the past hour.
What is the most appropriate initial nursing
action?

Administer mannitol

Administer Phenobarbital

Increase respiratory rate

Reposition while maintaining neck


alignment (Correct Answer)

Report an issue

The nurse is caring for a 4 year-old patient


with a diagnosis of cystic fibrosis and
pneumonia. The child is feeling better on
the 3rd day of the hospitalization and
“wants to play”
What would be the BEST choice of
entertainment?

Blowing bubbles (Correct Answer)

Looking at picture books

Watching videos

file:///Users/mohammad/Desktop/PrometricMCQ.com.webarchive Page 32 of 377


PrometricMCQ.com 11/03/2015 10:45

Riding in a wagon

Report an issue

During the immediate postoperative


period, a patient reveals an oxygen
saturation level of 91%. The nurse should

Position the patient on the left side

Administer supplemental oxygen


(Correct Answer)

Continue to provide supportive care

Lower the temperature of the room

Report an issue

An 83 year-old woman lives in a long term


care facility. During the past two weeks,
she has suffered two falls that resulted in
minor injuries and now complains of pain in
the left shoulder. On auscultation, bowel
sounds are diminished in all four
quadrants. On palpation, she has
tenderness and guarding over the upper
left quadrant and complain of a feeling of
fullness. What could be the most likely
underlying cause of the finding?

Constipation

Liver distention

file:///Users/mohammad/Desktop/PrometricMCQ.com.webarchive Page 33 of 377


PrometricMCQ.com 11/03/2015 10:45

Splenic rupture (Correct Answer)

Intestinal obstruction

Report an issue

In evaluating a patient after a


cerebrovascular accident (CVA), which of
the following is an expected outcome of
the nursing diagnosis of impaired physical
mobility related to motor deficits?

Oriented to person, lace, and time

Maintains body alignment, no


contractures (Correct Answer)

Communicates appropriately

Voids on command at 2-hour


intervals

Report an issue

A patient who has a diagnosis of multiple


sclerosis speaks slowly with long pauses
between syllables.
The patient is MOST likely in what stage of
multiple sclerosis?

file:///Users/mohammad/Desktop/PrometricMCQ.com.webarchive Page 34 of 377


PrometricMCQ.com 11/03/2015 10:45

Prodromal

Early

Middle

Late (Correct Answer)

Report an issue

A 70 year-old quadriplegic man has


ulceration on the left buttocks with full
thickness skin loss. Although several
interventions have been used to promote
healing, the ulcer does not improve (see
image)
Which dietary insufficiency is most likely?

Vitamin D

Calcium

Protein (Correct Answer)

Vitamin C

file:///Users/mohammad/Desktop/PrometricMCQ.com.webarchive Page 35 of 377


PrometricMCQ.com 11/03/2015 10:45

Report an issue

In planning a lecture for a community


group effect of acquired immunodeficiency
syndrome gerontological community, the
nurse would be correct in including which
of the following stat regarding the
incidence of the illness in people be 55 and
64 years of age?

The illness is almost unheard of in


this age group prior to 2003

The illness has more than doubled


between 1998 and 2003 (Correct
Answer)

There are not many people living


with the disease as it quickly fatal

Treatment is more effective in this


age group

Report an issue

What is the BEST nursing intervention?

Instruct the caretaker to change the


patient’s position every 2
hours (Correct Answer)

Apply hydrogel to the stage I


pressure sore every 8 hours

file:///Users/mohammad/Desktop/PrometricMCQ.com.webarchive Page 36 of 377


PrometricMCQ.com 11/03/2015 10:45

Refer the patient to a wound care


specialist for debridement

Encourage the patient to consume


an increased amount of calcium

Report an issue

A nurse is caring for an infant with


respiratory distress syndrome.
Which of the following nursing intervention
is appropriate?

Measure oxygen saturation level


once a shift

Suction frequently for 30-45


seconds each time (Correct
Answer)

Monitor for symptoms of


hyperglycemia

Maintain infant temperature 36.7°


and 37.8°C

Report an issue

A patient who sustained extensive


abdominal injuries in a motor vehicle
accident has developed a large stage II
pressure ulcer on the coccyx. A new
diagnosis of alteration in skin integrity is
added to the care plan.
What is the BEST short-term goal for the

file:///Users/mohammad/Desktop/PrometricMCQ.com.webarchive Page 37 of 377


PrometricMCQ.com 11/03/2015 10:45

patient?

Show evidence of healing within one


week

Have no discomfort from the


pressure ulcer

Eat at least 50% of each meal

Verbalize strategies to prevent


further skin breakdown (Correct
Answer)

Report an issue

A nursing home resident with chronic


osteoarthritis complains of knee pain. A
routine order of acetaminophen (Tylenol)
500 mg every 6 hours was administered 2
hours ago with no relief. The patient also
has an order for ibuprofen (Motrin) 400 mg
every 6 hours as needed. The patient
reports some relief, but is still bothered by
mild pain. The nurse should

Administer ibuprofen (Motrin)


400mg (Correct Answer)

Educate the patient that mild pain is


expected with osteoarthritis

Call the physician for additional pain


medication

Administer acetaminophen (Tylenol)


500 mg

file:///Users/mohammad/Desktop/PrometricMCQ.com.webarchive Page 38 of 377


PrometricMCQ.com 11/03/2015 10:45

Report an issue

A 55 year-old man has become very


anxious about skin lesions he has
developed. On the lower right forearm,
there is a well demarcated round patch of
skin that he feels could be cancerous. It is
2.5 centimeters in diameter and slightly
raised. On palpation it is scaly, dry and
rough. The affected area appears sun
tanned and reddened. The condition has
been persistent for the past four years but
has only recently become itchy.
What is the most likely underlying
problem?

Seborrheic keratosis (Correct


Answer)

Actinic keratosis

Eczematous dermatitis

Lupus erythematosus

Report an issue

A nurse is assessing several patients who


are over the age of 65 years.
Which question would demonstrate the
nurse’s understanding of changes
associated with the normal process of
aging?

Patient A

file:///Users/mohammad/Desktop/PrometricMCQ.com.webarchive Page 39 of 377


PrometricMCQ.com 11/03/2015 10:45

Patient B

Patient C

Patient D (Correct Answer)

Report an issue

A 65 year-old woman with a history of


unstable angina and hypertension presents
to the Emergency Department with a dull
chest pain that she describes as similar to
heartburn. The pain radiates down the left
arm. She had taken sublingual nitroglycerin
tablets without any relief. An
electrocardiograph is performed and
shows elevated S-T segments. The nurse
is to administer a thrombolytic by
intravenous infusion.
Which factor places this candidate at high
risk for bleeding?

Unstable angina

Hypertension (Correct Answer)

Age

Elevated S-T segments

Report an issue

The following blood pressures have been


documented in the patient’s chart for the
past six months; 124/76 mmHg, 136/83
mmHg and 128/80 mmHg. The Most likely
file:///Users/mohammad/Desktop/PrometricMCQ.com.webarchive Page 40 of 377
PrometricMCQ.com 11/03/2015 10:45

diagnosis is:

Normal blood pressure

Prehypertension (Correct Answer)

Stage 1 hypertension

Stage 2 hypertension

Report an issue

The nurse is performing a routine


examination for a 24 year-old man with
diabetes mellitus. He takes insulin
injections twice daily and monitors his
blood sugar regularly. He is vegetarian with
a body mass index of 17. The nurse is
concerned about his weight.
Which additional information is most likely
required to calculate his ideal daily caloric
intake?

Daily elimination pattern

Protein and carbohydrate intake

Activity and exercise level (Correct


Answer)

HDL and LDL cholesterol levels

Report an issue

file:///Users/mohammad/Desktop/PrometricMCQ.com.webarchive Page 41 of 377


PrometricMCQ.com 11/03/2015 10:45

A nurse is assessing a child with


suspected lead poisoning.
For which of the following should the nurse
assess?

Developmental delay, constipation,


anorexia, clumsiness, and pallor
(Correct Answer)

Confusion, delirium, seizures,


lethargy, stupor, and coma

Respiratory depression, obstruction,


pulmonary edema, and tachypnea

Peripheral circulatory collapse and


heart failure

Report an issue

The following pain medications are ordered


for a patient who had a right leg
amputation. Oxycodone 5 mg every 4
hours as needed and morphine 5 mg every
4 hours as needed. The nurse administered
oxycodone 2 hours ago, but the patient
reports pain rated 8 on a scale of 0 (no
pain) to 10 (severe pain) as the dressing
change beings.
After evaluating the effectiveness of the
pain medication, what action should the
nurse take?

Administer additional oxycodone 5


mg

Administer morphine 5 mg
(Correct Answer)

Change the dressing quickly

file:///Users/mohammad/Desktop/PrometricMCQ.com.webarchive Page 42 of 377


PrometricMCQ.com 11/03/2015 10:45

Encourage deep breathing

Report an issue

A 72 year-old woman who is alert and


oriented has been hospitalized with atrial
fibrillation. Since admission four days ago,
she has been receiving digoxin intravenous
injection. Today is the first day the digoxin
injection will be discontinued and she will
change to oral tablets. Although she
acknowledges the medication benefits, she
refuses to take it as it makes her feel
nauseous.
What is the most appropriate nursing
action?

Continue administering by
intravenous push

Withhold medication and notify the


physician (Correct Answer)

Withhold medication and file an


incident report

Provide it in liquid from mixed into a


drink

Report an issue

The district nurse visits a 30 year-old


woman at home following the delivery of
her second child, a full term girl. Following
the delivery of her first child, she had

file:///Users/mohammad/Desktop/PrometricMCQ.com.webarchive Page 43 of 377


PrometricMCQ.com 11/03/2015 10:45

developed a breast infection and stopped


breastfeeding because of the pain. She
asks the nurse how she can best prevent it
with this infant.
What is the most appropriate response?

Provide feedings on demand

Apply vitamin E cream to the nipples

Request a prophylactic antibiotic

Apply heat to the breasts after


feedings (Correct Answer)

Report an issue

A patient has come to the clinic for follow-


up one week after being discharged from
the hospital for treatment of a hypertensive
crisis. Blood pressure stabilized at 124/78
mm hg. The patient report feeling well, has
no edema, and no longer has daily
headaches. Blood pressure is 156/90
mmHg. During evaluation the patient
admits to having stopped taking
medication that had been ordered because
headaches are no longer present. Unless
the symptoms return, the patient states
they will not be returning to the clinic.
What should the nurse do?

Resolve and discontinue the entire


care plan per patient request,
suggest psychology consult

Resolve B and D, continue A and C

Continue entire care plan as written

file:///Users/mohammad/Desktop/PrometricMCQ.com.webarchive Page 44 of 377


PrometricMCQ.com 11/03/2015 10:45

Add a new problem to the care plan,


non-compliance and interventions to
determine potential (Correct
Answer)

Report an issue

A patient has returned to the medical-


surgical unit after surgery for a benign
ovarian tumor. The nurse observes that in
the first 30 minutes after admission to the
unit, the patient’s abdomen is rapidly
distending and the patient complains of
increasing pain. The nurse would
anticipate.

Aspiration pneumonia

Surgical site infection

Retention of surgical supplies

Intra-abdominal hemorrhage
(Correct Answer)

Report an issue

A nurse is caring for a postoperative


patient who is on subcutaneous, low-dose
heparin. When administering an injection
on the abdomen, the nurse avoids the
umbilicus area because of the possibility
of.

file:///Users/mohammad/Desktop/PrometricMCQ.com.webarchive Page 45 of 377


PrometricMCQ.com 11/03/2015 10:45

Entering a larger blood vessel


(Correct Answer)

Causing increased pain

Precipitating hyperventilation

Umbilical infection

Report an issue

The nurse caring for a mobile patient with a


destroyed tympanic membrane and an
intact stapes.
What type of repair process should the
nurse anticipate?

Myringoplasty (Correct Answer)

Tymoanostapedopexy

Cavum minor

Tympanoplasty and fenestration of


lateral semicircular canal

Report an issue

An 82 year-old woman with Alzheimer’s


disease had moved into a long-term care
facility two weeks previously. Since then,
the staff has found her wondering in the
hallways in middle of the night. When
approached, she is confused and
frustrated, often forgetting where she is.
Which intervention would most likely
file:///Users/mohammad/Desktop/PrometricMCQ.com.webarchive Page 46 of 377
PrometricMCQ.com 11/03/2015 10:45

decrease the patient’s confusion?

Administer a sleeping sedative


(Correct Answer)

Provide full-time nursing care

Place a nightlight in the room

Provide a large meal before bed

Report an issue

A 25 year-old woman resents to the clinic


with abdominal pain. She reports the last
bowel movement was two days previously.
There is no vomiting or diarrhea and her
body temperature is normal. During
percussion of the lower left quadrant the
nurse hears tympanic sounds.
What is the most likely underlying cause
for the finding?

Gas (Correct Answer)

Tumor

Liquid

Feces

Report an issue

What type of lifelong diet would be most

file:///Users/mohammad/Desktop/PrometricMCQ.com.webarchive Page 47 of 377


PrometricMCQ.com 11/03/2015 10:45

beneficial for this patient?

Increased magnesium and vitamin C

Increased calcium and vitamin D


(Correct Answer)

Increased dairy and iron

High protein and high calorie

Report an issue

A community is experiencing an outbreak


of staphylococcal infections. The nurse
instructs residents that the MOST common
mode of transmission is by

Respiratory droplets

Contaminated foods

Hands (Correct Answer)

Soil

Report an issue

file:///Users/mohammad/Desktop/PrometricMCQ.com.webarchive Page 48 of 377


PrometricMCQ.com 11/03/2015 10:45

A home care nurse changes the dressing


of a patient who has a chronic lower leg
ulcer and a history of leg swelling. The
wound bed is 3 cm diameter, red in color
and is covered with fibrous yellow tissue
(see image)
What type of ulcer is most likely?

Arterial

Venous (Correct Answer)

Decubitus

Neuropathic

Report an issue

The home care nurse visits a diabetic


patient, recently started on sliding scale
insulin injection. The patient is confused
about how many units of regular insulin to
inject. During the home visit the patient’s
glucose level before their meal is 301
mg/dL
How many units of regular insulin should
the nurse instruct the patient to
administer?

2 units

4 units

6 units (Correct Answer)

8 units

file:///Users/mohammad/Desktop/PrometricMCQ.com.webarchive Page 49 of 377


PrometricMCQ.com 11/03/2015 10:45

Report an issue

A 30 year-old woman has been prescribed


albuterol Pð​‘µð​‘¹ð​Ÿ​and prophylactic
inhaled corticosteroids to be taken once
per day. She has been taking the
prophylactic as prescribed but has needed
to use the albuterol more often than usual.
She has a chronic cough and often has air
hunger.
Which intervention is initially most
appropriate?

Refer for a chest X-ray

Administer magnesium sulfate

Assess peak flow measurement

Perform arterial blood gas analysis


(Correct Answer)

Report an issue

A first time mother of a three week-old


breastfed baby brings the infant to the
clinic and complains that her child has
been forcefully vomiting after feeding. He
was born at 40 weeks gestation, weighing
3.6 kilograms. He is constantly hungry and
irritable. Examination reveals a swollen
abdomen and a palpable mass in the
middle upper right quadrant.
What is the most likely health problem?

file:///Users/mohammad/Desktop/PrometricMCQ.com.webarchive Page 50 of 377


PrometricMCQ.com 11/03/2015 10:45

Intussusception

Pyloric stenosis (Correct Answer)

Gastroeosophageal reflux

Diaphragmatic hernia

Report an issue

A 40 year-old man present to the clinic with


a weight loss of three kilograms over the
past month. His health problem produces
moderate spastic body movements, hyper-
reflexia, muscular contractions followed by
relaxation, and a positive Babinski reflex.
He has difficulty speaking due to poor
muscle tone and has recently developed
swallowing difficulties. The nurse creates a
care plan to address his developing
problem.
Which is the priority nursing intervention?

Increase caloric intake

Aspiration prevention (Correct


Answer)

Infection control

Improved muscle control

Report an issue

A 62 year-old diabetes type II patient is


admitted to hospital for ampulation of the

file:///Users/mohammad/Desktop/PrometricMCQ.com.webarchive Page 51 of 377


PrometricMCQ.com 11/03/2015 10:45

right foot due to gangrene and ischemia,


Pre-operatively, the patient rates the pain
as 9 (using the 1-10 pain scale). How can
this patient’s phantom (sprit) pain best be
controlled?

Post-operative elevation of limp

Apply pressure bandage to stump

Control pain pre-operatively


(Correct Answer)

Apply ice to the site for twenty


minutes

Report an issue

The nurse cares for a 60 year-old woman


has a history of hypertension,
hypothyroidism and elevated cholesterol
levels. She takes tablets daily for each of
the health problems. The doctor orders a
routine dual-energy X-ray absorptimetry
test shows decreased bone density.
Which medication most likely contributed
to the test results?

Statins

Anti-hypertensive

Synthetic thyroid hormones


(Correct Answer)

Cholesterol absorption inhibitors

Report an issue

file:///Users/mohammad/Desktop/PrometricMCQ.com.webarchive Page 52 of 377


PrometricMCQ.com 11/03/2015 10:45

A 32 year-old woman with diabetes


mellitus type 1 underwent a
cholecystectomy and is now on day two of
recovery. The patient’s bowel sounds have
returned and she has resumed a normal
diet but has been finishing less than half of
each meal on the tray. The nurse enters the
room to perform a routine assessment and
finds the patient confused and shaky
Blood pressure 110/60 mmHg Heart rate
96/min Respiratory rate 22/mm
Temperature 37.0°C
What is the most appropriate initial
intervention?

Administer glucagon

Notify the physician (Correct


Answer)

Given an insulin injection

Offer a glass of juice

Report an issue

Prior to providing care for a hospitalized


infant, the nurse who focuses on
preventive measures must.

Introduce self to parent

Perform hand hygiene (Correct


Answer)

Have a witness present

file:///Users/mohammad/Desktop/PrometricMCQ.com.webarchive Page 53 of 377


PrometricMCQ.com 11/03/2015 10:45

Assess the child’s


developmental level

Report an issue

A nurse is admitting a two year-old child


with an umbilical hernia.
Which of the following interventions does
NOT meet the child’s developmental
needs?

Allowing the child to make choices


when possible

Providing rooming in and unlimited


visitation

Attempting to continue rituals used


house

Maintaining strict bed rest (Correct


Answer)

Report an issue

A woman who has been discharged from


hospital with a permanent colostomy in
lace is performing irrigation at home for the
first time. After five minutes from the start
of the procedure, she begins to have
stomach cramps.
Which intervention would most likely
decrease the symptoms?

file:///Users/mohammad/Desktop/PrometricMCQ.com.webarchive Page 54 of 377


PrometricMCQ.com 11/03/2015 10:45

Increase the flow of irrigation


solution

Decrease the temperature of the


water

Clamp the catheter for 1-2


minutes (Correct Answer)

Elevate the bag of irrigation solution

Report an issue

A 17 year-old is brought to the clinic by her


mother. She complains of episodes during
which her fingers become cold, numb and
white. After this subsides, the fingers turn
red, swell and cause severe and throbbing
pain. On examination, the fingers all
appear normal and the patient has no pain.
The nurse performs a history assessment
to identify potential causes.
Which trigger is most likely?

Excessive exercise

Ingestion of tea

Exposure to cold (Correct Answer)

Washing up

Report an issue

A patient on a medical floor is diagnosed


with pneumonia.

file:///Users/mohammad/Desktop/PrometricMCQ.com.webarchive Page 55 of 377


PrometricMCQ.com 11/03/2015 10:45

Which of the following interventions is


appropriate for a nursing diagnosis of
ineffective airway clearance related to
tracheobronchial secretions secondary to
infection?

Explain diagnostic procedures and


address any of patient’s
concerns

Assess patient for signs of


discomfort and pain

Monitor oxygen therapy and


encourage to cough and deep
breathe (Correct Answer)

Monitor patient for shock

Report an issue

The nurse is inserting a nasogastric (NG)


tube into a patient as prescribed. The
nurse has advanced the tube into the
patient’s posterior pharynx. The nurse
should now ask the patient to.

Hold the breath

Stare upwards with the eyes


towards the ceiling

Perform the Valsalva maneuver

Lower the chin towards the chest


(Correct Answer)

Report an issue

file:///Users/mohammad/Desktop/PrometricMCQ.com.webarchive Page 56 of 377


PrometricMCQ.com 11/03/2015 10:45

A community is experiencing an outbreak


of staphylococcal infections. The nurse
instructs residents that the MOST common
mode of transmission is by:

Respiratory droplets

Contaminated foods

Hands (Correct Answer)

Soil

Report an issue

A patient with bacterial pneumonia reports


increased pain during deep breathing and
coughing exercises. To minimize the
patient’s pain, the nurse should teach the
patient to.

Take a cough suppressant


prescribed P.R.N at regular time
intervals

Turn onto the side before doing the


respiratory exercises

Hold a pillow tightly against the


chest while coughing

Drink warm liquids right before


taking deep breaths (Correct
Answer)

file:///Users/mohammad/Desktop/PrometricMCQ.com.webarchive Page 57 of 377


PrometricMCQ.com 11/03/2015 10:45

Report an issue

A patient with a history of chronic


obstructive pulmonary disease prepares for
a colonoscopy with conserve sedation.
Due to the patient’s history the nurse
should very cautious about administering.

Antibiotics

Sedation (Correct Answer)

Oxygen

Intravenous fluid

Report an issue

A 45 year-old woman is receiving


chemotherapy for breast cancer. Two
weeks after the initial treatment she
telephoned the nurse at the cancer center
and reports she has hair loss, nausea,
tiredness, a body temperature is 38.1 C,
and air hunger
Which finding most likely indicates she
needs to report the clinic?

Pyrexia

Nausea

Hair loss

Air hunger (Correct Answer)

file:///Users/mohammad/Desktop/PrometricMCQ.com.webarchive Page 58 of 377


PrometricMCQ.com 11/03/2015 10:45

Report an issue

A patient present to the clinic voicing a


concern above being exposed to hepatitis
A (HAV) one week ago. Upon questioning,
the nurse finds the patient purchased for
from a person recently diagnosed with
HAV. The number would be MOST correct
when instructing the patient

The incubation period is 3 to 5


weeks (Correct Answer)

HAV is spread by sexual


transmission

HAV is spread by blood contact

The incubation period is 2 to 5


months

Report an issue

A gravid 3, para 2 presents to the Labor


and Delivery Unit with contractions
occurring every 3-5 minutes and lasting for
60-90 seconds. During contractions, she
closed her eyes and is focused on
breathing to cope with the pain. At the end
of each contraction she opens her eyes
and resumes talking with the nurse and her
husband.
What is the expected cervical dilatation in
centimeters?

file:///Users/mohammad/Desktop/PrometricMCQ.com.webarchive Page 59 of 377


PrometricMCQ.com 11/03/2015 10:45

1-3

4-7

8-10 (Correct Answer)

11-13

Report an issue

The nurse is preparing a discharge plane


for a 65 year-old man with a new diagnosis
of congestive heart failure. The discharge
orders include furosemide 40 milligrams by
mouth twice per day. The nurse
recommends foods to reduce unwanted
medication side effects.
Which of the following foods would be
most appropriate to include in the teaching
plan?

Green and leafy vegetables

Bananas and oranges (Correct


Answer)

Chicken and Fish

Report an issue

patient who is scheduled for a


tonsillectomy is in the preoperative unit.
The nurse notes an order for preanesthetic

file:///Users/mohammad/Desktop/PrometricMCQ.com.webarchive Page 60 of 377


PrometricMCQ.com 11/03/2015 10:45

medication to be given “on call to


operating room.” The nurse should give
this medication.

Immediately upon being notified to


prepare the patient for transport
(Correct Answer)

When the operating room staff


arrives to transport the patient

Only if clearly needed after

Report an issue

Refer to the accompanying figure.


Which of the following goals should receive
PRIORITY during the shift?

The patient will rest for 6 hours


without interruption

The patient will consume 90% of


each meal

The patient will perform deep


breathing and coughing exercises
(Correct Answer)

The patient will drink 1920 ml (64


ounces of fluid)

Report an issue

file:///Users/mohammad/Desktop/PrometricMCQ.com.webarchive Page 61 of 377


PrometricMCQ.com 11/03/2015 10:45

Which lifestyle modification would most


likely decrease this patient risk factor?

Decrease stress

Increase exercise (Correct Answer)

Report an issue

A 66 year-old woman is admitted to the


hospital with a history hypertension. She
present with breathing difficulties that
worsen with activity and while sleeping,
she is generally weak and feels that her
heart misses beats and that it sometimes
beats loudly. An electrocardiogram shows
atrial fibrillation, right ventricular
hypertrophy and deviation towards the
right.
Which nursing intervention is most
appropriate for this patient?

Provide a bedside commode


(portable toilet) (Correct Answer)

Place in right side lying position

Encourage family and friends to visit

Encourage independent hygienic


activities

Report an issue

file:///Users/mohammad/Desktop/PrometricMCQ.com.webarchive Page 62 of 377


PrometricMCQ.com 11/03/2015 10:45

66 year-old woman is admitted to the


hospital with a history hypertension. She
present with breathing difficulties that
worsen with activity and while sleeping,
she is generally weak and feels that her
heart misses beats An electrocardiogram
Conform atrial fibrillation, right ventricular
hypertrophy and deviation towards the
right
What was the most likely origin of the
disorder?

Hypertension (Correct Answer)

Rheumatic fever

Atherosclerosis

Genetic predisposition

Report an issue

A 76 year-old woman present with facial


drooping on the left side and muscle
weakness of the left arm and leg after
suffering a fall at home.
Which type of paresis is most likely?

Monoparesis

Paraparesis

Hemiparesis (Correct Answer)

Quadriparesis

Report an issue

file:///Users/mohammad/Desktop/PrometricMCQ.com.webarchive Page 63 of 377


PrometricMCQ.com 11/03/2015 10:45

The nurse has administered an intravenous


infusion of Cyclophosphamide to a 43
year-old patient with breast cancer. The
nurse formulates a care plan with the
nursing problem of risk for infection related
to leucopenia secondary to chemotherapy.
The patient is discharged home following
treatment.
Which findings should the patient report
immediately?

Dyspnea

Tachycardia

Sore throat (Correct Answer)

Discoloration of urine

Report an issue

When given postoperative discharge


instruction to a patient who had abdominal
surgery, all of the following regarding
wound healing is true EXPECT

Bathing to soak abdomen is


preferred over showering (Correct
Answer)

Avoid tight belts and clothes with


seams that may rub the wound

Pain medication may affect ability to


drive

file:///Users/mohammad/Desktop/PrometricMCQ.com.webarchive Page 64 of 377


PrometricMCQ.com 11/03/2015 10:45

Irregular bowel habits can be


expected

Report an issue

An eight year-old boy has a new diagnosis


of Attention Deficit Hyperactive Disorder
(ADHD) and begins pharmacological and
psychological therapies. His medication
regimen includes short-acting
methylphenidate 5 mg orally every 12
hours.
Which side effect is most likely?

Insomnia (Correct Answer)

Initial increased hyperactivity

Lethargy

Hypertension

Report an issue

The nurse is caring for a full-term new born


who was delivered vaginally 5 minutes ago.
The infant’s APGAR score was 8 at one
minute and 10 at 5 minutes.
Which of the following has the highest
PRIORITY?

Maintaining the infant in the supine


position

file:///Users/mohammad/Desktop/PrometricMCQ.com.webarchive Page 65 of 377


PrometricMCQ.com 11/03/2015 10:45

Assessing the infant’s red reflex

Preventing heat loss from the


infant (Correct Answer)

Administering humidified oxygen to


the infant

Report an issue

The district nurse attends a 73 year-old


woman on a daily basis. The woman’s
husband had passed away three years
previously and she now lives alone.
Besides assisting in activities of daily living
the nurse encourages the patient to talk
about her past experiences and events.
Which theoretical frame work is most likely
guiding the nurse’s care plan?

Erickson’s model of
psychosocial development

Piaget’s theory of cognitive


development

Watson’s theory of human


caring

Roy’s adaptation model


(Correct Answer)

Report an issue

file:///Users/mohammad/Desktop/PrometricMCQ.com.webarchive Page 66 of 377


PrometricMCQ.com 11/03/2015 10:45

A home care patient with chronic


obstructive pulmonary disease (COPD)
reports an upset stomach. The patient is
taking theophylline (Theo-Dur) and
triamcinolone acetonide (Azmacort), the
nurse should instruct the patient to take

Theo-Dur on an empty stomach

Theo-Dur and Azmacort at the same


time

Theo-Dur and Azmacort 12 hours


apart

Theo-Dur with milk or crackers


(Correct Answer)

Report an issue

A 64 year-old woman with hepatitis B has


significant scarring on the liver and present
to the clinic with swelling around the eyes.
She is disoriented to time and place and
seems confused. Her responses are slow
and she has difficulty staying awake. A
neurological assessment reveals
hypoactive reflexes. Blood pressure 164/96
mmHg Heart rate 58/min Respiratory rate
22/min shallow Temperature 37.0°C
tympanic
Which of the following nursing
interventions is most appropriate?

Prevent aspiration (Correct


Answer)

Encourage small meals

file:///Users/mohammad/Desktop/PrometricMCQ.com.webarchive Page 67 of 377


PrometricMCQ.com 11/03/2015 10:45

Administer fluid bolus

Inject

Report an issue

A patient is being followed in the clinic for


hypertension, adult onset diabetes, and
obesity. The patient is apathetic about
learning about nutritional guidelines to
reach the goals of weight loss and
consumption of a healthy diet. The patient
admitted to eating “whatever is put in front
of me”
Which of the following actions would the
nurse take?

Collaborate with the patient to set


goals

Add a nursing diagnosis of non-


compliance

Refer for psychiatric screening for


depression

Discuss nutritional interventions with


the spouse (Correct Answer)

Report an issue

A 16 year-old boy is in the Post-Operative


Care Unit two hours after a tonsillectomy.
He is alert and oriented but complains of
severe throat pain and difficulty
swallowing. He rates the pain at a level 7,
file:///Users/mohammad/Desktop/PrometricMCQ.com.webarchive Page 68 of 377
PrometricMCQ.com 11/03/2015 10:45

on a scale of 1-10. The urine output from


the folly catheter is 45 ml over the past two
hours. Blood pressure 130/74 mmHg Heart
rate 64/min Respiratory rate 18/min
Oxygen saturation 98% on room air
Which clinical finding is most important to
report to the doctor?

Oxygen saturation

Difficulty swallowing

Urinary output (Correct Answer)

Pain level

Report an issue

A patient is being treated for pneumonia.


Considering Maslow’s hierarchy of needs,
which of the following nursing diagnoses
would have HIGHEST priority?

Risk for impaired skin integrity

Ineffective coughing. (Correct


Answer)

Disturbed sleep pattern

Ineffective breathing pattern

Report an issue

file:///Users/mohammad/Desktop/PrometricMCQ.com.webarchive Page 69 of 377


PrometricMCQ.com 11/03/2015 10:45

An insulin dependent diabetic patient is


admitted to the hospital in ketoacidosis.
Of the nursing diagnoses shown in the
table which is the two with the HIGHEST
priority?

A, C

E, F (Correct Answer)

D, E

B, E

Report an issue

A patient who is receiving chemotherapy


has a platelet count of 49,000/ð​​¦ð​​¦ð​Ÿ‘
(normal value 150,000 to 400,000/ð​​¦ð​​¦ð​Ÿ‘)
Which of the following nursing action is
necessary?

Minimize invasive procedures


(Correct Answer)

Crush oral medications

Limit intake of vitamin k rich foods

Monitor the temperature every 4


hours

Report an issue

file:///Users/mohammad/Desktop/PrometricMCQ.com.webarchive Page 70 of 377


PrometricMCQ.com 11/03/2015 10:45

A child present to the emergency


department with complaint of chest pain
on exertion, shortness of breath, and
dizziness. Upon examination the nurse
notes palpitation and a systolic ejection
murmur at the right upper sterna border.
Skin color is normal. The nurse suspect.

Aortic stenosis

Aortic regurgitation (Correct


Answer)

Mitral stenosis

Mitral valve prolapse

Report an issue

A patient is on oxygen using a nasal


cannula at home. The doctor has ordered
this patient to receive oxygen delivered at
a concentration of 35 %. The nurse would
set the oxygen at:

2L

3L

4L (Correct Answer)

5L

Report an issue

file:///Users/mohammad/Desktop/PrometricMCQ.com.webarchive Page 71 of 377


PrometricMCQ.com 11/03/2015 10:45

A nurse who is assigned to care for a client


with dementia and agitated behaviors
knows that the client worked as a
housekeeper prior to becoming ill,
Which of the following activities is MOST
likely to help the client if agitation
develops?

Watching a movie about a house


keeper

Reading a sheet of paper that


contains stains removing tips

Dusting furniture in the assigned


room with a cloth (Correct Answer)

Singing songs about cleaning

Report an issue

A diabetic patient with an infected right


second toe is treated with a 14-day course
of antibiotics. The patient returns for a
follow- up visit after finishing the
prescribed course of antibiotics.
What can be inferred from the
accompanying blood laboratory result
drawn at the follow-up visit?

Diabetes is effectively managed but


toe infection is not managed well

Diabetes and toe infection are


effectively managed

Diabetes and toe infection are

file:///Users/mohammad/Desktop/PrometricMCQ.com.webarchive Page 72 of 377


PrometricMCQ.com 11/03/2015 10:45

effectively managed but toe


infection is effectively managed

Diabetes and toe infection are not


effectively managed (Correct
Answer)

Report an issue

A 60 year-old woman, with no previous


medical history is diagnosed with an
adenocarcinoma of the duodenum head of
the pancreas and the distal end of the bile
duct. Staging shows it has not grown into
the large blood vessels and has not
metastasized to any lymph node or distant
site. The patient undergoes surgery.
Which post-operative clinical finding is
most likely?

Increased serum glucose (Correct


Answer)

Constipation

Weight loss >15%

Paralytic ileus

Report an issue

A 62 year-old patient has been treated for


congestive heart failure and a nursing
diagnosis of fluid volume excess. After
diuretic therapy and dietary interventions,
the patient has met all short-term goals.
file:///Users/mohammad/Desktop/PrometricMCQ.com.webarchive Page 73 of 377
PrometricMCQ.com 11/03/2015 10:45

The nurse should.

Revise the care plan with a


diagnosis of risk for alteration in fluid
balance (Correct Answer)

Add a new diagnosis of risk of fluid


volume deficit

Discontinue the care plane as the


diagnosis is resolved

Continue the care plan as written

Report an issue

The nurse assists with a lumbar puncture


on a child with suspected bacterial
meningitis. If the diagnosis is correct the
cerebrospinal fluid should have which of
the following qualities?

Pink viscous appearance

Clear fluid appearance

Turbid appearance (Correct


Answer)

Blood-tinged appearance

Report an issue

file:///Users/mohammad/Desktop/PrometricMCQ.com.webarchive Page 74 of 377


PrometricMCQ.com 11/03/2015 10:45

A patient presents with pneumonia. Which


of the following intervention would the
nurse implement to improve the patient’s
gas exchange? Heart rate 80/min
Respiratory rate 32/min Temperature 40°C

Encourage the patient to increase


fluid intake (Correct Answer)

Position the patient in an upright


position

Instruct the patient the patient to


splint when coughing

Apply heat and or cold to the chest


as ordered

Report an issue

While providing care to a hospitalized child


with third-degree burns over 20% of the
body, the child rapidly becomes
disoriented. The nurse knows this is an
indicator of:

Septic shock

Separation anxiety

Dehydration (Correct Answer)

Depression

Report an issue

file:///Users/mohammad/Desktop/PrometricMCQ.com.webarchive Page 75 of 377


PrometricMCQ.com 11/03/2015 10:45

A brick-layer has been diagnosed with


multiple sclera. The nurse knows that
construction work places has increased
risk of aggravated symptoms because
exposure to.

Dust

Sunlight

Heat (Correct Answer)

Heights

Report an issue

A 68 year-old male patient has a chest


tube and properly fixed following cardiac
surgery. The chest tube is patent and
functioning. When the nurse removes the
dressing the tube falls out of the chest.
Which is the most appropriate initial
intervention the nurse should take?

Reinsert the chest tube and notify


the surgeon

Apply an occlusive dressing to the


insertion site (Correct Answer)

Place the open end of the tube in 20


cm of water

Administer ð​‘‚2at 10L/min via non-


rebreather mask

file:///Users/mohammad/Desktop/PrometricMCQ.com.webarchive Page 76 of 377


PrometricMCQ.com 11/03/2015 10:45

Report an issue

During surgery requiring general


anesthesia, the patient’s heart stops and a
carotid pulse is not palpated.
How many compressions per minute
should be administered?

50

60

80

100 (Correct Answer)

Report an issue

A 64 year-old man present with a weight


gain of four kilograms in the past week. A
physical examination shows an enlarged
liver, an enlarged abdomen with a fluid
wave, and jugular venous distension. The
nurse creates a care plan based on the
following nursing diagnosis. Decreased
cardiac output related to ventricular
damage, ischemia and restriction
secondary to fluid overload.
Which would be an appropriate defining
characteristic?

Altered preload

Altered afterload

file:///Users/mohammad/Desktop/PrometricMCQ.com.webarchive Page 77 of 377


PrometricMCQ.com 11/03/2015 10:45

Poor ventricular contractility


(Correct Answer)

Increased pulmonary congestion

Report an issue

After the nurse has administered a heparin


injection. The patient is observed rubbing
the area. The nurse should instruct the
patient to not rub the area, as it may

Increase the risk of bleeding.


(Correct Answer)

Prevent proper absorption of the


medication.

Cause increased pain.

Result in tissue death.

Report an issue

What type of treatment is most appropriate


for this patient?

Unfractured heparin

Anti-embolic stocking (Correct


Answer)

Warm compresses and heating pads

file:///Users/mohammad/Desktop/PrometricMCQ.com.webarchive Page 78 of 377


PrometricMCQ.com 11/03/2015 10:45

Anti-inflammatory medications

Report an issue

A hospitalized patient eats 20% of the


meal and states being too tired to eat more
.What should the nurse do?

Offer to feed the patient after short


rest period. (Correct Answer)

Encourage the patient to finish the


fluids.

Remove the meal tray and allow the


patient to rest.

Encourage the patient to finish the


protein portion of the meal

Report an issue

A 6-month-old boy is admitted with failure


to thrive
Based on his growth chart the nurse
should consider
Which of the following as the cause of the
child's failure
to thrive?

file:///Users/mohammad/Desktop/PrometricMCQ.com.webarchive Page 79 of 377


PrometricMCQ.com 11/03/2015 10:45

Parent-child interaction

Malrotation of the colon

Genetic chromosomal abnormalities

Congenital heart disease (Correct


Answer)

Report an issue

A 40 year-old woman presents with right


hip pain palpation of the pelvic girdle is
normal. An X-ray shows bone deformities,
with osteolytic lesions and bone
enlargement. The patient has not suffered
trauma and has been generally health.
Which serum laboratory analysis would be
most useful?

Prothrombin time

Alkaline phosphatase (Correct


Answer)

Acid phosphatase

Parathyroid hormone

file:///Users/mohammad/Desktop/PrometricMCQ.com.webarchive Page 80 of 377


PrometricMCQ.com 11/03/2015 10:45

Report an issue

Which type of intravenous fluid


administration is most appropriate?

Lactated Ringers solution (Correct


Answer)

5% dextrose in water solution

3.3% glucose in 0.3% sodium


chloride

5% dextrose in 0.45% normal saline

Report an issue

A 60 year-old man with a history of long-


term alcohol use presents to the clinic with
an enlarged abdominal girth, hemoptysis
and pruritus. His eyes and skin are visibly
yellow-coloured. Which additional finding
would indicate the need for immediate
hospital admission?

Azotemia (Correct Answer)

Asterixis

Tachycardia

Lower extremity edema

file:///Users/mohammad/Desktop/PrometricMCQ.com.webarchive Page 81 of 377


PrometricMCQ.com 11/03/2015 10:45

Report an issue

The district nurse attends a mother and a


girl who was born five days previously. At
birth, the baby had weighed 3.5 kilograms.
The mother becomes worried when she
hears that the baby weighs 3.3 kilograms
today. The infant is pink, alert and active
with 6-8 wet diapers and four stools per
day. Blanching of the skin shows a light
yellow color of the forehead but a pink
sternum.
What is the most appropriate nursing
action?

Notify the pediatrician (Correct


Answer)

Obtain blood for bilirubin analysis

Reassure the mother and continue


observing

Request the mother go to the clinic


to re-weigh

Report an issue

Based on the patient’s history which of the


following medication orders should the
nurse verify?

Glucophage

Zocor (Correct Answer)

file:///Users/mohammad/Desktop/PrometricMCQ.com.webarchive Page 82 of 377


PrometricMCQ.com 11/03/2015 10:45

Lasix

Imdur

Report an issue

Patient experienced (CVA) one week ago.


The doctor order the patient discharged
from the hospital to home health care. The
visiting nurse evaluates the patient and
notes the patient has difficulty initiating
speech and forming words.
Which of the following types of aphasia is
most likely in this patient?

Receptive

Expressive (Correct Answer)

Fluent

Conduction

Report an issue

A community health nurse assesses a68


year-old patient who lives in a group home.
The patient reports decreased appetite
after transferring to the group home
because the food tastes bland.
What type of data is the nurse collecting
from the above information?

Analytical

file:///Users/mohammad/Desktop/PrometricMCQ.com.webarchive Page 83 of 377


PrometricMCQ.com 11/03/2015 10:45

Derived

Objective

Subjective (Correct Answer)

Report an issue

A patient had a retinal detachment


surgically repaired the nurse identifies that
the detachment would most likely be
corrected and unlikely to reoccur if the
retina remains attached at least :

Three days

Two weeks (Correct Answer)

Two months

Three months

Report an issue

A patient complains of incoordination,


weakness and fatigue, double vision, and
slurred speech laboratory test results are
normal except for magnetic resonance
imaging (MRI) scan which shows small
plaques scattered throughout the white
matter. The nurse would anticipate the
diagnosis of :

Multiple sclerosis (Correct Answer)


file:///Users/mohammad/Desktop/PrometricMCQ.com.webarchive Page 84 of 377
PrometricMCQ.com 11/03/2015 10:45

Brain metastasis

Amyotrophic lateral sclerosis (Lou


Gehrig disease )

Polyradiculoneuritis ( Guillain-Barre
syndrome )

Report an issue

A nurse is caring for child who is post-


tonsillectomy and adenoidectomy. The
nurse should plane to assess which of the
following complications?

Pulmonary hypertension

Hemorrhage (Correct Answer)

Hearing loss

Cor pulmonale

Report an issue

A home care nurse visits a diabetic patient


who was started on insulin injection, upon
examination, the nurse observes small
lumps and dents on the right upper arm
where the patient has injected insulin.
What is the BEST nursing intervention?

file:///Users/mohammad/Desktop/PrometricMCQ.com.webarchive Page 85 of 377


PrometricMCQ.com 11/03/2015 10:45

Refer the patient to dermatologist


for diabetic cellulitis

Instruct the patient to rotate the


sites of injection (Correct Answer)

Report an issue

According to information provided in the


accompanying graphic. A descending
colon would be expected to produce

Fluid feces

Mushy faces

Semi-solid faces (Correct Answer)

Solid faces

Report an issue

file:///Users/mohammad/Desktop/PrometricMCQ.com.webarchive Page 86 of 377


PrometricMCQ.com 11/03/2015 10:45

A gravida 3 Para 2 presents to the


Maternity Triage Unit after the amniotic
membranes ruptured at home. The fluid is
noted to be clear. The neonate's head is
engaged into the pelvis and the patient is
having contractions every 5 to 7 minutes.
Each contraction lasts for 60-90 seconds.
An examination of the cervix finds 4
centimeters dilatation and 90%
effacement. She is uncomfortable during
contractions and rates the pain at a level 7,
on a pain scale of 1-10
Which finding is most indicative that she is
in true labor?

Level of pain

Cervical dilatation and effacement

Engagement of presenting part


fractions

Frequency and length of


contraction (Correct Answer)

Report an issue

A patient is planning to have an elective


surgical procedure to repair an umbilical
hernia. The patient is 68 years old. Weighs
136 kg (300 lbs.) Has diabetes mellitus.
Which of the following approaches would
be the MOST beneficial in order to reduce
the patients surgical risk?

Monitor blood glucose levels


monthly

file:///Users/mohammad/Desktop/PrometricMCQ.com.webarchive Page 87 of 377


PrometricMCQ.com 11/03/2015 10:45

Avoid fluid overload by restricting


fluids

Discourage any changes in routine


before surgery

Encourage weight reduction


(Correct Answer)

Report an issue

The nurse is discussing the human


immunodeficiency virus (HIV) with a group
of high - risk patients. The nurse should
state that this virus is found MOST
commonly in which of the following body
fluids?

Blood (Correct Answer)

Saliva

Breast milk

vaginal secretions

Report an issue

The nurse has provided dietary counseling


to a woman who is 12 weeks pregnant with
her first child. She is concerned about her
nutrition as she is vegetarian. She was
advised by a friend to eat a variety of foods
that contain proteins and asks the nurse
which food might be best included in a

file:///Users/mohammad/Desktop/PrometricMCQ.com.webarchive Page 88 of 377


PrometricMCQ.com 11/03/2015 10:45

meal plan.
Which combination would be most
appropriate?

Soya beans with carrots

chickpeas with whole grain bread

Nuts with a glass of cow's milk


(Correct Answer)

Avocado with a slice of cheese

Report an issue

After total knee replacement a patient is


being discharged to home after which he
will ambulate with a four-prong cane. When
providing patient teaching regarding going
up and down stairs with the cane, the first
step in gong up stairs is to

Place the unaffected extremity up on


the step (Correct Answer)

Place the cane up on the step

Step-up on the affected extremity

Step-up on the unaffected extremity

Report an issue

A nurse educates a patient about the use

file:///Users/mohammad/Desktop/PrometricMCQ.com.webarchive Page 89 of 377


PrometricMCQ.com 11/03/2015 10:45

of incentive spirometry to prevent


atelectasis after a surgery.
The nurse is performing what step of the
nursing process?

Diagnosis

Assessment

implementation (Correct Answer)

evaluation

Report an issue

A 65 year-old male presents with sever iron


deficiency anemia.
Which of the following would the nurse
expect to find in the initial physical
assessment?

Clubbing

Koilonychia (Correct Answer)

file:///Users/mohammad/Desktop/PrometricMCQ.com.webarchive Page 90 of 377


PrometricMCQ.com 11/03/2015 10:45

Beus line

Paronychia

Report an issue

Palpation, there is guarding and rebound


tenderness. Further investigation reveals a
small stone which is trapped in the
common bile duct On admission:
Blood pressure 148/80 mmHg
Heart rate 90min
Respiratory rate 21 shallow
Temperature 37.8
Which additional clinical finding is most
likely?

Yellowing of the sclera (Correct


Answer)

Dark and tarry stools

light-amber coloured urine

Clubbing of the fingers

Report an issue

A 10 year-old child with a seven year


history of diabetes mellitus presents to the
office with a parent for routine follow-up.
The patient wants to manage capillary
blood testing and insulin injections
independently. The parent believes the
child is not mature enough to manage the

file:///Users/mohammad/Desktop/PrometricMCQ.com.webarchive Page 91 of 377


PrometricMCQ.com 11/03/2015 10:45

disease, while the child is knowledgeable


about dietary needs and can recognize the
signs of hyperglycemia and hypoglycemia.
What are the two HIGHEST priority
interventions?

A,E

C,G

B,D

E,F (Correct Answer)

Report an issue

A home care nurse is instructing parents to


perform chest Physiotherapy (CPT) on their
child with cystic fibrosis the nurse has the
parents place their child in a prone position
with the body tilted downward before
performing CPT.
Based on this information, the nurse is
instructing the parents to remove drainage
from which of the following Jung lobe
segment?

file:///Users/mohammad/Desktop/PrometricMCQ.com.webarchive Page 92 of 377


PrometricMCQ.com 11/03/2015 10:45

Upper posterior

Lower anterior

Lower posterior (Correct Answer)

Upper anterior

Report an issue

Which of the following medications would


the patient be instructed to stop 2 weeks
prior the surgery?

Antiarrhythmic

Antifungal

Antidiabetics

antiplatelet (Correct Answer)

Report an issue

file:///Users/mohammad/Desktop/PrometricMCQ.com.webarchive Page 93 of 377


PrometricMCQ.com 11/03/2015 10:45

Which precautions would most effectively


prevent transmission?

Private room, gloves and gown for


care takers.

A mask over the patient’s mouth


and nose. (Correct Answer)

Negative pressure room and N95


mask for staff.

Hand washing and disposable


patient equipment.

Report an issue

A community health nurse screens a group


of high-risk adults for tuberculosis.
Which gauge needle should the nurse use
for an intradermal injection on the volar
surface of the forearm?

16 gauge needle

20 gauge needle

22 gauge needle

26 gauge needle (Correct Answer)

Report an issue

file:///Users/mohammad/Desktop/PrometricMCQ.com.webarchive Page 94 of 377


PrometricMCQ.com 11/03/2015 10:45

A 45 year-old women is admitted for a


surgical procedure. Local anesthetic
medication, bupivacaine, and fentanyl are
administered through a tiny needle placed
into the subarachnoid space. The
anesthetic is administered with the patient
in an upright and seated position. five
minutes later, the nurse records the
patients vital signs:
Blood pressure 88/58 mmHg
Heart rate 110/min
Respiratory rate 18/min
Temperature 38.0 oc tympanic

Blood pressure (Correct Answer)

Pulse rate

Respiratory rate

Body temperature

Report an issue

The nurse should understand that one


dose of which of the following vaccinations
is recommended for this patient before the
age of 65 years:

Varicella

Influenza

Hepatitis B

Pneumococcal (Correct Answer)

file:///Users/mohammad/Desktop/PrometricMCQ.com.webarchive Page 95 of 377


PrometricMCQ.com 11/03/2015 10:45

Report an issue

A home care nurse visits a patient with a


new below –the-knee amputation. The site
of the incision is red, warm and tender with
purulent yellow drainage. The patient has a
new prescription for cephalexin (Keflex)
and oxycodone (OxyContin)
Which would the nurse instruct the patient
to do FIRST?

Take oxycodone as soon as possible

Take cephalexin as soon as possible

Wash the incision site and apply


bacitracin cream (Correct Answer)

Wash the incision site and apply


hydrocortisone.

Report an issue

A 44 year-old woman presents with fatigue,


weakness and anorexia. She has recently
lost 10 kilograms in weight. She also
complains that she has difficulty
concentration and that her mood swings
easily between being happy and sad.
There is some hyperpigmentation of the
skin (see clinical signs). The nurse provides
nutritional counseling for the patient (see
lab result).
Test result normal value
Cortisol 3.0 5 -25 mcg/dl
Which dietary change is most appropriate?

file:///Users/mohammad/Desktop/PrometricMCQ.com.webarchive Page 96 of 377


PrometricMCQ.com 11/03/2015 10:45

Increased sodium (Correct


Answer)

Decrease protein

Low carbohydrates

decreased potassium

Report an issue

39 year-old male has the following vital


signs the patient has received 2 unites of
packed red blood cells (PRBCs) and is
intubated.
Which of the vital signs is considered out
of normal range?
Blood pressure 126/78 mmHg
Heart rate 62/min
O2 saturation 89% on room air
Temperature 37 oC

Heart rate

O2 saturation rate (Correct


Answer)

Core temperature

Blood pressure

Report an issue

According to the hypothetical graph of

file:///Users/mohammad/Desktop/PrometricMCQ.com.webarchive Page 97 of 377


PrometricMCQ.com 11/03/2015 10:45

urinary output versus time of intravenous


(IV) administration, at what hour of IV
administration dose the urinary output
attain the desired output of 30 ml/hr.:

2 hours (Correct Answer)

4 hours

6 hours

8 hours

Report an issue

A 22 year-old man presented to the Mental


Health Clinic with a low-mood, a general
loss of interest in activities and inability to
experience pleasure. He admitted to
suicidal thoughts and extreme lack of
energy. He was prescribed a selective
serotonin reuptake inhibitor to be taken
daily. One month later, he presented to the
clinic and reports feeling more energetic,
but still has a low-mood.
What is the patient’s level of risk
committing suicide at this time?

None

Low

Medium

file:///Users/mohammad/Desktop/PrometricMCQ.com.webarchive Page 98 of 377


PrometricMCQ.com 11/03/2015 10:45

High (Correct Answer)

Report an issue

An alert oriented 77 year-old woman lives


in a long-term care nursing home although
she is usually busy participating in the
coordinated activities of the nursing home,
the nurse finds her sitting alone in the
corridor. Various tests are ordered
including a urine analysis (see X-ray).
Blood pressure 128/74 mmHg
Heart rate 98/min
Respiratory rate 26/min
Body temperature 38.3 °C oral
Which additional clinical finding would
most likely be an indicator of the early
onset of her problem?

Low urea concentration

Acute confusion

Increased blood pressure

Respiratory rate > 22/minute


(Correct Answer)

Report an issue

A patient requiring long-term antibiotics


has a central line catheter inserted into the
right subclavian vein by the physician.
Which of the following must be verified
prior to the first use of the catheter?

file:///Users/mohammad/Desktop/PrometricMCQ.com.webarchive Page 99 of 377


PrometricMCQ.com 11/03/2015 10:45

Blood return (Correct Answer)

X-ray

Catheter patency

Length of catheter

Report an issue

A 35 year-old woman with excessive facial


and body hair and acne has had weight
gain in the shoulder, upper back and face.
She does not drink alcohol and does not
have diabetes. She takes glucocorticoids
regularly. Which dietary plan would be
most beneficial for this patient?

Increased calcium (Correct


Answer)

Decreased sodium

Increased potassium

Decreased protein

Report an issue

A 45 year-old woman presents to the clinic


with urinary incontinence. Point-to-point
testing, in which she must touch the tip of
her nose with the doctor’s index finger on
command, reveals slowness and

file:///Users/mohammad/Desktop/PrometricMCQ.com.webarchive Page 100 of 377


PrometricMCQ.com 11/03/2015 10:45

uncertainty. Alternate hand movement


testing shows asymmetry, slowness and
increased concentration. Rhythmic,
involuntary movements of the hands are
noted at rest. During Romberg’s test, the
patient easily loses balance.
Which type of urinary incontinence
treatment is most appropriate?

Bladder retraining and anti-


cholinergic agents

Timed voiding and biofeedback


training

Intermittent straight catheter and


biofeedback

Timed voiding and anti-cholinergic


agents (Correct Answer)

Report an issue

A nurse is caring for an eight year-old male


with cystic fibrosis.
Based on the nurse’s understanding of the
disease what nursing intervention should
the nurse expect to perform?

Restrict sodium and fluid intake

Give antidiarrheal medications

Discourage coughing after postural


drainage

Administer pancreatic enzymes with


each meal (Correct Answer)

file:///Users/mohammad/Desktop/PrometricMCQ.com.webarchive Page 101 of 377


PrometricMCQ.com 11/03/2015 10:45

Report an issue

A patient hospitalized with tuberculosis


(TB) has a productive cough and
hemoptysis.
Which of the following types of isolation
room would be the BEST choice for the
patient?

Reverse isolation

Standard isolation

Positive-pressure

Negative-pressure (Correct
Answer)

Report an issue

A 78 year-old man presents to his care


provider with persistent abdominal and
back pain. He also occasionally has
numbness and pain in the legs. On
examination a palpable mass in the lower
abdominal midline is palpated. On
auscultation a blowing sound is heard over
his mass. The nurse is concerned about
possible rupture.
Which additional finding most significantly
indicates the patient is at an increased
risk?

Hypertension (Correct Answer)

file:///Users/mohammad/Desktop/PrometricMCQ.com.webarchive Page 102 of 377


PrometricMCQ.com 11/03/2015 10:45

High blood sugar

Constipation

Decreased oxygen saturation

Report an issue

When administrating an enema to an adult


patient, how far should the nurse insert the
tubing into the rectum?

2.2 to 4.4 cm (1 to 2 inches)

4.4 to 6.6 cm (2 to 3 inches)

6.6 to 8.8 cm (3 to 4 inches)


(Correct Answer)

8.8 to 11 cm (4 to 5 inches)

Report an issue

A nurse plans to teach a group of 20 to 25


year-old women about oral contraceptives.
The nurse should instruct that oral
contraceptives may:

Increase risk of pelvic inflammatory


disease

Cause acne to worsen

Decrease the risk of breast and

file:///Users/mohammad/Desktop/PrometricMCQ.com.webarchive Page 103 of 377


PrometricMCQ.com 11/03/2015 10:45

cervical cancer

Decrease the risk of endometriosis


(Correct Answer)

Report an issue

A 35 year-old man twisted his right ankle.


Since then, he has had swelling and
stiffness of the ankle in the morning hours.
He presented to the clinic six months after
the injury and complains that he is in too
much pain to participate in sports. An
arthroscopy reveals thickening and
inflammation of the lining of the ankle joint.
Which treatment is most likely to be
ordered for this client?

Aspiration of fluids

Surgical intervention

Physical therapy

NSAID administration (Correct


Answer)

Report an issue

An eight year-old boy presents with


abdominal cramping and a feeling of
fullness. He frequently passes semi-formed
stools that are light yellow colored and oily,
with an abnormally foul odor. His mother
states that the child has lost his appetite
lately and is concerned about weight loss.
file:///Users/mohammad/Desktop/PrometricMCQ.com.webarchive Page 104 of 377
PrometricMCQ.com 11/03/2015 10:45

Endoscopic examination reveals villous


atrophy.
Which food would be most suitable for this
child?

Rye

Oats (Correct Answer)

Barley

Wheat

Report an issue

A three year-old child is day three on the


postoperative care unit following a heart
transplant. He is prescribed cyclosporine
by intravenous infusion.
Blood pressure 140/80 mmHg
Heart rate 100/min
Respiratory rate 22/m
Temperature 37.0 °C
Oxygen saturation 92% on room air
Which vital sign most likely indicates an
adverse medication effect?

Hypertension (Correct Answer)

Decreased O2 saturation

Tachycardia

Tachypnea

Report an issue

file:///Users/mohammad/Desktop/PrometricMCQ.com.webarchive Page 105 of 377


PrometricMCQ.com 11/03/2015 10:45

A 6-month-old premature infant is being


discharged from the neonatal intensive
care unit after having no apneic periods for
the past week. Although the child has had
no apneic periods for the past week, an
apnea monitor will be sent home with the
parents. The BEST short term goal relating
to the nursing diagnosis of ineffective
breathing pattern related to apnea is that
the:

Apnea monitor will sound loudly and


stimulate the child to breath with
episodes of apnea

Apnea monitor will sound loudly and


alert the parents to stimulate the
child to breath with episodes of
apnea (Correct Answer)

Child will have no episodes of apnea


or cyanosis while on apnea monitor

Child will sleep in their parent’s


bed each night until there is no
apnea two weeks

Report an issue

A 25 year-old female presented to the


emergency room with lethargy, decreased
reflexes, hypoventilation, hypotension, and
fixed and dilated pupils. A family member
who is accompanying the patient has an
empty bottle of diazepam (Valium) which
the label states was recently refilled. The
family member also indicates that the
patient has a history of depression.

file:///Users/mohammad/Desktop/PrometricMCQ.com.webarchive Page 106 of 377


PrometricMCQ.com 11/03/2015 10:45

What intervention should the nurse expect


to administer?

Naloxone (Narcan)

Activated charcoal (Correct


Answer)

Magnesium sulfate to reduce the


risk of seizures

A tap water enema

Report an issue

A nurse is documenting in patient records


several events that occurred during home
visits.
Which of the following is an example of the
correct way to document patient
information?

Patient fell walking to bathroom.


Busy preparing for sterile dressing
change when patient left the room

Patient got out of bed without


assistance. Denies any symptoms
when ambulating alone (Correct
Answer)

Patient sitting in chair. Strict bedrest


orders ignored

Patient showering. Electronic


epidural infusion pump on floor by
shower stall

file:///Users/mohammad/Desktop/PrometricMCQ.com.webarchive Page 107 of 377


PrometricMCQ.com 11/03/2015 10:45

Report an issue

A 50 year-old man is hospitalized after


presenting with long-term fatigue, weight
gain and hematemesis. On examination he
is jaundiced and the abdomen distended
and tense, and there is shifting dullness
present. Following treatment, the nurse
provides discharge instructions.
Which instructions would be most
appropriate?

Decrease daily iron intake

Maintain constant bed rest


(Correct Answer)

Consume a low protein diet

Restrict fluid intake

Report an issue

The nurse is assessing an infant with


Gastroeosophageal reflux disease (GERD).
To help identify any complications of
GERD, what is the most important
question the nurse should ask the infant’s
parents?

Frequency of respiratory tract


infection (Correct Answer)

Frequency of vomiting

file:///Users/mohammad/Desktop/PrometricMCQ.com.webarchive Page 108 of 377


PrometricMCQ.com 11/03/2015 10:45

Time between bowl movements

Time between feeds

Report an issue

A patient with an unsteady gait and a


history of falls has care plan interventions
that include keeping the walker in reach
and pathways free of obstacles. On
evaluation after 1 week, the patient has
had no falls, but the gait remains unsteady.
The nurse should:

Continue the plan of care as written

Allow the patient to replace the


walker with a cane

Allow the patient to ambulate short


distances without the walker
(Correct Answer)

Have the patient practice stepping


over small objects

Report an issue

A patient complains of severe menstrual


cramping. Bleeding is not usually heavy,
and the patient has no uterine disorders.
Which of the following interventions should
the nurse anticipate the doctor will order to
promote comfort?

file:///Users/mohammad/Desktop/PrometricMCQ.com.webarchive Page 109 of 377


PrometricMCQ.com 11/03/2015 10:45

Acetaminophen (Tylenol)

Strict bed rest

Heating pad to the back of neck

Ibuprofen (Motrin) (Correct


Answer)

Report an issue

A patient is admitted to the emergency


department with stable angina. The
physician writes an order for nitroglycerin
1/150 grains sublingually (SL).
Which is the correct dose in milligrams
(mg)?

0.04 mg

0.4 mg (Correct Answer)

4 mg

40 mg

Report an issue

A perimenopausal woman with a small


body frame is at high risk for a condition in
which bone resorption exceeds the rate of
bone formation. She drinks one-three cups
of coffee per day since she was 25 years
old and smokes five-ten cigarettes per day
for the past 20 years. The nurse advises

file:///Users/mohammad/Desktop/PrometricMCQ.com.webarchive Page 110 of 377


PrometricMCQ.com 11/03/2015 10:45

the patient to increase her dietary intake of


milk and green leafy vegetables.
Which additional foods are most
appropriate to recommend?

Egg yolk, cheese (Correct Answer)

Peanuts, poultry

Whole grains, cereals

Lentils and black beans

Report an issue

A 30 year-old woman with type 1 diabetes


mellitus receives mixed type of insulin in
the morning and before bed time. She
reports that the level of her fasting blood
sugar is constantly high when she checks
it every morning at home.
Which dose of insulin is most likely causing
this problem?

Low morning, regular insulin

High morning NPH1

High evening regular insulin

Low evening NPH insulin (Correct


Answer)

Report an issue

file:///Users/mohammad/Desktop/PrometricMCQ.com.webarchive Page 111 of 377


PrometricMCQ.com 11/03/2015 10:45

A patient who is preparing for hip surgery


has an order for external pneumatic
compression devices. The nurse teaches
the patient that pneumatic compression
can help prevent:

Upper respiratory infection

Decreased breath sounds

Deep vein thrombosis (Correct


Answer)

Bleeding at the surgical site

Report an issue

Following an open-cholecystectomy, the


nurse would instruct the patient to expect
to resume normal activities in:

1 to 2 weeks

2 to 3 weeks

4 to 6 weeks (Correct Answer)

6 to 8 weeks

Report an issue

A five month-old boy has been vomiting


green colored vomit for ten hours. He has
intermittent abdominal pain during which

file:///Users/mohammad/Desktop/PrometricMCQ.com.webarchive Page 112 of 377


PrometricMCQ.com 11/03/2015 10:45

he draws his legs up to his chest, turns


pale and cries forcefully. On observation,
there is bleeding in the stool which has a
jelly-like consistency. Abdominal palpation
reveals a long tube-like mass. There is no
fever, rash nor diarrhea. Bowel sounds are
hyperactive in all quadrants.
Which is the most likely form of initial
treatment?

Manual manipulation

Surgical resection

Barium enema (Correct Answer)

Endoscopy

Report an issue

The nurse is assisting a patient to


ambulate in hall. The patient has a history
of coronary artery disease (CAD) and had
coronary artery bypass graft surgery
(CABG) 3 days ago. The patient reports
chest pain rated 3 on a scale of 0 (no pain)
to 10 (severe pain). The nurse should first:

Determine how long it has been


since the patient’s last dose of
aspirin

Obtain a chair for the patient to sit


down (Correct Answer)

Assess the patient’s radial pulse

Ask the patient to take several slow,


deep breaths.

file:///Users/mohammad/Desktop/PrometricMCQ.com.webarchive Page 113 of 377


PrometricMCQ.com 11/03/2015 10:45

Report an issue

A 52 year-old man with congestive heart


failure presents to the Emergency
Department with rapid and irregular
heartbeats, and feeling dizzy and light-
headed. The attending physician verbally
calls out an order to the nurse to
administer digoxin 0.25 milligrams by
intravenous injection. How will the nurse
complete the paperwork for this order?

Write, sign and repeat order back to


the physician

Ask the physician to write and sign


the order

Write the order and ask the


physician to sign (Correct Answer)

Verbally repeat the order and


administer drug

Report an issue

An 82 year-old patient has Parkinson’s


disease. During the assessment, the nurse
would expect which of the following
actions to produce the MOST tremor
activity of the hands?

Eating with a fork

Resting hands in lap (Correct


file:///Users/mohammad/Desktop/PrometricMCQ.com.webarchive Page 114 of 377
PrometricMCQ.com 11/03/2015 10:45

Answer)

Standing with hands loose at sides

Rolling a small pill between the


fingers

Report an issue

A 67 year-old man was admitted to the


hospital following a closed bone fracture.
An intramedullary nail is inserted and the
patient is placed in balanced skeletal
traction. The following day, the patient
becomes restless, drowsy and confused,
he has difficulty breathing and appears
very tired.
Which additional sign or symptom would
require immediate intervention?

Anxiety

Cold skin (Correct Answer)

Constipation

Petechiae on chest

Report an issue

A patient is preparing for a total knee


replacement. During the preoperative
interview process the patient reports an
allergic reaction to penicillin.
Which of the following is considered a side
effect and not a true allergy to medication?

file:///Users/mohammad/Desktop/PrometricMCQ.com.webarchive Page 115 of 377


PrometricMCQ.com 11/03/2015 10:45

Shortness of breath

Tingling lips and tongue

Rash

Upset stomach (Correct Answer)

Report an issue

A 54 year-old woman presented to the


Emergency Department with sharp upper-
right abdominal pain that radiates to the
right scapula. While performing the
admission assessment, the patient
becomes nauseous and begins vomiting.
She states that she has had pain in the
upper right quadrant previously but that
this time it was far worse. There’s a
positive Murphy’s sign and an ultrasound
confirms gallbladder wall thickening and
pericholecystic fluid collections. Which of
the following would most likely be
associated with her clinical findings?

Relief by drinking milk

Alleviation with exercise

Triggered by fatty meal (Correct


Answer)

Worsening on empty stomach

Report an issue

file:///Users/mohammad/Desktop/PrometricMCQ.com.webarchive Page 116 of 377


PrometricMCQ.com 11/03/2015 10:45

A 68 year-old woman is receiving


parenteral nutrition at home. The district
nurse visits the woman and notes that she
has gained one kilogram of weight since a
health provider had visited one week ago.
There is pitting edema of 2+ of the lower
extremities. The patient is alert, active and
oriented.
Which nursing diagnosis is most
appropriate?

Non-compliance

Impaired gas exchange

Imbalanced nutrition

Fluid volume overload (Correct


Answer)

Report an issue

A patient has a central line catheter and is


receiving a three-in-one total parenteral
nutrition that contains glucose, proteins
and lipids. The pump is set to deliver the
infusion over a 12-hour period. After how
many hours should the intravenous
administration set be changed?

12

24

48

72 (Correct Answer)

file:///Users/mohammad/Desktop/PrometricMCQ.com.webarchive Page 117 of 377


PrometricMCQ.com 11/03/2015 10:45

Report an issue

A 45 year-old man who is hospitalized feels


the constant need to keep things in order,
particularly whilst eating. The nurse
observes him arranging the food on his
plate into symmetrical and equal bite-sized
pieces. He constantly worries that food
served could be outdated and potentially
cause illness.
Which nursing diagnosis is most
important?

Ineffective verbal communication

Self-esteem disturbance

Impaired social interaction

Anxiety (Correct Answer)

Report an issue

A 12 year- old patient had a cast removed


from the left leg after wearing if for eight
weeks. The patient wants to resume sports
as soon as possible. In order to regain
muscle strength lost while wearing cast,
the nurse will instruct the patient in
performance of:

Resistive range of motion exercises


to left leg

file:///Users/mohammad/Desktop/PrometricMCQ.com.webarchive Page 118 of 377


PrometricMCQ.com 11/03/2015 10:45

Passive range of motion exercises to


right leg

Active- assistive range of motion


exercises to the right leg

Active range of motion exercises to


both legs (Correct Answer)

Report an issue

A three year-old has returned to the clinic 4


days after being diagnosed with
gastroenteritis and dehydration. A parent
reports that the vomiting has stopped, and
the child is tolerating liquids, rice,
applesauce, and bananas. The diarrhea
persists, but seems to be decreasing in
volume. When evaluating for signs of
dehydration, the nurse will assess the
patient’s skin turgor by:

Grasping the skin over the abdomen


with two fingers and raising the skin
with two fingers (Correct Answer)

Grasping the skin over the forehead


with two fingers and raising the skin
with two fingers

Holding the patient’s mouth


open and assessing the tongue for
deep creases or furrows

Drawing two tubes of blood and


running a blood urea nitrogen (BUN)
and creatinine (Cr)

Report an issue

file:///Users/mohammad/Desktop/PrometricMCQ.com.webarchive Page 119 of 377


PrometricMCQ.com 11/03/2015 10:45

A patient has had hepatitis B (HBV) and is


now a chronic carrier. In planning care, the
nurse would explain an HBV carrier would
most likely be at risk for developing a
super infection with which other type of
hepatitis?

D (Correct Answer)

Report an issue

A female patient has been advised that


laboratory tests confirm herpes simplex
virus (HSV), type 2. The nurse should teach
the patient that a Papanicolaou test (Pap
smear) is recommended:

Every 6 months if symptoms persist


despite treatment (Correct Answer)

Every year even if asymptomatic

Whenever symptoms recur

Every 3 years if other Pap smears


have been negative

Report an issue

file:///Users/mohammad/Desktop/PrometricMCQ.com.webarchive Page 120 of 377


PrometricMCQ.com 11/03/2015 10:45

A 30 year-old married man presents to the


clinic with complaints of feeling sad for the
past three months. He is unable to
maintain a regular sleep routine, has lost
his appetite and has difficulty
concentrating. He is prescribed a
medication which prevents the reuptake of
specific neurotransmitters that could
contribute to his mental health problem.
Which side effects would be most
important for the nurse to advise the
patient of?

Polyuria

Photophobia

Fluid retention

Sexual dysfunction (Correct


Answer)

Report an issue

According to information provided in the


accompanying graphic, a transverse

file:///Users/mohammad/Desktop/PrometricMCQ.com.webarchive Page 121 of 377


PrometricMCQ.com 11/03/2015 10:45

colostomy would be expected to produce:

Fluid feces

Mushy feces (Correct Answer)

Semi-fluid feces

Solid feces

Report an issue

A patient presents with pneumonia and


dyspnea. The patient has an ineffective
airway clearance related to excessive
tracheobronchial secretions.
Which of the following interventions would
the nurse implement to enhance the
patient’s airway clearance?
Heart rate 80/min
Respiratory rate 32/min
Temperature 40 °C

Administer oxygen as ordered

Maintain a comfortable position

Increase fluid intake (Correct


Answer)

file:///Users/mohammad/Desktop/PrometricMCQ.com.webarchive Page 122 of 377


PrometricMCQ.com 11/03/2015 10:45

Administer prescribed analgesics

Report an issue

The nurse assesses a patient who is 16-


weeks pregnant. The patient states that
she had taken isotretinoin (Accutane) , a
known teratogen for acne during her third ,
fourth , and fifth week of pregnancy
According to the chart , the nurse CAN
expect fetal damage to the central nervous
system as well as the :

Palate and ears.

Heart, lower limbs, and palate.

Limbs, eyes, and teeth.

Heart, eyes, and limbs. (Correct


Answer)

Report an issue

RTA. The patient appears restless


confused and disoriented. He reports that
he had hit his head against the steering
wheel of the car when it had collided with
the car directly in front of him.
Blood pressure 110/68 mmHg
Heart rate 100/min
Respiratory rate 22/min
Body Temperature 37.0 c oral
Oxygen saturation 98 % on room air

file:///Users/mohammad/Desktop/PrometricMCQ.com.webarchive Page 123 of 377


PrometricMCQ.com 11/03/2015 10:45

What is the most important next step in


management?

Immobilize head and neck.


(Correct Answer)

Administer oxygen.

Establish an intravenous line.

Arrange for an MRI scan.

Report an issue

Which of the following is a desired


expected outcome 24 hours
postoperatively?

Gag reflex present. (Correct


Answer)

Cerebral perfusion pressure,


68mmHg

Intracranial pressure, 21mmHg.

Decreased lacrimation.

Report an issue

What is the Proper procedure for doing a


breast self-exam?

file:///Users/mohammad/Desktop/PrometricMCQ.com.webarchive Page 124 of 377


PrometricMCQ.com 11/03/2015 10:45

Use the palm of the hand to feel for


lumps.

Apply three different levels of


pressure to feel breast tissue.

Stand when performing a breast


self-exam. (Correct Answer)

Perform self-exam annually

Report an issue

A 45 year-old patient has had difficulty


sleeping and has lost ten kilograms despite
having a large appetite on examination
there is a palpable thyroid gland.
Blood pressure 108/58 mmHg
Heart rate 116/min
Respiratory rate 22/min
Body temperature 38.0 c oral
Height 164
Weight 50 kilograms
Which additional symptom is most likely?

Heart palpitations. (Correct


Answer)

Depression.

Anorexia.

Paresthesia

Report an issue

file:///Users/mohammad/Desktop/PrometricMCQ.com.webarchive Page 125 of 377


PrometricMCQ.com 11/03/2015 10:45

Nurse prepares to delegate tasks to the


nursing assistant Among her patients is a
50 year-old woman who is day two of
recovery following a laparoscopic resection
of the colon post-operative orders are
follow :
Ambulate every six hours.
Evaluate vital signs every two hours.
Lactated Ringer's IV at 50 ml/hour.
Wound assessment every eight hours.
Nasogastric tube until bowel sounds
present.
Which is most appropriate to delegate?

Ambulate the patient. (Correct


Answer)

Evaluation of vital signs.

Change intravenous fluid bags.

Assess nasogastric tube placement.

Report an issue

The nurse is teaching a patient with


diabetes mellitus about self-administration
of insulin. Which syringe angle is correct

file:///Users/mohammad/Desktop/PrometricMCQ.com.webarchive Page 126 of 377


PrometricMCQ.com 11/03/2015 10:45

when administering insulin?

B (Correct Answer)

Report an issue

Which of the following anatomical terms


best describes the motion in the picture
above:

Adduction

Abduction (Correct Answer)

Supination

Pronation

file:///Users/mohammad/Desktop/PrometricMCQ.com.webarchive Page 127 of 377


PrometricMCQ.com 11/03/2015 10:45

Report an issue

Which of the above locations represents


the best sites to inspect chest retractions
in a
child with lower respiratory tract disorder:

A&B

B&C

C&D

D&E (Correct Answer)

Report an issue

Your patient's finger stick blood sugar at


07:30 is 300 mg/dl, according to the
following sliding scale; how many unites
you would give your patient:

file:///Users/mohammad/Desktop/PrometricMCQ.com.webarchive Page 128 of 377


PrometricMCQ.com 11/03/2015 10:45

6 i.u

8 i.u

14 i.u (Correct Answer)

18 i.u

Report an issue

The nurse is caring for a diabetic child


whose 11 a.m. blood glucose monitoring
check is 269 mg/dL. The physician orders
the following coverage schedule:
According to the coverage schedule; how
many unites you would give your patient:

4 units

6 units (Correct Answer)

8 units

10 units

Report an issue

file:///Users/mohammad/Desktop/PrometricMCQ.com.webarchive Page 129 of 377


PrometricMCQ.com 11/03/2015 10:45

The following syringe contains morphine


sulphate in a concentration of 2 mg/cc,
how
much total morphine this syringe contains:

0.5 mg

1 mg

2 mg

4 mg (Correct Answer)

Report an issue

The following syringe contains 2 gm of


morphine sulphate, how many grams are
there in 1 ml:

file:///Users/mohammad/Desktop/PrometricMCQ.com.webarchive Page 130 of 377


PrometricMCQ.com 11/03/2015 10:45

0.5 gm

1 gm (Correct Answer)

2 gm

4 gm

Report an issue

The following condition may develop in


females due to an increase in the
production
of which of the following hormones:

Androgens (Correct Answer)

Progesterone

Estrogen

Growth

Report an issue

file:///Users/mohammad/Desktop/PrometricMCQ.com.webarchive Page 131 of 377


PrometricMCQ.com 11/03/2015 10:45

What is the name of The process of


movement of a dissolved substance
(solutes)
from an area of high concentration to an
area of low concentration:

Effusion

Diffusion (Correct Answer)

Emulsion

Excretion

Report an issue

Which of the following is the incision site


for an open cholecystectomy operation:

file:///Users/mohammad/Desktop/PrometricMCQ.com.webarchive Page 132 of 377


PrometricMCQ.com 11/03/2015 10:45

2 (Correct Answer)

Report an issue

Which of the following is the incision site


for an open gastrectomy operation:

file:///Users/mohammad/Desktop/PrometricMCQ.com.webarchive Page 133 of 377


PrometricMCQ.com 11/03/2015 10:45

4 (Correct Answer)

Report an issue

Which of the following is the incision site


for surgical appendectomy operation:

file:///Users/mohammad/Desktop/PrometricMCQ.com.webarchive Page 134 of 377


PrometricMCQ.com 11/03/2015 10:45

C (Correct Answer)

Report an issue

What is the most probable type of


arrhythmia in the following ECG (lead ll):

file:///Users/mohammad/Desktop/PrometricMCQ.com.webarchive Page 135 of 377


PrometricMCQ.com 11/03/2015 10:45

Atrial fibrillation (Correct Answer)

Atrial flutter

Ventricular fibrillation

Supra-ventricular fibrillation

Report an issue

What is the most probable type of


arrhythmia in the following ECG (lead V1):

Atrial fibrillation

Atrial flutter

Ventricular tachycardia (Correct


Answer)

Supra-ventricular fibrillation

Report an issue

The following picture suggests that the


patient has:

file:///Users/mohammad/Desktop/PrometricMCQ.com.webarchive Page 136 of 377


PrometricMCQ.com 11/03/2015 10:45

Renal disease

Hepatic disease

Pulmonary disease (Correct


Answer)

Cerebral disease

Report an issue

The following picture represents the pain


distribution of which nerve affection:

file:///Users/mohammad/Desktop/PrometricMCQ.com.webarchive Page 137 of 377


PrometricMCQ.com 11/03/2015 10:45

C2

C6 (Correct Answer)

T3

T6

Report an issue

On assessing the client’s pressure ulcer


(see photo), the nurse would document
this as which stage:

Stage I

Stage II

Stage III (Correct Answer)

Stage IV

Report an issue

file:///Users/mohammad/Desktop/PrometricMCQ.com.webarchive Page 138 of 377


PrometricMCQ.com 11/03/2015 10:45

The following picture represents an auto


immune disease known as:

Psoriasis

Systemic Lupus Erythematosus


(Correct Answer)

Molluscum Contagiosum

Eczema

Report an issue

What is the name of the equipment this


picture shows?

file:///Users/mohammad/Desktop/PrometricMCQ.com.webarchive Page 139 of 377


PrometricMCQ.com 11/03/2015 10:45

Oropharyngeal airway (OPA)

Nasopharyngeal airway (NPA)


(Correct Answer)

Laryngeal mask airway (LMA)

Urinary catheter

Report an issue

A client is being prepared for a


thoracentesis. A nurse assists the client to
which
position for the procedure:

Lying in bed on the affected side

Lying in bed on the unaffected


side (Correct Answer)

Sims’ position with the head of


the bed flat

Prone with the head turned to the


side and supported by a pillow

Report an issue

A nurse is preparing to insert a


nasogastric tube into a client. The nurse
places the
client in which position for insertion:

file:///Users/mohammad/Desktop/PrometricMCQ.com.webarchive Page 140 of 377


PrometricMCQ.com 11/03/2015 10:45

Right side

Low Fowler’s

High Fowler’s (Correct


Answer)

Supine with the head flat

Report an issue

A nurse develops a plan of care for a client


with deep vein thrombosis. Which client
position or activity in the plan will be
included:

Out-of-bed activities as desired

Bedrest with the affected extremity


kept flat

Bedrest with elevation of the


affected extremity (Correct
Answer)

Bedrest with the affected extremity


in a dependent position

Report an issue

A client in labor is transported to the


delivery room and is prepared for a
cesarean
delivery. The client is transferred to the
delivery room table, and the nurse places
the client in the:
file:///Users/mohammad/Desktop/PrometricMCQ.com.webarchive Page 141 of 377
PrometricMCQ.com 11/03/2015 10:45

Trendelenburg's position with the


legs in stirrups

Semi-Fowler position with a pillow


under the knees

Prone position with the legs


separated and elevated

Supine position with a wedge under


the right hip (Correct Answer)

Report an issue

The nurse is caring for a client who is 1 day


postoperative for a total hip
replacement. Which is the best position in
which the nurse should place the client:

Side-lying on the operative side

On the nonoperative side with the


legs abducted (Correct Answer)

Side-lying with the affected leg


internally rotated

Side-lying with the affected leg


externally rotated

Report an issue

file:///Users/mohammad/Desktop/PrometricMCQ.com.webarchive Page 142 of 377


PrometricMCQ.com 11/03/2015 10:45

Which of the following is the best position


to increase the brain perfusion( cerebral
perfusion):

Prone

Supine

Semi- Fowler’s

Trendelenburg’s (Correct
Answer)

Report an issue

A nurse is administering a cleansing enema


to a client with a fecal impaction.
Before administering the enema, the nurse
places the client in which position:

Left Sims’ position (Correct


Answer)

Right Sims’ position

On the left side of the body, with the


head of the bed elevated 45 degrees

On the right side of the body, with


the head of the bed elevated 45
degree

Report an issue

file:///Users/mohammad/Desktop/PrometricMCQ.com.webarchive Page 143 of 377


PrometricMCQ.com 11/03/2015 10:45

A client has just returned to a nursing unit


after an above-knee amputation of the
right leg. A nurse places the client in which
position:

Prone

Reverse Trendelenburg’s

Supine, with the amputated limb flat


on the bed

Supine, with the amputated limb


supported with pillows (Correct
Answer)

Report an issue

A nurse is caring for a client with a severe


burn who is scheduled for an autograft
to be placed on the lower extremity. The
nurse develops a postoperative plan of
care
for the client and includes which of the
following in the plan:

Maintain the client in a prone


position

Elevate and immobilize the grafted


extremity (Correct Answer)

Maintain the surgical extremity in a


flat position

Keep the surgical extremity covered


with a blanket

file:///Users/mohammad/Desktop/PrometricMCQ.com.webarchive Page 144 of 377


PrometricMCQ.com 11/03/2015 10:45

Report an issue

A nurse is preparing to care for a client


who has returned to the nursing unit
following cardiac catheterization performed
through the femoral artery. The nurse
checks the physician’s prescription and
plans to allow which client position or
activity following the procedure:

Bedrest in high Fowler’s position

Bedrest with bathroom privileges


only

Bedrest with head elevation at 60


degrees

Bedrest with head elevation no


greater than 30 degrees (Correct
Answer)

Report an issue

After the client undergoes a total hip


replacement, how should the nurse
position
the affected hip?

Adduct the hip

Abduct the hip (Correct Answer)

Flex the hip

file:///Users/mohammad/Desktop/PrometricMCQ.com.webarchive Page 145 of 377


PrometricMCQ.com 11/03/2015 10:45

Extend the hip

Report an issue

A client has an open reduction and internal


fixation for a fractured hip.
Postoperatively the nurse should position
the client's affected extremity in:

External rotation

Slight hip flexion

Moderate abduction (Correct


Answer)

Anatomical body alignment

Report an issue

Which of the following positions should the


nurse place a client for rectal tube
insertion:

left lateral position with


hyperextension of right knee

left lateral position with hyperflextion


of left knee

left lateral position with hyperflextion


of right knee (Correct Answer)

left lateral position with

file:///Users/mohammad/Desktop/PrometricMCQ.com.webarchive Page 146 of 377


PrometricMCQ.com 11/03/2015 10:45

hyperextension of left knee

Report an issue

Which of the following position is


appropriate for the patient to use for
selfadministration
of fleet enema:

Left lateral position right leg flex

Face down and leg flex

Face upward and leg free (on back


and flex hip and knee) (Correct
Answer)

Chest- knee position

Report an issue

The best position for a child with

file:///Users/mohammad/Desktop/PrometricMCQ.com.webarchive Page 147 of 377


PrometricMCQ.com 11/03/2015 10:45

myelomeningocele is?

Prone (Correct Answer)

supine

semi-fowler’s

modified trendelenburg

Report an issue

Unless the physician orders otherwise, in


which position should the nurse place the
infant during the postoperative period of
surgical repair of the myelomeningocele:

Supine

Prone (Correct Answer)

Right or left side-lying

Whichever position is most


comfortable for the infant

Report an issue

The nurse is caring for a client with an


above-the-knee amputation (AKA). To
prevent contractures, the nurse should:

file:///Users/mohammad/Desktop/PrometricMCQ.com.webarchive Page 148 of 377


PrometricMCQ.com 11/03/2015 10:45

Place the client in a prone position


15–30 minutes twice a day
(Correct Answer)

Keep the foot of the bed elevated on


shock blocks

Place trochanter rolls on either side


of the affected leg

Keep the client’s leg elevated on


two pillows

Report an issue

Postoperative nursing care of the infant


following surgical repair of a cleft lip
would include:

Feeding the infant with a spoon to


avoid sucking

Positioning the infant on the


abdomen to facilitate drainage

Applying elbow restraints to protect


the surgical area (Correct Answer)

Providing minimal stimulation to


prevent injury to the incision

Report an issue

The client has returned to the nursing unit


following a right below-the-knee

file:///Users/mohammad/Desktop/PrometricMCQ.com.webarchive Page 149 of 377


PrometricMCQ.com 11/03/2015 10:45

amputation. How should the nurse position


the client?

Supine with head turned to the side

With shock blocks placed under the


foot of the bed (Correct Answer)

Semi-sitting position with knees


bent

Left lateral with pillows between the


knees

Report an issue

What position should the nurse place the


head of the bed in to obtain the most
accurate reading of jugular vein distention:

High fowler’s

Raised 10 degrees

Raised 30 degrees (Correct


Answer)

Supine position

Report an issue

Which of the following body positions is


best for a patient with increased ICP:

file:///Users/mohammad/Desktop/PrometricMCQ.com.webarchive Page 150 of 377


PrometricMCQ.com 11/03/2015 10:45

Prone

Trendelenburg

HOB elevated 90 degrees and hips


flexed

HOB elevated 20 degrees (Correct


Answer)

Report an issue

Immediately after a percutaneous liver


biopsy, the nurse should place the client in
which of the following positions:

Left- sided lying

Right- sided lying (Correct Answer)

Lithotomy

Trendelenburg

Report an issue

An adult has low back pain. Which position


is likely to be most comfortable for the
client:

Prone

file:///Users/mohammad/Desktop/PrometricMCQ.com.webarchive Page 151 of 377


PrometricMCQ.com 11/03/2015 10:45

Supine

Side-lying with knees flexed


(Correct Answer)

Semi-sitting with legs extended

Report an issue

When planning the client’s postoperative


care, which is the least desirable position
in which the nurse can place the client:

Lying prone

Lying supine

Sitting in a chair (Correct Answer)

Standing to shower

Report an issue

Which of the following is the appropriate


position for a patient who is suffering
from acute asthma attack is:

Right lateral

Supine

High fowler (Correct Answer)

file:///Users/mohammad/Desktop/PrometricMCQ.com.webarchive Page 152 of 377


PrometricMCQ.com 11/03/2015 10:45

Prone

Report an issue

A patient with COPD is admitted to the


hospital. How can the nurse best position
the patient to improve gas exchange:

Sitting up at the bedside in a chair


and leaning slightly forward
(Correct Answer)

Resting in bed with the head


elevated to 45 to 60 degrees

In the Trendelenburg's position with


several pillows behind the head

Resting in bed in a high-Fowler's


position with the knees flexed

Report an issue

Following a tympanoplasty, the nurse


should maintain the client in which
position:

Semi-Fowler’s with the


operative ear facing down

Low Trendelenburg’s with the


head in neutral position

file:///Users/mohammad/Desktop/PrometricMCQ.com.webarchive Page 153 of 377


PrometricMCQ.com 11/03/2015 10:45

Flat with the head turned to the side


with the operative ear facing up
(Correct Answer)

Supine with a small neck roll to


allow for drainage

Report an issue

Which of the following is the proper


position for a patient undergoing a
cardiac surgery:

Supine (Correct Answer)

Prone

Lateral

Fowler’s

Report an issue

A nurse is caring for a toddler after surgical


repair of a cleft palate. The nurse
should position the child:

On his back

On his stomach (Correct Answer)

On his back with his head slightly


elevated

file:///Users/mohammad/Desktop/PrometricMCQ.com.webarchive Page 154 of 377


PrometricMCQ.com 11/03/2015 10:45

For comfort

Report an issue

The nurse observes that the infant's


anterior fontanelle is bulging after
placement of a ventriculoperitoneal shunt.
The nurse positions this infant:

Prone, with the head of the bed


elevated

Supine, with the head flat

Side-lying on the operative side

In a semi-fowler's position
(Correct Answer)

Report an issue

A nurse is providing care for a client


following surgery to remove a cataract
from the right eye. In which position should
the nurse place the client?

Right-side lying

Prone

Supine (Correct Answer)

Trendelenburg’s

file:///Users/mohammad/Desktop/PrometricMCQ.com.webarchive Page 155 of 377


PrometricMCQ.com 11/03/2015 10:45

Report an issue

An adult is admitted for a neurological


workup and is scheduled for a spinal
tap. When preparing the client for the
procedure, the nurse should position the
client in which position:

Prone

On the side with knees drawn up to


chest (Correct Answer)

Lithotomy

Semi-sitting

Report an issue

When suctioning the oropharynx, which of


the following is the proper position
of the patient:

Prone

Supine

Semi- Fowler’s (Correct


Answer)

Trendelenburg’s

file:///Users/mohammad/Desktop/PrometricMCQ.com.webarchive Page 156 of 377


PrometricMCQ.com 11/03/2015 10:45

Report an issue

To prevent headache after spinal


anesthesia the patient should be
positioned:

Semi- fowler’s

Flat on bed for 6 to 8 hours


(Correct Answer)

Prone position

Modified Trendelenburg

Report an issue

If a patient develops autonomic


hyperreflexia the first action his caring
nurse
should take is to:

Elevate head of bed 90 degree


(move from supine to sitting)
(Correct Answer)

Make the bed in flat position

Apply ice on the axillary and groin

But the patient in trendelenburg


position

file:///Users/mohammad/Desktop/PrometricMCQ.com.webarchive Page 157 of 377


PrometricMCQ.com 11/03/2015 10:45

Report an issue

To protect susceptible patients in the


hospital from aspiration pneumonia, the
nurse will plan to:

Turn and reposition immobile


patients at least every 2 hours.

Position patients with altered


consciousness in lateral positions.
(Correct Answer)

Monitor frequently for respiratory


symptoms in patients who are
immunosuppressed.

Provide for continuous subglottic


aspiration in patients receiving
enteral feedings.

Report an issue

Which nursing action is essential when


providing continuous enteral feeding?

Elevating the head of the bed


(Correct Answer)

Positioning the patient on the left


side

Warming the formula before


administrating it

file:///Users/mohammad/Desktop/PrometricMCQ.com.webarchive Page 158 of 377


PrometricMCQ.com 11/03/2015 10:45

Hanging a full day’s worth


formula at one time

Report an issue

Which of the following positions is


appropriate for the patient who has
hypovolemic shock:

Prone

Supine

Semi-fowler’s

Flat with elevating the feet (Correct


Answer)

Report an issue

When caring for a patient admitted post-


stroke (CVA) who has altered
consciousness, the nurse should place the
patient in which position:

Side-lying (Correct Answer)

Supine

Prone

Semi-fowler's

file:///Users/mohammad/Desktop/PrometricMCQ.com.webarchive Page 159 of 377


PrometricMCQ.com 11/03/2015 10:45

Report an issue

A nurse is providing instructions to a client


and the family regarding home care
after right eye cataract removal. Which
statement by the client would indicate an
understanding of the instructions:

“I should not sleep on my left


side.â€​

I should not sleep on my right side.â


€​ (Correct Answer)

“I should not sleep with my head


elevated.â€​

“I should not wear my glasses at


any time.â€​

Report an issue

The day after an amputation, the client


begins to hemorrhage from his stump.
What action should the nurse take first:

Apply a pressure dressing to the


stump

Place a tourniquet above the


stump (Correct Answer)

Notify the physician

file:///Users/mohammad/Desktop/PrometricMCQ.com.webarchive Page 160 of 377


PrometricMCQ.com 11/03/2015 10:45

Apply an ice pack to the stump

Report an issue

A 72-year-old male client has a total hip


replacement for long-standing
degenerative bone disease of the hip.
When assessing this client postoperatively,
the nurse considers that the most common
complication of hip surgery is:

Pneumonia

Hemorrhage

Wound infection

Pulmonary embolism (Correct


Answer)

Report an issue

The nurse is caring for a client who had a


right below-the-knee amputation
three days ago. The client complains of
pain in the right foot and asks for pain
medication. What nursing action is
appropriate initially?

Elevate the stump

Administer a placebo

Administer ordered medications

file:///Users/mohammad/Desktop/PrometricMCQ.com.webarchive Page 161 of 377


PrometricMCQ.com 11/03/2015 10:45

(Correct Answer)

Encourage the client to discuss his


feelings

Report an issue

During the assessment of an injury for a


possible fracture, which of the
following manifestations would make you
suspicious that a fracture has
occurred:

Impaired sensation

Hotness

Loss of function (Correct Answer)

Pointed tenderness

Report an issue

A client who has had an above-the-knee


amputation develops a dime-sized
bright red spot on the dressing after 45
minutes in the postanesthesia recovery
unit. The nurse should:

Elevate the stump

Reinforce the dressing

Call the surgeon

file:///Users/mohammad/Desktop/PrometricMCQ.com.webarchive Page 162 of 377


PrometricMCQ.com 11/03/2015 10:45

Draw a mark around the site


(Correct Answer)

Report an issue

How you will interfere to relieve swelling of


the lower extremity in a cast for 3 days
now:

Change the cast

Call the doctor

Elevate the extremity (Correct


Answer)

Give massage

Report an issue

The nurse is developing a bowel-retraining


plan for a client with multiple sclerosis.
Which measure is likely to be least helpful
to the client:

Limiting fluid intake to 1000mL per


day (Correct Answer)

Providing a high-roughage diet

Elevating the toilet seat for easy


access

file:///Users/mohammad/Desktop/PrometricMCQ.com.webarchive Page 163 of 377


PrometricMCQ.com 11/03/2015 10:45

Establishing a regular schedule for


toileting

Report an issue

A diabetic patient with foot gangrene


undergone above knee amputation, while
the
nurse changing the dressing he complains
of pain on the same knee which was
amputated. What should the nurse do:

Inform physician about it

Re-do dressing to Assessment the


wound

Psychiatry consultation to the


patient because he wound above
knee

Give analgesic as needed (Correct


Answer)

Report an issue

A diabetic patient with foot gangrene


undergone above knee amputation he
complain of pain and swelling at the
wound site which is oozing bus and has a
bad
odor, the physician ordered cephalexin and
metronidazole. Which of the following
you should do immediately:

file:///Users/mohammad/Desktop/PrometricMCQ.com.webarchive Page 164 of 377


PrometricMCQ.com 11/03/2015 10:45

Give cephalexin direct first action

Give metronidazole direct first action

Do wash on wound with N/Sand put


bacitracin (Correct Answer)

Do wash on wound with N/Sand put


hydrocortisone

Report an issue

The physician has prescribed a cleansing


enema to a client scheduled for colon
surgery. The nurse would place the client:

Prone

Supine

Left sim’s ( left lateral )


(Correct Answer)

Dorsal recumbent

Report an issue

At which side of the patient will the nurse


stand when inserting a rectal enema:

Right side (Correct Answer)

Left side

file:///Users/mohammad/Desktop/PrometricMCQ.com.webarchive Page 165 of 377


PrometricMCQ.com 11/03/2015 10:45

Any side

Both sides

Report an issue

While undergoing a soapsuds enema, the


client complains of abdominal cramping.
The nurse should:

Immediately stop the infusion

Lower the height of the enema


container (Correct Answer)

Advance the enema tubing 2 to 3


inches

Clamp the tubing

Report an issue

The nurse must administer an enema to an


adult patient with constipation. Which
of the following would be a safe and
effective distance for the nurse to insert the
tubing into the patient's rectum:

1:2 cm

3:4 cm

5.5:6.5 cm

file:///Users/mohammad/Desktop/PrometricMCQ.com.webarchive Page 166 of 377


PrometricMCQ.com 11/03/2015 10:45

6.5:8 cm (Correct Answer)

Report an issue

The nurse is caring for a patient receiving


IV furosemide (Lasix) 40 mg and
Enalapril (Vasotec) 5 mg PO bid for ADHF
with severe orthopnea. When evaluating
the patient response to the medications,
the best indicator that the treatment has
been effective is:

Weight loss of 2 pounds overnight.

Improvement in hourly urinary


output.

Reduction in systolic bp.

Decreased dyspnea with the head of


the bed at 30 degrees. (Correct
Answer)

Report an issue

The nurse in preparing to insert RYLE’S


tube (NGT) into an infant, the nurse
knows that the length of the tube should
be taken as following:

file:///Users/mohammad/Desktop/PrometricMCQ.com.webarchive Page 167 of 377


PrometricMCQ.com 11/03/2015 10:45

From the nose down to the chin and


then to the umbilicus

From the nose to the earlobe and


then to the xiphoid process
(Correct Answer)

From the nose to the mouth to the


xiphoid process

From the nose to the earlobe to the


umbilicus

Report an issue

The nurse is caring for a client who has


had a chest tube inserted and connected
to
water seal drainage. The nurse determines
the drainage system is functioning
correctly when which of the following is
observed:

Continuous bubbling in the water


seal chamber

Fluctuation in the water seal


chamber (Correct Answer)

Suction tubing attached to a wall

file:///Users/mohammad/Desktop/PrometricMCQ.com.webarchive Page 168 of 377


PrometricMCQ.com 11/03/2015 10:45

unit

Vesicular breath sounds throughout


the lung fields

Report an issue

The nurse is caring for a client who has just


had a chest tube attached to a water
seal drainage system. To ensure that the
system is functioning effectively the nurse
should:

Observe for intermittent bubbling in


the water seal chamber (Correct
Answer)

Flush the chest tubes with 30-60 ml


of NSS every 4-6 hours

Maintain the client in an extreme


lateral position

Strip the chest tubes in the direction


of the client

Report an issue

The nurse enters the room of a client who


has a chest tube attached to a water seal
drainage system and notices the chest
tube is dislodged from the chest. The most
appropriate nursing intervention is to:

file:///Users/mohammad/Desktop/PrometricMCQ.com.webarchive Page 169 of 377


PrometricMCQ.com 11/03/2015 10:45

Notify the physician

Insert a new chest tube

Cover the insertion site with


petroleum gauze (Correct Answer)

Instruct the client to breathe deeply


until help arrives

Report an issue

which type of isolation category is


indicated for patient with tuberculosis:

Airborne isolation (Correct Answer)

Strict isolation

Reverse isolation

Contact isolation

Report an issue

While attempting to get out of bed, a


patient accidentally disconnects the chest
tube
from the Pleur-evac drainage system.
Which of the following actions should the
nurse take first?

Insert the end of the chest tube in a


container of sterile solution

file:///Users/mohammad/Desktop/PrometricMCQ.com.webarchive Page 170 of 377


PrometricMCQ.com 11/03/2015 10:45

(Correct Answer)

Clamp the chest tube near the


Pleur-evac drainage system

Raise the end of the chest tube


above the level of the insertion of
the chest tube

Apply pressure dressing to the chest


tube insertion site

Report an issue

which type of isolation category is


indicated for a burn patient:

Airborne isolation

Strict isolation

Reverse isolation (Correct Answer)

No isolation required

Report an issue

which type of isolation category is


indicated for patient with diphtheria:

Airborne

Droplet (Correct Answer)

file:///Users/mohammad/Desktop/PrometricMCQ.com.webarchive Page 171 of 377


PrometricMCQ.com 11/03/2015 10:45

Contact

Blood

Report an issue

A 68-years-old woman diagnosed with


thrombocytopenia due to acute
lymphocytic
leukemia is admitted to the hospital. The
nurse should assign the patient to a:

To a private room so she will not


infect other patients and health care
workers.

To a private room so she will not be


infected by other patients and health
care workers. (Correct Answer)

To a semiprivate room so she will


have stimulation during her
hospitalization.

To a semiprivate room so she will


have the opportunity to express her
feelings about her illness.

Report an issue

A nurse who begins to administer


medications to a client via a nasogastric
feeding
tube suspects that the tube has become
clogged. The nurse should take which safe

file:///Users/mohammad/Desktop/PrometricMCQ.com.webarchive Page 172 of 377


PrometricMCQ.com 11/03/2015 10:45

action first:

Aspirate the tube (Correct Answer)

Flush the tube with warm water

Prepare to remove and replace the


tube

Flush with a carbonated liquid such


as cola

Report an issue

Which of the following instructions is


appropriate for the nurse to give to a
female
client a who complains of abdominal upset
after cholecystectomy operation:

Increase fluid intake

Avoid fatty meals (Correct Answer)

Increase protein intake

Daily exercise

Report an issue

Which of the following pulses should be


checked before administrating Digoxin:

file:///Users/mohammad/Desktop/PrometricMCQ.com.webarchive Page 173 of 377


PrometricMCQ.com 11/03/2015 10:45

Apical pulse (Correct Answer)

Radial pulse

Femoral pulse

Dorsalis pedis pulse

Report an issue

which of the following interventions must


the nurse take when administrating
digoxin to the patient:

Give him the medication with a glass


of orange juice

Check him for signs of hypokalemia


before giving the medication

Instruct him to place the medication


under the tongue

Withhold the medication if his pulse


is less than 60 beats/ minute
(Correct Answer)

Report an issue

The nurse must withhold Digoxin from a


patient if his pulse rate is:

Less than 45/m

file:///Users/mohammad/Desktop/PrometricMCQ.com.webarchive Page 174 of 377


PrometricMCQ.com 11/03/2015 10:45

More than 60/m

Less than 60/m (Correct Answer)

More than 100/m

Report an issue

The nurse is assessing the client for


possible evidence of digitalis toxicity. The
nurse
acknowledges that which is included in the
signs and symptoms for digitalis toxicity:

Pulse (heart) rate of 100 beats/min

Pulse of 72 with an irregular rate

Pulse greater than 60 beats/min and


irregular rate

Pulse below 60 beats/min and


irregular rate (Correct Answer)

Report an issue

A newly admitted client takes digoxin 0.25


mg/day. The nurse knows that the
serum therapeutic range for digoxin is:

0.1 to 1.5 ng/mL

0.5 to 2.0 ng/mL (Correct Answer)

file:///Users/mohammad/Desktop/PrometricMCQ.com.webarchive Page 175 of 377


PrometricMCQ.com 11/03/2015 10:45

1.0 to 2.5 ng/mL

2.0 to 4.0 ng/mL

Report an issue

A patient complains of severe pain which


he stated to be 9/10, the physician
ordered morphine 50 mg IV every 4 hours,
the last dose was given 2 hours ago, what
is the best action his caring nurse would
take:

Give another dose of morphine

Inform the doctor to change the


order (Correct Answer)

Distract the patient by TV, radio or


games for 2 hours

Ignore the patient completely

Report an issue

The nurse is caring for a patient admitted 1


week ago with an acute spinal cord
injury. Which of the following assessment
findings would alert the nurse to the
presence of autonomic dysreflexia
(hyperreflexia)?

Tachycardia

file:///Users/mohammad/Desktop/PrometricMCQ.com.webarchive Page 176 of 377


PrometricMCQ.com 11/03/2015 10:45

Hypotension

Hot, dry skin

Throbbing headache (Correct


Answer)

Report an issue

Which of the following is the best way for


long term feeding for a patient
experiencing severe dysphagia:

NGT

Naso-dudenal tube

Gastrostomy (Correct Answer)

Parenteral

Report an issue

Patient records are as following, he had


650cc of IV fluid (n/s), 50 ml of orange
Juice in the NG tube, 100 water with the
medications, and urinate 500 cc, and 100
cc
collected in the NG bag. What is the total
intake for this patient?

800 cc (Correct Answer)

file:///Users/mohammad/Desktop/PrometricMCQ.com.webarchive Page 177 of 377


PrometricMCQ.com 11/03/2015 10:45

650 cc

200 cc

600 cc

Report an issue

The physician ordered to give the patient


10,000 unit of heparin, the preparation
available is 40,000/ml. how much would
the nurse give this patient:

4 ml

2.5 ml

0.25 ml (Correct Answer)

25 ml

Report an issue

The nurse is caring for a client with a


fracture. The client develops a deep vein
thrombosis in the opposite extremity.
Physician orders include a heparin drip of
D5W 250 mL with heparin 12,500 units at
16 mL/hr. How many units of heparin is
this client receiving per hour:

200 unit/h

400 unit/h

file:///Users/mohammad/Desktop/PrometricMCQ.com.webarchive Page 178 of 377


PrometricMCQ.com 11/03/2015 10:45

600 unit/h

800 unit/h (Correct Answer)

Report an issue

A patient suffers from diarrhea and muscle


weakness, the nurse would assess this
patient for which of the following abnormal
laboratory blood tests:

Blood calcium

Blood magnesium

Blood potassium (Correct Answer)

Blood sodium

Report an issue

When assessing for therapeutic effects of


mannitol, the nurse would expect to see:

Decreased intracranial pressure


(Correct Answer)

Decreased excretion of therapeutic


medications

Increased urine osmolality

Decreased serum osmolality

file:///Users/mohammad/Desktop/PrometricMCQ.com.webarchive Page 179 of 377


PrometricMCQ.com 11/03/2015 10:45

Report an issue

Which drug would be used to treat a


patient who has increased intracranial
pressure (ICP) resulting from head trauma
after an accident:

Mannitol (Correct Answer)

Atropine sulfate

Epinephrine hydrochloride

Sodium bicarbonate

Report an issue

Which of the following is the drug of choice


for treatment of cerebral edema:

Mannitol (Correct Answer)

Atropine sulfate

Epinephrine hydrochloride

Sodium bicarbonate

Report an issue

file:///Users/mohammad/Desktop/PrometricMCQ.com.webarchive Page 180 of 377


PrometricMCQ.com 11/03/2015 10:45

The nurse would assess which laboratory


value to determine the effectiveness of
intravenous heparin:

Complete blood count (CBC)

Activated partial thromboplastin


time (aPTT) (Correct Answer)

Prothrombin time (PT)

BUN

Report an issue

The antidote to heparin is:

Aspirin

Vitamin k

Warfarin

Protamine sulfate (Correct Answer)

Report an issue

How do we monitor the effectiveness of


Coumadin (Warfarin):

file:///Users/mohammad/Desktop/PrometricMCQ.com.webarchive Page 181 of 377


PrometricMCQ.com 11/03/2015 10:45

Monitor vital signs

PT/INR (Correct Answer)

Weight

Ask the patient how they feel

Report an issue

Which medication is used to treat a patient


suffering from severe adverse effects of
a narcotic analgesic:

Naloxone (Narcan) (Correct


Answer)

Acetylcysteine (Mucomyst)

Methylprednisolone (Solu-Medrol)

Protamine sulfate

Report an issue

A patient who has been anticoagulated


with warfarin (Coumadin) is admitted with
gastrointestinal bleeding. The nurse will
anticipate administering which substance:

Vitamin E

Vitamin K (Correct Answer)

file:///Users/mohammad/Desktop/PrometricMCQ.com.webarchive Page 182 of 377


PrometricMCQ.com 11/03/2015 10:45

Protamine sulfate

Calcium gluconate

Report an issue

While admitting a patient for treatment of


an acetaminophen overdose, the nurse
prepares to administer which of the
following medications to prevent toxicity:

Naloxone (Narcan)

Acetylcysteine (Mucomyst)
(Correct Answer)

Methylprednisolone (Solu-Medrol)

Vitamin K

Report an issue

If the nurse gave a patient antihypertensive


drug that is prescribed for another
patient, what should she do:

Ignore what she has done

Call urgent CPR

Document that in nursing report


(Correct Answer)

Inform doctor after 2 days

file:///Users/mohammad/Desktop/PrometricMCQ.com.webarchive Page 183 of 377


PrometricMCQ.com 11/03/2015 10:45

Report an issue

When a physician orders an arterial blood


gas, which artery is the appropriate one
to use to obtain the specimen:

Cerebral artery

Radial artery (Correct Answer)

Carotid artery

Ulnar artery

Report an issue

A client has an order to have a set of


arterial blood gases (ABG's) drawn. The
intended site is the radial artery. The nurse
ensures that which of the following is
positive before the ABGs are drawn

Homan’s sign

Brudzinski's sign

Babinski reflex

Allen's test (Correct Answer)

Report an issue

file:///Users/mohammad/Desktop/PrometricMCQ.com.webarchive Page 184 of 377


PrometricMCQ.com 11/03/2015 10:45

A client is scheduled for blood to be drawn


from the radial artery for an ABG
determination. A nurse assists with
performing Allen's test before drawing the
blood to determine the adequacy of the:

Ulnar circulation (Correct Answer)

Carotid circulation

Femoral circulation

Brachial circulation

Report an issue

A nurse has administered approximately


half of an enema solution to a
preoperative client when the client
complains of pain and cramping. Which
nursing action is the most appropriate:

Raise the enema bag so that the


solution can be instilled quickly.

Clamp the tubing for 30 seconds


and restart the flow at a slower
rate. (Correct Answer)

Reassure the client and continue the


flow.

Discontinue the enema and notify

file:///Users/mohammad/Desktop/PrometricMCQ.com.webarchive Page 185 of 377


PrometricMCQ.com 11/03/2015 10:45

the physician

Report an issue

When caring for a client who is receiving


total parenteral nutrition (TPN), which
of the following complications would be
most important for the nurse to assess:

Chest pain

Hemorrhage and air embolus

Pneumonia and hyperglycemia

Electrolyte imbalance and sepsis


(Correct Answer)

Report an issue

A client has a nasogastric tube after a


gastric resection. The nurse should expect
to
observe:

Vomiting

Gastric distention

Intermittent periods of diarrhea

Bloody drainage for the first 12


hours (Correct Answer)

file:///Users/mohammad/Desktop/PrometricMCQ.com.webarchive Page 186 of 377


PrometricMCQ.com 11/03/2015 10:45

Report an issue

While assessing a client with


hypoparathyroidism, the nurse taps the
client's facial
nerve and observes twitching of the mouth
and tightening of the jaw. The nurse
would document this finding as which of
the following:

Positive Trousseau's sign

Positive Chvostek's sign (Correct


Answer)

Tetany

Hyperactive deep tendon reflex

Report an issue

Which of the following clinical


manifestations of type 2 diabetes occurs if
glucose
levels are very high:

Hyperactivity

Blurred vision (Correct Answer)

Oliguria

Increased energy

file:///Users/mohammad/Desktop/PrometricMCQ.com.webarchive Page 187 of 377


PrometricMCQ.com 11/03/2015 10:45

Report an issue

Which of the following would the nurse


expect to find in a client with severe
hyperthyroidism:

Tetany

Buffalo hump

Exophthalmos (Correct Answer)

Striae

Report an issue

A client is receiving long-term treatment


with high-dose corticosteroids. Which of
the following would the nurse expect the
client to exhibit:

Weight loss

Pale thick skin

Hypotension

Moon face (Correct Answer)

Report an issue

file:///Users/mohammad/Desktop/PrometricMCQ.com.webarchive Page 188 of 377


PrometricMCQ.com 11/03/2015 10:45

After undergoing a thyroidectomy, a client


develops hypocalcemia and tetany.
Which electrolyte should the nurse
anticipate administering:

Sodium phosphorus

Sodium bicarbonate

Calcium gluconate (Correct


Answer)

Potassium chloride

Report an issue

The nurse who elicits a positive Chvostek’s


sign would suspect that the patient has
which condition:

Hyperkalemia

Hypocalcemia (Correct Answer)

Hypercalcemia

Hypernatremia

Report an issue

Which of the following is the best method

file:///Users/mohammad/Desktop/PrometricMCQ.com.webarchive Page 189 of 377


PrometricMCQ.com 11/03/2015 10:45

to decrease confusion and irritability


for asthmatic patient:

Give antibiotic

Give sedative

Give vasodilator

Give oxygen (Correct Answer)

Report an issue

Which one of the baseline vital signs that


has the most effect on spO2:

Heart rate

Respiratory rate (Correct Answer)

Blood pressure

Temperature

Report an issue

A patient is admitted to the emergency


department with a stab wound to the right
chest. Air can be heard entering his chest
with each inspiration. To decrease the
possibility of a tension pneumothorax in
the patient, the nurse should:

file:///Users/mohammad/Desktop/PrometricMCQ.com.webarchive Page 190 of 377


PrometricMCQ.com 11/03/2015 10:45

Position the patient so that the right


chest is dependent.

Administer high-flow oxygen using a


non-rebreathing mask.

Cover the sucking chest wound with


an occlusive dressing.

Tape a nonporous dressing on three


sides over the chest wound.
(Correct Answer)

Report an issue

A client with pneumonia has a temperature


of 102.6 ° F (39.2 ° C), is
diaphoretic, and has a productive cough.
The nurse should include which of
the following measures in the plan of care:

Position changes every 4 hours.

Nasotracheal suctioning to clear


secretions.

Frequent linen changes (Correct


Answer)

Frequent offering of a bedpan

Report an issue

The nurse reviews an arterial blood gas


report for a client with chronic

file:///Users/mohammad/Desktop/PrometricMCQ.com.webarchive Page 191 of 377


PrometricMCQ.com 11/03/2015 10:45

obstructive pulmonary disease (COPD). PH


7.35; PC02 62; PO2 70; HCO3 34
the nurse should:

Apply a 100% non-rebreather mask.

Assess the vital signs. (Correct


Answer)

Reposition the client.

Prepare for intubation

Report an issue

If the nurse notes the following symptoms


after the client begins taking
sertraline (Zoloft), which one is most likely
drug-related:

Polyuria

Diplopia

Drooling

Insomnia (Correct Answer)

Report an issue

Which of the following diets would be most


appropriate for a client with
chronic obstructive pulmonary disease
(COPD):

file:///Users/mohammad/Desktop/PrometricMCQ.com.webarchive Page 192 of 377


PrometricMCQ.com 11/03/2015 10:45

Low-fat, low-cholesterol diet.

Bland, soft diet.

Low-sodium diet.

High-calorie, high-protein diet.


(Correct Answer)

Report an issue

The physician has prescribed sertraline


(Zoloft) 50 mg daily for a client with
depression. Which finding should be
reported to the physician?

The client takes Tagamet


(cimetidine) for acid reflux (Correct
Answer)

The client takes the medication with


meals

The client takes the medication once


a day at bedtime

The client takes Aleve (naproxen) for


arthritis

Report an issue

A patient was diagnosed with depression 6

file:///Users/mohammad/Desktop/PrometricMCQ.com.webarchive Page 193 of 377


PrometricMCQ.com 11/03/2015 10:45

weeks ago, and was prescribed


anti-depressant drug.1 week ago the
patient started to complain of irritability
and had difficult sleep for only 3 hours
daily. Which of the following is the
best nursing action:

Inform the doctor to increase the


dose

Ignore the patient complaint

That’s a side effect of the


medication (Correct Answer)

Give the patient hypnotics

Report an issue

16 years old girl tried to suicide by taking a


large amount of valium
(benzodiazepines), the nurse knows that
the antidote for this drug is:

Flumazenil (Correct Answer)

Benztropine

Meperidine

Naloxone

Report an issue

file:///Users/mohammad/Desktop/PrometricMCQ.com.webarchive Page 194 of 377


PrometricMCQ.com 11/03/2015 10:45

The nurse would expect to find which


information when reviewing the history
of a client diagnosed with multiple
sclerosis:

Visual problems (Correct Answer)

Increased sensitivity to pain

Ascending weakness and numbness

Confusion and disorientation

Report an issue

The antidote for morphine overdose is:

Naloxone (Narcan) (Correct


Answer)

Flumazenil (Romazicon)

Benztropine (Cogentin)

Meperidine (Demerol)

Report an issue

The nurse assesses a respiratory rate of 10


on a client with cancer who has just
received a hydromorphone hydrochloride
(Dilaudid) injection. Which drug
should the nurse be prepared to
administer:

file:///Users/mohammad/Desktop/PrometricMCQ.com.webarchive Page 195 of 377


PrometricMCQ.com 11/03/2015 10:45

Naloxone (Narcan) (Correct


Answer)

Flumazenil (Romazicon)

Benztropine (Cogentin)

Meperidine (Demerol)

Report an issue

Which of the following drugs is the antidote


for magnesium toxicity:

Calcium gluconate (Kalcinate)


(Correct Answer)

Hydralazine (Apresoline)

Naloxone (Narcan)

Rho (D) immune globulin (RhoGAM)

Report an issue

A client with myasthenia gravis is


suspected of having cholinergic crisis.
Which of the following indicate that this
crisis exists:

Ataxia

file:///Users/mohammad/Desktop/PrometricMCQ.com.webarchive Page 196 of 377


PrometricMCQ.com 11/03/2015 10:45

Mouth sores

Hypotension

Hypertension (Correct Answer)

Report an issue

A client with myasthenia gravis is receiving


pyridostigmine (Mestinon). The
nurse monitors for signs and symptoms of
cholinergic crisis caused by
overdose of the medication. The nurse
checks the medication supply to ensure
that which medication is available for
administration if a cholinergic crisis
occurs:

Vitamin K

Atropine sulfate (Correct Answer)

Protamine sulfate

Acetylcysteine (Mucomyst)

Report an issue

Which of the following is the purpose of


providing atropine prior to anesthesia
and surgery:

To decrease salivation (decrease

file:///Users/mohammad/Desktop/PrometricMCQ.com.webarchive Page 197 of 377


PrometricMCQ.com 11/03/2015 10:45

secretions) (Correct Answer)

To increase blood pressure

To facilitate breathing

To decrease muscle tone

Report an issue

A client is diagnosed as having secondary


Cushing’s syndrome. The nurse
knows that the client has most likely been
taking which medication:

Estrogen

Penicillin

Lovastatin

Prednisone (Correct Answer)

Report an issue

A patient has the following preoperative


medication order: morphine 10 mg
with atropine 0.4 mg IM. The nurse informs
the patient that this injection will:

Decrease nausea and vomiting


during and after surgery

Decrease oral and respiratory

file:///Users/mohammad/Desktop/PrometricMCQ.com.webarchive Page 198 of 377


PrometricMCQ.com 11/03/2015 10:45

secretions, thereby drying the


mouth (Correct Answer)

Decrease anxiety and produce


amnesia of the preoperative period

Induce sleep, so the patient will not


be aware during transport to the
operating room

Report an issue

A 40-year-old woman is admitted in labor


with high blood pressure, edema,
and proteinuria. She is started on
magnesium sulfate. The nurse caring for
her
should be sure to keep which drug at the
bedside:

Calcium gluconate (Correct


Answer)

Naloxone (Narcan)

Phenytoin (Dilantin)

Glucose

Report an issue

Which of the following diets would be most


appropriate for a patient has
Cushing syndrome:

file:///Users/mohammad/Desktop/PrometricMCQ.com.webarchive Page 199 of 377


PrometricMCQ.com 11/03/2015 10:45

High protein diet

High potassium diet

High calcium diet (Correct Answer)

High iron diet

Report an issue

Vitamin K is prescribed for a neonate. A


nurse prepares to administer the
medication in which muscle site:

Deltoid

Triceps

Vastus lateralis (Correct Answer)

Biceps

Report an issue

Which of the following interventions should


be taken to help an older client to
prevent osteoporosis:

Decrease dietary calcium intake

Increase sedentary lifestyles

Increase dietary protein intake

file:///Users/mohammad/Desktop/PrometricMCQ.com.webarchive Page 200 of 377


PrometricMCQ.com 11/03/2015 10:45

Encourage regular exercise


(Correct Answer)

Report an issue

An adult is admitted with heart failure. The


nurse notes that he has neck vein
distention and slight peripheral edema. The
nurse knows that these signs
indicate which of the following:

Pneumothorax

Right-sided heart failure (Correct


Answer)

Cardiogenic shock

Left-sided heart failure

Report an issue

An adult client is admitted with a diagnosis


of left-sided congestive heart
failure. Which assessment finding would
most likely be present?

Distended neck veins

Dyspnea (Correct Answer)

Hepatomegaly

file:///Users/mohammad/Desktop/PrometricMCQ.com.webarchive Page 201 of 377


PrometricMCQ.com 11/03/2015 10:45

Pitting edema

Report an issue

When the nurse on duty accidentally


bumps the bassinet, the neonate throws
out its arms, hands opened, and begins to
cry. The nurse interprets this
reaction as indicative of which of the
following reflexes:

Moro Reflex (Correct Answer)

Babinski reflex

Grasping Reflex

Tonic Neck Reflex

Report an issue

A patient who has required prolonged


mechanical ventilation has the
following arterial blood gas results: pH
7.48, PaO2 85 mm Hg, PaCO2 32 mm
Hg, and HCO3 25 mEq/L. The nurse
interprets these results as:

Metabolic acidosis

Metabolic alkalosis

Respiratory acidosis

file:///Users/mohammad/Desktop/PrometricMCQ.com.webarchive Page 202 of 377


PrometricMCQ.com 11/03/2015 10:45

Respiratory alkalosis (Correct


Answer)

Report an issue

A patient has the following ABG results: pH


7.32, PaO2 88 mm Hg, PaCO2 37
mm Hg, and HCO3 16 mEq/L. The nurse
interprets these results as:

Respiratory acidosis

Respiratory alkalosis

Metabolic acidosis (Correct


Answer)

Metabolic alkalosis

Report an issue

The nurse forgot to give the patient his


prescribed drug, when the head nurse
asked who is responsible for this the nurse
replayed that was my wrong this
considered:

Responsibility (Correct Answer)

Ignorance

Abandonment

Honor

file:///Users/mohammad/Desktop/PrometricMCQ.com.webarchive Page 203 of 377


PrometricMCQ.com 11/03/2015 10:45

Report an issue

When the patient vomits postoperatively,


the most important nursing
objective is to prevent:

Dehydration

Aspiration (Correct Answer)

Rupture of suture line

Met. Alkalosis

Report an issue

The nurse is discussing dietary sources of


iron with a client who has iron
deficiency anemia. Which menu, if selected
by the client, indicates the best
understanding of the diet?

Milkshake, hot dog, and beets

Beef steak, spinach, and grape


juice (Correct Answer)

Chicken salad, green peas, and


coffee

Macaroni and cheese, coleslaw, and


lemonade

file:///Users/mohammad/Desktop/PrometricMCQ.com.webarchive Page 204 of 377


PrometricMCQ.com 11/03/2015 10:45

Report an issue

A client is recovering from abdominal


surgery and has a large abdominal
wound. A nurse encourages the client to
eat which food item that is naturally
high in vitamin C to promote wound
healing:

Milk

Oranges (Correct Answer)

Bananas

Chickens

Report an issue

Which of the following reflexes is


considered a normal reflex of the newborn:

Moro reflex

Rooting reflex (Correct Answer)

Planter reflex

Tonic neck reflex

Report an issue

file:///Users/mohammad/Desktop/PrometricMCQ.com.webarchive Page 205 of 377


PrometricMCQ.com 11/03/2015 10:45

Patient with aortic aneurysm, which of the


following is the most dangerous
complications:

Embolism

Rupture (Correct Answer)

Stenosis

Hypotension

Report an issue

56 years old patient suffers from heart


failure, when the nurse examines his
lungs she noticed the presence of crackles
which means that the patient has:

Right-sided- failure

Left-sided- failure (Correct


Answer)

Biventricular failure

Congestive heart failure

Report an issue

The nurse teaches a client with chronic

file:///Users/mohammad/Desktop/PrometricMCQ.com.webarchive Page 206 of 377


PrometricMCQ.com 11/03/2015 10:45

obstructive pulmonary disease


(COPD) to assess for signs and symptoms
of right-sided heart failure. Which
of the following signs and symptoms
should be included in the teaching plan:

Clubbing of nail beds

Hypertension

Peripheral edema (Correct Answer)

Increased appetite

Report an issue

A surgery procedure that must be done


within 24 hours is called:

Selective surgery

Elective surgery

Urgent surgery (Correct Answer)

Emergency surgery

Report an issue

Early sign of ARDS in a patient at risk:

Elevated CO2 level

file:///Users/mohammad/Desktop/PrometricMCQ.com.webarchive Page 207 of 377


PrometricMCQ.com 11/03/2015 10:45

Hypoxia not responsive to O2


(cyanosis) (Correct Answer)

Metabolic acidosis

Severe, unexplained light imbalance

Report an issue

A nurse is assessing a female client with


multiple trauma who is at risk for
developing acute respiratory distress
syndrome. The nurse assesses for which
earliest sign of acute respiratory distress
syndrome:

Bilateral wheezing

Inspiratory crackles

Intercostal retractions

Increased respiratory ra (Correct


Answer)

Report an issue

36 years old male patient complaining of


fever and headache for 3 days now,
when examining this patent which of the
following signs would indicate that
this patient may has meningitis:

file:///Users/mohammad/Desktop/PrometricMCQ.com.webarchive Page 208 of 377


PrometricMCQ.com 11/03/2015 10:45

Positive Kernig’s sign (Correct


Answer)

Negative Brudzinski’s sign

Positive homan’s sign

Negative Kernig’s sign

Report an issue

A patient presented with high fever,


headache, vomiting and neck stiffness for
the past 3 days, which of the following is
the first diagnostic intervention for
this patient:

Urine and stool analysis

Lumber puncture with CSF


aspiration (Correct Answer)

Complete blood count

Chest and abdomen x-ray

Report an issue

MRSA (methicillin resistant


Staphylococcus aureus) is transmitted by:

Contact (Correct Answer)

Airborne

file:///Users/mohammad/Desktop/PrometricMCQ.com.webarchive Page 209 of 377


PrometricMCQ.com 11/03/2015 10:45

Droplet

Standard

Report an issue

What is the best indicator of effective


treatment of CHF in patient taking
Lasix?

Calculating total intake daily

Weighting himself daily (Correct


Answer)

Calculating urine output daily

Assessing fluid status daily

Report an issue

What supplemental medication is most


frequently ordered in conjunction with
furosemide (Lasix)?

Chloride

Digoxin

Potassium (Correct Answer)

Sodium

file:///Users/mohammad/Desktop/PrometricMCQ.com.webarchive Page 210 of 377


PrometricMCQ.com 11/03/2015 10:45

Report an issue

What is the first intervention for a client


experiencing myocardial infarction?

Administer morphine

Administer oxygen (Correct


Answer)

Administer sublingual nitroglycerine

Obtain an electrocardiogram

Report an issue

Which type of medications is commonly


used to treat rheumatoid arthritis?

Glucocorticoids

Non- steroidal ant- inflammatory


drugs (NSAIDs) (Correct Answer)

Antimalarial drugs

Gold salt

Report an issue

file:///Users/mohammad/Desktop/PrometricMCQ.com.webarchive Page 211 of 377


PrometricMCQ.com 11/03/2015 10:45

A male client undergoes total gastrectomy.


Several hours after surgery, the
nurse notes that the client’s nasogastric
(NG) tube has stopped draining. How
should the nurse respond?

Notify the physician (Correct


Answer)

Reposition the tube

Irrigate the tube

Increase the suction level

Report an issue

Postoperatively, patient is expected to void


after:

6-8 hours (Correct Answer)

2-4 hours

12-24 hours

10-12 hours

Report an issue

file:///Users/mohammad/Desktop/PrometricMCQ.com.webarchive Page 212 of 377


PrometricMCQ.com 11/03/2015 10:45

A female client has severe menstrual pain,


which of the following drugs you
would recommend for this patient to relieve
her dysmenorrhea:

Ibuprofen (Correct Answer)

Zantac

Aspirin

Cortisone

Report an issue

The nurse must suction a child with a


tracheostomy. Interventions should
include:

Encouraging the child to cough to


raise the secretions before
suctioning

Selecting a catheter with a diameter


three fourths as large as the
diameter of the tracheostomy tube

Ensuring that each pass of the


suction catheter take no longer than
5 seconds (Correct Answer)

Allowing the child to rest after every


five times the suction catheter is
passed

Report an issue

file:///Users/mohammad/Desktop/PrometricMCQ.com.webarchive Page 213 of 377


PrometricMCQ.com 11/03/2015 10:45

A 40 years old female complains of


abdominal pain. It is worse after eating,
especially if she has a meal that is spicy or
high in fat. She has tried over-thecounter
antacids, but they have not helped the
pain. After examining her
abdomen, you strongly suspect
cholecystitis. Which sign on examination
increases your suspicion for this diagnosis:

Psoas sign

Rovsing’s sign

Murphy’s sign (Correct


Answer)

Grey turner’s sign

Report an issue

Which pulse should the nurse palpate


during rapid assessment of an
unconscious male adult?

Radial

Brachial

Femoral

Carotid (Correct Answer)

Report an issue

file:///Users/mohammad/Desktop/PrometricMCQ.com.webarchive Page 214 of 377


PrometricMCQ.com 11/03/2015 10:45

The most important nursing measure in the


prevention of thrombophlebitis
for the post-partum mother is:

Elastic stocking

Early ambulation (Correct Answer)

Anticoagulants

Isometric exercises

Report an issue

Which of the following is the best aid to


prevent breast cancer:

Teaching women about breast


cancer

Public knowledge about


chemotherapy

To eat fruits and vegetables only

Encourage women to perform self-


breast examination monthly
(Correct Answer)

Report an issue

file:///Users/mohammad/Desktop/PrometricMCQ.com.webarchive Page 215 of 377


PrometricMCQ.com 11/03/2015 10:45

Which of the following factors would


contribute to a high risk pregnancy:

Blood type O positive

First pregnancy at 33 years old

History of allergy to honey bee


pollen

History of insulin dependent DM


(Correct Answer)

Report an issue

When the nurse puts an infant with high


temperature in cold water in order to
lower his temperature, she knows that the
mechanism of heat loss is called:

Evaporation

Conduction (Correct Answer)

Radiation

Convection

Report an issue

A child is admitted to the hospital with a


diagnosis of Wilm's tumor, Stage II.
Which of the following statements most
accurately describes this stage:

file:///Users/mohammad/Desktop/PrometricMCQ.com.webarchive Page 216 of 377


PrometricMCQ.com 11/03/2015 10:45

The tumor is less than 3 cm. in size


and requires no chemotherapy

The tumor did not extend beyond


the kidney and was completely
resected

The tumor extended beyond the


kidney but was completely
resected (Correct Answer)

The tumor has spread into the


abdominal cavity and cannot be
resected

Report an issue

When assessing a 12 year old child with


Wilm's tumor, the nurse should keep
in mind that it most important to avoid
which of the following:

Measuring the child's chest


circumference

Palpating the child's abdomen


(Correct Answer)

Placing the child in an upright


position

Measuring the child's occipitofrontal


circumference

Report an issue

file:///Users/mohammad/Desktop/PrometricMCQ.com.webarchive Page 217 of 377


PrometricMCQ.com 11/03/2015 10:45

The nurse is teaching a 45 year old woman


how to increase the potassium in
her diet. The woman says she knows
bananas are high in potassium, but she
doesn't like their taste. What foods should
the nurse recommend the client
include in her diet:

Carrots, broccoli, yogurt

Rhubarb, tofu, celery

Potatoes, spinach, raisins (Correct


Answer)

Onions, corn, oatmeal

Report an issue

When developing a plan of care for a


patient with SIADH (syndrome of
inappropriate antidiuretic hormone
secretion), which interventions will the
nurse include:

Encourage fluids to 2000 ml/day

Long-term fluid restriction (Correct


Answer)

Monitor for increased peripheral


edema

Keep head of bed elevated to 30


degrees
file:///Users/mohammad/Desktop/PrometricMCQ.com.webarchive Page 218 of 377
PrometricMCQ.com 11/03/2015 10:45

Report an issue

A client is admitted for treatment of the


syndrome of inappropriate
antidiuretic hormone (SIADH). Which
nursing intervention is appropriate?

Infusing I.V. fluids rapidly as ordered

Administering glucose-containing
I.V. fluids as ordered

Restricting fluids (Correct Answer)

Encouraging increased oral intake

Report an issue

How streptococci infection is transmitted


from person to another:

Airborne

Contact (Correct Answer)

Slandered

Blood

Report an issue

file:///Users/mohammad/Desktop/PrometricMCQ.com.webarchive Page 219 of 377


PrometricMCQ.com 11/03/2015 10:45

hospitalized client is found to be comatose


and hypoglycemic with a blood
sugar of 50 mg/dL. Which of the following
would the nurse do first?

Infuse 1000 mL D5W over a 12-hour


period

Encourage the client to drink orange


juice with added sugar

Check the client's urine for the


presence of sugar and acetone

Administer 50% glucose


intravenously (Correct Answer)

Report an issue

A patient receives TPN, which of the


following nursing actions is important to
minimize the risk for fluid volume excess:

Increase diuretic dose if swelling


occurs

Limit the amount of free water in


relation to sodium intake

Monitor his or her skin turgor

Weigh the patient daily on the same


scale (Correct Answer)

file:///Users/mohammad/Desktop/PrometricMCQ.com.webarchive Page 220 of 377


PrometricMCQ.com 11/03/2015 10:45

Report an issue

Which of the following findings would most


likely indicate the presence of a
respiratory infection in a client with asthma:

Cough productive of yellow


sputum (Correct Answer)

Bilateral expiratory wheezing

Chest tightness

Respiratory rate of 30 breaths/


minute

Report an issue

When teaching a mother how to administer


eye drops, where should the nurse
tell her to place them?

In the conjunctival sac that is formed


when the lower lid is pulled down
(Correct Answer)

Carefully under the eye lid while it is


gently pulled upward

On the sclera while the child looks


to the side

Anywhere as long as drops contact


the eye's surface

file:///Users/mohammad/Desktop/PrometricMCQ.com.webarchive Page 221 of 377


PrometricMCQ.com 11/03/2015 10:45

Report an issue

One nursing intervention for patient with


asthma is to facilitate removal of
secretions. This can be done by:

Encourage the patient to perform


slow and shallow breathing

Encourage the patient to increase


fluid intake (Correct Answer)

Encourage the patient to


hyperventilate

Encourage the patient to decrease


fluid intake

Report an issue

The nurse is caring for a 6-year-old boy


following revision of
ventriculoperitoneal shunt. An expected
nursing intervention is:

Request for an x-ray to evaluate


shunt placement

Daily measurement of head


circumference (Correct Answer)

Frequent palpation of the fontanels

file:///Users/mohammad/Desktop/PrometricMCQ.com.webarchive Page 222 of 377


PrometricMCQ.com 11/03/2015 10:45

ining the child in a prone position

Report an issue

the patient have an itchy, erythematous,


vesicular, weeping, and crusting
patches on his skin which skin disease is
this considered:

Psoriasis

Impetigo

Eczema (Correct Answer)

Urticaria

Report an issue

Giving an intramuscular injection to an


infant in the Gluteus Maximus muscle
may produce an injury to which of the
following nerves:

Femoral nerve

Sciatic nerve (Correct Answer)

Vagus nerve

Popliteal nerve

file:///Users/mohammad/Desktop/PrometricMCQ.com.webarchive Page 223 of 377


PrometricMCQ.com 11/03/2015 10:45

Report an issue

Which of the following is one of the side


effects of albuterol nebulizer:

Hypertension

Hypotension

Fast irregular heart beats (Correct


Answer)

Tachypnea

Report an issue

A patient, age 49, returns from the


postanesthesia care unit after a total
abdominal hysterectomy and bilateral
salpingo-oophorectomy to treat cervical
cancer. Which nursing intervention has the
highest priority at this time?

Monitor the patient for indications of


hemorrhage (Correct Answer)

Assess the patient’s pain level


and response to analgesics

Encourage the patient to do deep


breathing and leg exercises

Provide emotional support to the


patient

file:///Users/mohammad/Desktop/PrometricMCQ.com.webarchive Page 224 of 377


PrometricMCQ.com 11/03/2015 10:45

Report an issue

When planning the care of a client who has


undergone an abdominal
hysterectomy, which nursing measure is
most helpful for preventing
postoperative complications and
facilitating an early discharge?

Reestablishing oral fluids and


nutrition

Promoting ambulation and


movement (Correct Answer)

Maintaining accurate intake and


output

Exploring feelings about altered


image

Report an issue

Which of the following is considered one of


the common complications
following abdominal total hysterectomy:

DVT (Correct Answer)

Paralytic ileus

Constipation

file:///Users/mohammad/Desktop/PrometricMCQ.com.webarchive Page 225 of 377


PrometricMCQ.com 11/03/2015 10:45

Perforation

Report an issue

A client had a hysterectomy 10 hours ago.


The nurse assesses the client and
finds that her blood pressure has fallen
abruptly. Which action by the nurse is
most appropriate at this time?

Continue to monitor the blood


pressure every 15 minutes

Document the information on the


client’s chart

Inform the surgeon about the clientâ


€™s condition (Correct Answer)

Change the client to a Fowler’s


position

Report an issue

The nurse is teaching a client regarding risk


factors for stroke (CVA). The
greatest risk factor is which of the
following:

Diabetes

Heart disease

Renal insufficiency

file:///Users/mohammad/Desktop/PrometricMCQ.com.webarchive Page 226 of 377


PrometricMCQ.com 11/03/2015 10:45

Hypertension (Correct Answer)

Report an issue

The nurse recognizes that the most


common type of brain attack (CVA) is
related to which of the following:

Ischemia (Correct Answer)

Hemorrhage

Headache

Vomiting

Report an issue

A woman comes to the clinic and states,


"I've been sick for so long! My eyes
have gotten so puffy, and my eyebrows
and hair have become coarse and dry."
The nurse will assess for other signs and
symptoms of:

Cachexia

Parkinson's syndrome

Myxedema (Correct Answer)

Scleroderma

file:///Users/mohammad/Desktop/PrometricMCQ.com.webarchive Page 227 of 377


PrometricMCQ.com 11/03/2015 10:45

Report an issue

A 32 year-old man is brought to the


emergency department by a friend with
whom he had been playing football with
earlier that evening. His symptoms
came on after twenty minutes of the
football match. He appears anxious and
restless. Auscultation of the lung reveals
wheezing on exhalation. The patient
states that he is unable to get a full breath
of air. He had this problem as a
child a couple of times after exercising in
cold weather.
Blood pressure 126/72mmhg
Heart rate 90/min
Respiratory rate 28/min shallow
Temperature 37.0C tympanic
Oxygen saturation 94% on room air
Which position is the best for this patient?

Supine

Reverse Trendelenburg’s

Prone

High fowler’s (Correct Answer)

Report an issue

A 43 years- old man is 30- hour post-


operative following placement of a
partial thickness skin autograft for a burn
injury on the lower anterior leg.
During a routine assessment, the nurse

file:///Users/mohammad/Desktop/PrometricMCQ.com.webarchive Page 228 of 377


PrometricMCQ.com 11/03/2015 10:45

observes the wound is bleeding


continuously.
What is the priority nursing action?

Use a pen to outline and monitor the


area

Perform a wound swab for


laboratory analysis

Incise and drain fluids from the


wound bed

Apply firm and direct pressure for 10


minutes (Correct Answer)

Report an issue

Coarse hair may indicate:

Dehydration (Correct Answer)

Inflammation

Under nutrition

Anemia

Report an issue

The nurse performs a home visit for a 32


years- old woman who had given
birth to her first infant three days before.
The mother has concerns about

file:///Users/mohammad/Desktop/PrometricMCQ.com.webarchive Page 229 of 377


PrometricMCQ.com 11/03/2015 10:45

breastfeeding and the nurse observes the


infant feeding. The mother sits
supported upright with cushions and the
infant positioned in a cradle hold.
The infant’s head and body are aligned
against the mother’s abdomen. The
infant sucks intermittently with the lips
turned outwards and the nipple in the
mouth.
Which intervention is most appropriate?

Reposition the infant

Assist the infant to turn lips inwards

Ensure the latch includes the


areola (Correct Answer)

Stimulate the infant to suck


constantly

Report an issue

A 52 year- old woman is scheduled to


undergo an abdomino- perineal
resection in three days for removal of a
cancer of the rectum. The nurse
reviews the care plan with the patient. The
patient will receive prophylactic
antibiotics and will be given a mechanical
bowel preparation the day before.
Which additional preparation should the
patient undertake at this time?

Wear pressure stockings

Perform leg strengthening exercises

Maintain high- protein, low- residue


diet (Correct Answer)

file:///Users/mohammad/Desktop/PrometricMCQ.com.webarchive Page 230 of 377


PrometricMCQ.com 11/03/2015 10:45

Take daily ferrous iron tablets

Report an issue

A patient is preparing for a scheduled hip


replacement.
Which lab value should be reported to the
physician?

Sodium

Calcium

Potassium (Correct Answer)

Magnesium

Report an issue

A nurse is admitting a six month- old infant


with pneumonia. Which of the
following interventions supports this
infant’s emotional needs?

Allow the parents to leave the room


during painful procedures

Encourage parents to distract the


infant from crying (Correct Answer)

Interview the patents to learn the


infant’s comforting habits

file:///Users/mohammad/Desktop/PrometricMCQ.com.webarchive Page 231 of 377


PrometricMCQ.com 11/03/2015 10:45

Enforce strict visiting schedule and


routines

Report an issue

While caring for a patient in the post-


anesthesia care unit (PACU) who has
developed hypovolemic shock, a nurse
should position the patient:

Flat with legs elevated (Correct


Answer)

In trendelenburg position

With the head of the bed elevated


45 degrees

Completely flat

Report an issue

A five years – old child girl had presented


with tenderness, headache and
petechiae. She was pale and complains of
joint pain. On palpation there was
an enlarged spleen, liver and lymph nodes.
A lumbar puncture showed central
nervous system involvement. The child
underwent chemotherapy treatment
and is now attended by the nurse for
regular routine examinations.
Which condition would require more
attention from the nurse?

file:///Users/mohammad/Desktop/PrometricMCQ.com.webarchive Page 232 of 377


PrometricMCQ.com 11/03/2015 10:45

Increased leukocytes

Lack of muscle coordination

Bleeding while brushing teeth


(Correct Answer)

Occasional nausea and vomiting

Report an issue

During the evaluation at a community


clinic, the patient completes the medical
history. Which of the following is NOT a
risk factor for an acute myocardial
infarction?

Coronary artery disease

Smoking

Hemophilia (Correct Answer)

Hyperlipidemia

Report an issue

A postoperative patient has the nursing


diagnosis of ineffective tissue
perfusion. To assess for tissue perfusion
the nurse should check all of the
following except:

file:///Users/mohammad/Desktop/PrometricMCQ.com.webarchive Page 233 of 377


PrometricMCQ.com 11/03/2015 10:45

Skin and nail bed color

Temperature of extremities
(Correct Answer)

Respiratory rate

Peripheral pulses

Report an issue

A patient visits the clinic for a 2- week


checkup after a corneal transplantation
(keratoplasty). The nurse observes the
patent’s sclera is red and the patient
complains of the eye feeling irritated. The
nurse suspects the patient may
have:

Infection

Hemorrhage

Graft rejection (Correct Answer)

Postoperative glaucoma

Report an issue

The nurse is assessing a patient who just


had surgery under general
anesthesia. The patient’s respiration rate is
4 per minute and the O2
saturation on 3L per minute of oxygen via
nasal cannula is 84%.
The nurse is awaiting the results of an

file:///Users/mohammad/Desktop/PrometricMCQ.com.webarchive Page 234 of 377


PrometricMCQ.com 11/03/2015 10:45

arterial blood gas (ABG) and


anticipates that which of the following will
be ELEVATED?

Arterial oxygen saturation (SaO2)

Hydrogen ion concentration (pH)

Partial pressure of arterial oxygen


(PaO2)

Partial pressure of arterial carbon


dioxide (PaCO2) (Correct Answer)

Report an issue

During the immediate postoperative


period, a patient reveals an oxygen
saturation of 91%. The nurse should:

Position the patient on the left side

Administer supplemental oxygen


(Correct Answer)

Continue to provide supportive care

Lower the temperature of the room

Report an issue

The nurse performs an assessment of a 23


year- old man who believes that

file:///Users/mohammad/Desktop/PrometricMCQ.com.webarchive Page 235 of 377


PrometricMCQ.com 11/03/2015 10:45

people are spying on him. During the


interview, he keeps his eyes to the floor
and answers questions awkwardly. He has
never had an intimate relationship
and avoids contact with his family
members. He has never been employed
and
tells the nurse that he is not looking for a
job. The nurse considers Erickson’s
theory of psychosocial development.
Which stage is this patient most likely
experiencing?

Autonomy versus shame and doubt

Initiative versus guilt

Trust versus mistrust (Correct


Answer)

Identity versus confusion

Report an issue

A 32 year- old woman with a motor neuron


disease has progressive weakness
of cranial nerves V, IX, XII. She has recently
been experiencing sudden
outbursts of crying and laughing. Both
stroke and myasthenia gravis have
been ruled out.
Which voluntary muscle activity would be
most significantly impacted?

Walking

Swallowing (Correct Answer)

Breathing

file:///Users/mohammad/Desktop/PrometricMCQ.com.webarchive Page 236 of 377


PrometricMCQ.com 11/03/2015 10:45

Smelling

Report an issue

Causes of primary hypothyroidism in adults


include:

Malignant or benign thyroid nodules

Surgical removal or failure of the


pituitary gland

Surgical removal or radiation of


thyroid gland

Autoimmune-induced atrophy of the


gland (Correct Answer)

Report an issue

A 72-year-old patient is diagnosed with


hypothyroidism, and levothyroxine
(Synthroid) is prescribed. During initiation
of thyroid replacement for the
patient, it is most important for the nurse to
assess:

Mental status

Nutritional level

Cardiac function (Correct Answer)

file:///Users/mohammad/Desktop/PrometricMCQ.com.webarchive Page 237 of 377


PrometricMCQ.com 11/03/2015 10:45

Fluid balance

Report an issue

A client presents with Hypocalcemia,


hyperphosphatemia, muscle cramps, and
positive Trousseau's sign. What diagnosis
does this support?

Diabetes insipidus

Conn's syndrome

Hypoparathyroidism (Correct
Answer)

Acromegaly

Report an issue

The nurse acknowledges that the first-line


drug for treating of client's blood
pressure might be which drug:

Diuretic (Correct Answer)

Alpha blocker

ACE inhibitor

Alpha/beta blocker

file:///Users/mohammad/Desktop/PrometricMCQ.com.webarchive Page 238 of 377


PrometricMCQ.com 11/03/2015 10:45

Report an issue

The nurse is teaching a client about


clopidogrel (Plavix). What is important
information to include?

Constipation may occur

Hypotension may occur

Bleeding may increase when taken


with aspirin (Correct Answer)

Normal dose is 25 mg tablet per day

Report an issue

The nurse knows that the client's


cholesterol level should be within which
range:

150 to 200 mg/dL (Correct


Answer)

200 to 225 mg/dL

225 to 250 mg/dL

Greater than 250 mg/dL

Report an issue

file:///Users/mohammad/Desktop/PrometricMCQ.com.webarchive Page 239 of 377


PrometricMCQ.com 11/03/2015 10:45

The nurse is performing an assessment in


a client with a suspected diagnosis
of cataract. The chief clinical manifestation
that the nurse would expect to
note in the early stages of cataract
formation is:

Eye pain

Floating spots

Blurred vision (Correct Answer)

Diplopia

Report an issue

A client who has a history of Crohn's


disease is admitted to the hospital with
fever, diarrhea, cramping, abdominal pain,
and weight loss. The nurse should
monitor the client for:

Hyperalbuminemia

Thrombocytopenia

Hypokalemia (Correct Answer)

Hypercalcemia

Report an issue

file:///Users/mohammad/Desktop/PrometricMCQ.com.webarchive Page 240 of 377


PrometricMCQ.com 11/03/2015 10:45

Adult patient complains of diarrhea,


vomiting, abdomen cramp and pain
within the past 2 weeks. The patient
reported that the pain increases when he
eats and relieves when he passes stool.
Which of the following may be the
cause:

Appendicitis

Crohn’s disease (Correct


Answer)

Ectopic pregnant

Cholecystitis

Report an issue

When caring for a 3-day-old neonate who


is receiving phototherapy to treat
jaundice, the nurse in charge would expect
to do which of the following:

Turn the neonate every 6 hours

Encourage the mother to


discontinue breast-feeding

Notify the physician if the skin


becomes bronze in color

Check the vital signs every 2 to 4


hours (Correct Answer)

file:///Users/mohammad/Desktop/PrometricMCQ.com.webarchive Page 241 of 377


PrometricMCQ.com 11/03/2015 10:45

Report an issue

After 3 days of breast-feeding, a


postpartum patient reports nipple
soreness.
To relieve her discomfort, the nurse should
suggest that she:

Apply warm compresses to her


nipples just before feedings

Lubricate her nipples with expressed


milk before feeding (Correct
Answer)

Dry her nipples with a soft towel


after feedings

Apply soap directly to her nipples,


and then rinse

Report an issue

Which of the following interventions would


be helpful to a breastfeeding
mother who is experiencing engorged
breasts:

Applying ice

Applying a breast binder

Teaching how to express her breasts


in a warm shower (Correct

file:///Users/mohammad/Desktop/PrometricMCQ.com.webarchive Page 242 of 377


PrometricMCQ.com 11/03/2015 10:45

Answer)

Administering bromocriptine
(Parlodel)

Report an issue

What type of milk is present in the breasts


7 to 10 days postpartum?

Colostrum

Hind milk

Mature milk

Transitional milk (Correct Answer)

Report an issue

Which of the following is a sign of rejection


after a renal transplantation
operation:

Decrease potassium

Decrease sodium

Decrease creatinine

Decrease urine out put (Correct


Answer)

file:///Users/mohammad/Desktop/PrometricMCQ.com.webarchive Page 243 of 377


PrometricMCQ.com 11/03/2015 10:45

Report an issue

Which of the following is among the signs


of magnesium toxicity:

Cardiac arrhythmia

Loss of deep tendon reflex


(Correct Answer)

Patient confusion and irritability

Respiratory failure

Report an issue

A nurse is caring for a client who has a


potassium level of 5.4 mEq/L. The
nurse should assess the client for:

ECG changes (Correct Answer)

Constipation

Polyuria

Hypotension

Report an issue

file:///Users/mohammad/Desktop/PrometricMCQ.com.webarchive Page 244 of 377


PrometricMCQ.com 11/03/2015 10:45

Moderate to severe hypokalemia in a


patient will cause the nurse to observe:

Muscle spasms and slow


respirations

Muscle weakness and cardiac


dysrhythmias (Correct Answer)

Confusion and irritability

Vomiting and diarrhea

Report an issue

Which of the following electrolytes


decrease would cause PVC (premature
ventricular contraction):

Potassium (Correct Answer)

Sodium

Calcium

Magnesium

Report an issue

Which of the following electrolytes


decrease would cause recurrent vomiting
and general weakness:

file:///Users/mohammad/Desktop/PrometricMCQ.com.webarchive Page 245 of 377


PrometricMCQ.com 11/03/2015 10:45

Potassium (Correct Answer)

Sodium

Calcium

Magnesium

Report an issue

The nurse is educating a couple who has


had difficulty with conception. The
client asks about the hormone, which is
responsible for the production of eggs.
The nurse answers that this hormone is
called:

Melanocyte-stimulating hormone

Luteinizing hormone (LH)

Follicle-stimulating hormone (FSH)


(Correct Answer)

Interstitial cell-stimulating hormone


(ICSH)

Report an issue

The patient has an order to receive 45 mg


of prednisone by mouth daily.
Available are 10-mg tablets. How many
tablets should the nurse prepare to
give?

file:///Users/mohammad/Desktop/PrometricMCQ.com.webarchive Page 246 of 377


PrometricMCQ.com 11/03/2015 10:45

1.6

4.5 (Correct Answer)

Report an issue

The nurse caring for a 54-year-old patient


hospitalized with diabetes mellitus
would look for which of the following
laboratory test results to obtain
information on the patient's past glucose
control:

Prealbumin level

Urine ketone level

Fasting glucose level

Glycosylated hemoglobin level


(Correct Answer)

Report an issue

Which laboratory test is most important for


the nurse to monitor to determine
how effectively the client’s diabetes is
being managed?

file:///Users/mohammad/Desktop/PrometricMCQ.com.webarchive Page 247 of 377


PrometricMCQ.com 11/03/2015 10:45

Fasting blood glucose

Blood chemistry profile

Complete blood count

Glycosylated hemoglobin (HbA1c)


(Correct Answer)

Report an issue

Blood sugar for new employee at company


with sedentary life style and obese
and by result of Fasting blood Sugar 6.9
mmol (abnormal reading ) what is the
best next step to do to estimate his blood
sugar condition:

Draw blood sample for HbA1c


(Correct Answer)

Repeat the test for the next 2 days

Estimate the postprandial sugar


level

Write a recommendation note for not


hiring him

Report an issue

HbA1c as a measurement instrument for


glucose level in the blood, it

file:///Users/mohammad/Desktop/PrometricMCQ.com.webarchive Page 248 of 377


PrometricMCQ.com 11/03/2015 10:45

measures blood glucose level within:

Last 3 months (Correct Answer)

Last 5 months

Last 7 months

Last 1 year

Report an issue

Which laboratory test indicates liver


cirrhosis?

Decreased red blood cell count

Decreased serum acid phosphatase


level

Elevated white blood cell count

Elevated serum aminotransferase


(Correct Answer)

Report an issue

Which statement best describes the


difference between the pain of angina and
the pain of myocardial infarction:

Pain associated with angina is

file:///Users/mohammad/Desktop/PrometricMCQ.com.webarchive Page 249 of 377


PrometricMCQ.com 11/03/2015 10:45

relieved by rest (Correct Answer)

Pain associated with myocardial


infarction is always more severe

Pain associated with angina is


confined to the chest area

Pain associated with myocardial


infarction is referred to the left arm

Report an issue

The nurse is developing a bowel-retraining


plan for a client with multiple
sclerosis. Which measure is likely to be
least helpful to the client?

Limiting fluid intake to 1000mL per


day (Correct Answer)

Providing a high-roughage diet

Elevating the toilet seat for easy


access

Establishing a regular schedule for


toiletingG

Report an issue

Which organism is responsible for the


onset of rheumatic fever?

file:///Users/mohammad/Desktop/PrometricMCQ.com.webarchive Page 250 of 377


PrometricMCQ.com 11/03/2015 10:45

Staphylococcus aureus

Listeriosis

Group A β-hemolytic
streptococcus (Correct Answer)

Epstein-Barr virus

Report an issue

For what reason are patients instructed to


do Kegel exercises when they are
discharged after childbirth:

They help her get back in shape

They prevent her breasts from


becoming engorged

They help strengthen the perineal


muscles (Correct Answer)

They help reduce strain on the back

Report an issue

You are preparing to discharge a patient


who suffered third degree burn,
which of the following instructions is
appropriate to give the patient to
decrease infection:

file:///Users/mohammad/Desktop/PrometricMCQ.com.webarchive Page 251 of 377


PrometricMCQ.com 11/03/2015 10:45

You must take large amount of fluids

You should wash your hands


thoroughly (Correct Answer)

You better increase vitamin intake

You should avoid fatty meals

Report an issue

The most effective procedure to prevent


spared of infection is:

Wearing gloves

Using antiseptic

Sterilization

Hand washing (Correct Answer)

Report an issue

What is the normal pulse range for an


adult?

120 to 160 beats per minute

90 to 140 beats per minute

60 to 100 beats per minute


(Correct Answer)

file:///Users/mohammad/Desktop/PrometricMCQ.com.webarchive Page 252 of 377


PrometricMCQ.com 11/03/2015 10:45

50 to 80 beats per minute

Report an issue

A four year- old girl was playing outside in


a park when she came running to
her mother crying and holding her right,
upper arm. Two hours later they
presented to the emergency department.
There was a swelling over the upper
right arm, with pain and itching at the site
as well as swelling of the oral
mucosa. The child seems anxious.
What is the next most appropriate step in
management?

Administer subcutaneous
epinephrine (Correct Answer)

Maintain a patent airway

`Administer oral diphenhydramine

Prepare intubation equipment

Report an issue

A 65 year- old man presents with a resting


tremor in the right forearm. An
assessment of gait reveals decreased arm
swinging and slight dragging of the
foot on the right side. His body movements
are slow. He has not been sleeping
well at night for the past six months due to
leg pain and says that he feels

file:///Users/mohammad/Desktop/PrometricMCQ.com.webarchive Page 253 of 377


PrometricMCQ.com 11/03/2015 10:45

constantly tired and weak. He reports that


he has not suffered any recent fall
and that the symptoms seem to be slowly
worsening.
Which medication is most likely to be
administered?

Levodopa (Correct Answer)

Haloperidol

Phenytoin

Benzodiazepine

Report an issue

When planning discharge teaching for the


parent of an infant with
bronchiolitis, the nurse should
EMPHASIZE:

Use of supplemental oxygen at night

Frequent hand washing (Correct


Answer)

Sleeping in the supine position

Rice- thickened formula during


night- time feedings

Report an issue

file:///Users/mohammad/Desktop/PrometricMCQ.com.webarchive Page 254 of 377


PrometricMCQ.com 11/03/2015 10:45

A home care nurse visits a diabetic patient


who was started on insulin
injections. Upon examination, the nurse
observes small lumps and dents on
the right upper arm where the patient has
injected insulin.
What is the BEST nursing intervention?

Refer the patient to a dermatologist


for diabetic cellulitis

Instruct the patient to rotate the


sites of injection (Correct Answer)

Refer the patient to an


endocrinologist for better control of
glucose level

Instruct the patient to inject in the


muscular area instead of the
subcutaneous area

Report an issue

A patient has an order for 1000 milliliters


(ml) of intravenous (IV) fluid to
infuse over eight hours. The available IV
tubing has a drip factor of 10 gtts/ml.
Which of the following rates is correct?

125 ml/hour

125 drop/minute

21 drops/minute (Correct Answer)

21 ml/hour

file:///Users/mohammad/Desktop/PrometricMCQ.com.webarchive Page 255 of 377


PrometricMCQ.com 11/03/2015 10:45

Report an issue

When is the correct time for the nurse to


administer the child’s morning dose
of a combination regular and NPH insulin?

30 minutes before breakfast is


served (Correct Answer)

15 minutes before breakfast is


served

30 minutes after breakfast is served

15 minutes after breakfast is served

Report an issue

The client has an order for administration


of 10 units of regular insulin to be
given at 7:00 a.m. The nurse should offer a
snack at:

3:00 p.m.

1:00 p.m.
file:///Users/mohammad/Desktop/PrometricMCQ.com.webarchive Page 256 of 377
PrometricMCQ.com 11/03/2015 10:45

11:00 a.m.

9:00 a.m. (Correct Answer)

Report an issue

The client is being treated with NPH insulin


at 8:00 a.m. The nurse should
offer a snack at:

9:00 a.m.

11:00 a.m.

6:00 p.m.

2:00 p.m. (Correct Answer)

Report an issue

A client with diabetes comes to the


emergency department. The nurse obtains
a blood glucose measurement with a
glucometer and notes that it is 510
mg/dL. The physician orders I.V. insulin.
Which type of insulin can be given
both intravenously and subcutaneously?

Regular (Correct Answer)

NPH

Lente
file:///Users/mohammad/Desktop/PrometricMCQ.com.webarchive Page 257 of 377
PrometricMCQ.com 11/03/2015 10:45

70/30

Report an issue

If a client with type 1 (insulin-dependent)


diabetes mellitus receives 5 units of
NPH insulin every morning at 7 a.m., the
nurse should closely monitor the
client for signs of hypoglycemia at what
time:

3:00 p.m. (Correct Answer)

12:00 a.m.

7:30 a.m.

10:00 p.m.

Report an issue

NPH is an example of which type of insulin:

Short-acting

Intermediate-acting (Correct
Answer)

Rapid-acting

Long-acting

file:///Users/mohammad/Desktop/PrometricMCQ.com.webarchive Page 258 of 377


PrometricMCQ.com 11/03/2015 10:45

Report an issue

A 70 years- old woman presents with


increasing dyspnea on exertion. She feels
breathless and restless while performing
household tasks, such as making the
bed and sweeping the floor. Her previous
medical history includes a
myocardial infarction at 57 years old. She
sleeps with her head elevated on
three pillows. Examination reveals bilateral
basal crackles and cold, damp
skin.
Blood pressure 172/94 mmhg
Heart rate 94/min
Respiratory rate 36/min
Temperature 37.1 C
Oxygen saturation 90% on room air
Which heart chamber most likely failed
first?

Right atrial

Right ventricle

Left ventricle (Correct Answer)

Left atrial

Report an issue

The nurse is preparing a discharge plan for


a 65 year- old man with a new
diagnosis of congestive heart failure. The
discharge orders include furosemide
40 milligrams by mouth twice per day. The
nurse recommends food to reduce

file:///Users/mohammad/Desktop/PrometricMCQ.com.webarchive Page 259 of 377


PrometricMCQ.com 11/03/2015 10:45

unwanted medication side effects.


Which of the following food would be most
appropriate to include in the
teaching plan?

Green and leafy vegetables

Bananas and oranges (Correct


Answer)

Chicken and fish

Whole wheat grains

Report an issue

A child comes into the clinic with several


lesions to the scalp. The round
lesions have dandruff like scaling with hair
loss.
What is the MOST likely diagnosis for this
skin condition?

Impetigo

Ringworm (tinea capitis) (Correct


Answer)

Pediculosis capitis

Scabies

Report an issue

file:///Users/mohammad/Desktop/PrometricMCQ.com.webarchive Page 260 of 377


PrometricMCQ.com 11/03/2015 10:45

A 40 years- old woman suffered an


automobile accident one month ago. The
accident resulted in a C-4 spinal cord
injury. Her preventive care includes
independent daily performance of
coughing and deep breathing.
Which range- of- motion exercise would be
most beneficial for this patient?

Active

Passive (Correct Answer)

Combined

Resistive

Report an issue

A nine year- old girl has a nursing


diagnosis of altered body image related to
changes in appearance secondary to
varicella infection. The child’s body is
covered with a rash and many large,
weeping pustules. The nurse provides
counseling to the mother who is concerned
that the child will develop scarring.
Which intervention is most appropriate?

Keep the skin out of direct sunlight

Apply calamine lotion to skin every


two hours

Wash pustules with soap and keep


dry

Soak in colloidal oatmeal bath three


times daily (Correct Answer)

file:///Users/mohammad/Desktop/PrometricMCQ.com.webarchive Page 261 of 377


PrometricMCQ.com 11/03/2015 10:45

Report an issue

A nurse is assessing a client with possible


Cushing's syndrome. In a client with
Cushing's syndrome, the nurse expects to
find:

Weight gain in arms and legs

Thick, coarse skin

Hypotension

Deposits of adipose tissue in the


trunk and dorsocervical area
(Correct Answer)

Report an issue

Which vitamin deficiency is most likely to


be a long-term consequence of a
full-thickness burn injury?

Vitamin A

Vitamin B

Vitamin C

Vitamin D (Correct Answer)

Report an issue

file:///Users/mohammad/Desktop/PrometricMCQ.com.webarchive Page 262 of 377


PrometricMCQ.com 11/03/2015 10:45

Sildenafil (Viagra) is prescribed to treat a


client with erectile dysfunction. A
nurse reviews the client's medical record
and would question the prescription
if which of the following is noted in the
client's history:

Neuralgia

Insomnia

Use of nitroglycerin (Correct


Answer)

Use of multivitamins

Report an issue

The physician ordered 1 liter of normal


saline infusion for four hours, how
much N/S should be delivered to the
patient in one hour:

300 ml

250 ml (Correct Answer)

200 ml

150 ml

Report an issue

file:///Users/mohammad/Desktop/PrometricMCQ.com.webarchive Page 263 of 377


PrometricMCQ.com 11/03/2015 10:45

Best diagnostic test for suspected


leukemia is:

Bone marrow aspiration (Correct


Answer)

Blood chemistry

CBC

PT AND PTT

Report an issue

The primary nursing goal when caring of a


child with leukemia is to:

Take vital signs

Provide good nutrition

Give analgesics

Prevent infection (Correct Answer)

Report an issue

The oxygen flow rate for the nasal cannula


is considered to be:

file:///Users/mohammad/Desktop/PrometricMCQ.com.webarchive Page 264 of 377


PrometricMCQ.com 11/03/2015 10:45

1-6 liter/m (Correct Answer)

3-8 liter/m

5-10 liter/m

10-15 liter/m

Report an issue

If the client develops a thrombus in one of


the leg veins, which client response
would the nurse expect when eliciting
Homans’ sign?

Sharp, immediate calf pain


(Correct Answer)

Sudden numbness in the foot

Inability to bend the knee when


asked

Tingling throughout the affected leg

Report an issue

When the nurse assesses the client for


Homans’ sign, which technique is most
accurate?

Have the client push each foot

file:///Users/mohammad/Desktop/PrometricMCQ.com.webarchive Page 265 of 377


PrometricMCQ.com 11/03/2015 10:45

against the mattress.

Have the client extend the legs and


flex each foot toward the head
(Correct Answer)

Ask the client to sit up in bed and


point all the toes forward

Ask the client to contract the thigh


muscles

Report an issue

The phase that include action of the


nursing care plan is called:

Implementation (Correct Answer)

Diagnosis

Assessment

Evaluation

Report an issue

After vein ligation and stripping operation,


the patient should:

Wear elastic stocking (Correct


Answer)

file:///Users/mohammad/Desktop/PrometricMCQ.com.webarchive Page 266 of 377


PrometricMCQ.com 11/03/2015 10:45

Do exercises

Rest the leg

Avoid walking

Report an issue

Rabies disease is considered:

Viral disease (Correct Answer)

Bacterial disease

Protozoan disease

Fungal disease

Report an issue

A client receiving IV infusion. The skin


around the IV insertion site is red,
warm to touch and painful. The nurse
should first:

Discontinue the IV (Correct


Answer)

Slow the IV rate for 30 minutes then


reassess the site

Place a cold compression on the


area

file:///Users/mohammad/Desktop/PrometricMCQ.com.webarchive Page 267 of 377


PrometricMCQ.com 11/03/2015 10:45

Place a warm compression on the


area

Report an issue

Clients who have casts applied to an


extremity must be monitored for
complications. The most significant
complication for which the nurse should
assess the client's extremity is:

Warmth

Numbness (Correct Answer)

Skin desquamation

Generalized discomfort

Report an issue

A nurse is assessing the legs of a client


who's 36 weeks pregnant. Which
finding should the nurse expect?

Absent pedal pulses

Bilateral dependent edema


(Correct Answer)

ish capillary refill

Unilateral calf enlargement

file:///Users/mohammad/Desktop/PrometricMCQ.com.webarchive Page 268 of 377


PrometricMCQ.com 11/03/2015 10:45

Report an issue

Which of the following is the best nursing


action to assess pulse in a toddler
patient:

Assess the brachial artery with the


middle finger and index (Correct
Answer)

Assess the Dorsalis pedis artery with


your palm

Assess the carotid artery with the


four fingers

Assess the radial artery using three


finger

Report an issue

One nursing intervention for patient with


productive cough is to facilitate
removal of secretions. This can be done
by:

Encourage the patient to perform


slow and shallow breathing

Encourage the patient to increase


fluid intake (Correct Answer)

Encourage the patient to

file:///Users/mohammad/Desktop/PrometricMCQ.com.webarchive Page 269 of 377


PrometricMCQ.com 11/03/2015 10:45

hyperventilate

Encourage the patient to decrease


fluid intake

Report an issue

A nurse examines a client’s level of


responsiveness. She finds that the patient
opens his eyes to verbal commands,
obeys verbal commands, and is oriented to
time, place, and person. What’s the client’s
Glasgow Coma Scale:

11/15

12/15

13/15

14/15 (Correct Answer)

file:///Users/mohammad/Desktop/PrometricMCQ.com.webarchive Page 270 of 377


PrometricMCQ.com 11/03/2015 10:45

Report an issue

A 73 year old patient looks at you when


you speak to her. When you ask her
the date, she says "blue." You note left-
sided weakness when she grips your
fingers. What is her Glasgow Coma Score?

11/15

12/15

13/15 (Correct Answer)

14/15

Report an issue

A patient during examination opens his


eyes in response to pain, makes no
verbal response, but withdraws from pain.
What is the Glasgow Coma Score
(GCS) for this patient?

7 (Correct Answer)

11

Report an issue

file:///Users/mohammad/Desktop/PrometricMCQ.com.webarchive Page 271 of 377


PrometricMCQ.com 11/03/2015 10:45

A newborn APGAR score at 1 and 5


minutes is 5 and 10, half an hour later the
baby became bluish in color with heart rate
of 140/m, your first action would
be:

Estimate the score again

Shower the baby with warm water

Give oxygen immediately (Correct


Answer)

Ignore the finding because it is


normal

Report an issue

The primary critical observation for Apgar


scoring is the:

Heart rate (Correct Answer)

Respiratory rate

Presence of meconium

Evaluation of the Moro reflex

Report an issue

file:///Users/mohammad/Desktop/PrometricMCQ.com.webarchive Page 272 of 377


PrometricMCQ.com 11/03/2015 10:45

When performing a newborn assessment,


the nurse should measure the vital
signs in the following sequence:

Pulse, respirations, temperature

Temperature, pulse, respirations

Respirations, temperature, pulse

Respirations, pulse, temperature


(Correct Answer)

Report an issue

Within 3 minutes after birth the normal


heart rate of the infant may range
between:

100 and 180

130 and 170

120 and 160 (Correct Answer)

100 and 130

Report an issue

The expected respiratory rate of a neonate


within 3 minutes of birth may be as

file:///Users/mohammad/Desktop/PrometricMCQ.com.webarchive Page 273 of 377


PrometricMCQ.com 11/03/2015 10:45

high as:

50

60 (Correct Answer)

80

100

Report an issue

When performing nursing care for a


neonate after a birth, which intervention
has the highest nursing priority:

Obtain a dextrostix

Give the initial bath

Give the vitamin K injection

Cover the neonates head with a


cap (Correct Answer)

Report an issue

Which of the following instructions are


appropriate for the nurse to give the
patient about the time of taking
omeprazole tablet:

file:///Users/mohammad/Desktop/PrometricMCQ.com.webarchive Page 274 of 377


PrometricMCQ.com 11/03/2015 10:45

Take the tablet during the meal

Take the tablet 2 hours after the


meal

Take the tablet 30-45 minute before


the meal (Correct Answer)

Take the tablet before bed time

Report an issue

Patient obtained second degree burn to his


abdomen and his entire back,
according to rule of nine what is the
percentage of burn of total body surface

area:

9%

18%

27% (Correct Answer)

36%

Report an issue

file:///Users/mohammad/Desktop/PrometricMCQ.com.webarchive Page 275 of 377


PrometricMCQ.com 11/03/2015 10:45

R.T.A. Patient arrived to E.R, on


examination the patient has increased
heart
rate, low blood pressure and decreased
level of consciousness, you must think
that the patient is developing:

Coma

UTI

Shock (Correct Answer)

Respiratory arrest

Report an issue

Which of the following disease has no


vaccinations:

Small pox

Measles

Poliomyelitis

Chicken pox (Correct Answer)

Report an issue

The type of burn in which all the dermis

file:///Users/mohammad/Desktop/PrometricMCQ.com.webarchive Page 276 of 377


PrometricMCQ.com 11/03/2015 10:45

and epidermis, is destroyed and there


is involvement of underlying structures is
called:

Superficial or first degree burn

Partial thickness or second degree


burn

Full-thickness or third degree burn


(Correct Answer)

Fourth degree burn

Report an issue

Which of the following is the drug of choice


for treatment of rheumatic fever:

Amoxicillin

Azithromycin

Cephalexin

Penicillin (Correct Answer)

Report an issue

What disease is associated with a Vitamin


C deficiency?

file:///Users/mohammad/Desktop/PrometricMCQ.com.webarchive Page 277 of 377


PrometricMCQ.com 11/03/2015 10:45

Pellegria

Neural tube defects

Scurvy (Correct Answer)

Pitted edema

Report an issue

A 59 year old woman is diagnosed with


Hemolytic anemia. What foods would
be recommended for her to eat:

Fish oils

Eggs, cheese

Yellow fruits

Grains, nuts (Correct Answer)

Report an issue

The physician orders an intradermal


injection of 5 tuberculin units/0.1 ml of
tuberculin purified derivative. Which needle
is appropriate for this injection:

12 gauge needle

18 gauge needle

22 gauge needle

file:///Users/mohammad/Desktop/PrometricMCQ.com.webarchive Page 278 of 377


PrometricMCQ.com 11/03/2015 10:45

26 gauge needle (Correct Answer)

Report an issue

Which nursing diagnosis takes priority for a


newly diagnosed patient with a
left- sided stroke?

Risk of impaired swallowing related


to absent gag reflex (Correct
Answer)

Risk for impaired skin integrity


related to immobility

Risk for infection related to invasive


line placement

Risk for impaired speech related to


left sided stroke

Report an issue

When caring for a patient with left-sided


homonymous hemianopsia resulting
from a stroke, which intervention should
the nurse include in the plan of care
during the acute period of the stroke?

Apply an eye patch to the left eye

Approach the patient from the left


side

file:///Users/mohammad/Desktop/PrometricMCQ.com.webarchive Page 279 of 377


PrometricMCQ.com 11/03/2015 10:45

Place objects needed for activities


of daily living on the patient's right
side (Correct Answer)

Reassure the patient that the visual


deficit will resolve as the stroke
progresses

Report an issue

A client with a serum glucose level of


618mg/dl is admitted to the facility. He’s
awake and oriented, has hot dry skin, and
has the following vital signs:
Temperature 38.1º C
Heart rate 116 beats/minute
Blood pressure 108/70 mm Hg
Based on these assessment findings,
which nursing diagnosis takes the highest
priority?

Deficient fluid volume related to


osmotic diuresis (Correct Answer)

Decreased cardiac output related to


elevated heart rate

Imbalanced nutrition: less than body


requirements related to insulin
deficiency

Ineffective thermoregulation related


to dehydration

Report an issue

file:///Users/mohammad/Desktop/PrometricMCQ.com.webarchive Page 280 of 377


PrometricMCQ.com 11/03/2015 10:45

A patient has a blood glucose level of


60mg/dl. The patient is to receive 15 g of
carbohydrate if the blood glucose is less
than 70mg/dl.
How many mls of orange juice should the
patient receive?

120 ml (Correct Answer)

90 ml

60 ml

30 ml

Report an issue

A patient has a blood glucose level of


60mg/dl. The patient is to receive 15 g of
carbohydrate if the blood glucose is less
than 70mg/dl.
How many candies should the patient
receive?

file:///Users/mohammad/Desktop/PrometricMCQ.com.webarchive Page 281 of 377


PrometricMCQ.com 11/03/2015 10:45

4 candies

6 candies (Correct Answer)

8 candies

10 candies

Report an issue

A patient is scheduled for a


pneumonectomy in the morning.
Which of the following diagnosis is the
MOST likely indication for this type
of surgery?

Lung carcinoma (Correct Answer)

Pulmonary tuberculosis

Benign pulmonary nodule

Mediastinal shift

file:///Users/mohammad/Desktop/PrometricMCQ.com.webarchive Page 282 of 377


PrometricMCQ.com 11/03/2015 10:45

Report an issue

The nurse is caring for a patient with a


deep vein thrombosis (DVT). The
patient’s heparin sodium infusion has been
discontinued and the patient is
receiving prescribed warfarin sodium
(Coumadin)
The nurse should advise the patient that
which of the following needs to be
continued?

Daily complete blood count (CBC)

Laboratory tests for partial


thromboplastin time (PTT)

Strict Bedrest

Wearing elasticized support


stockings (Correct Answer)

Report an issue

A patient receiving a total parenteral


nutrition through a central line suddenly
has difficulty breathing and is restless.
Chest auscultation reveals a heart
murmur.
Blood pressure 90/60 mmhg
Heart rate 120/min
Respiratory rate 22/min
Temperature 37.1 C
Oxygen saturation 90% on room air
What is the most appropriate initial nursing
action?

file:///Users/mohammad/Desktop/PrometricMCQ.com.webarchive Page 283 of 377


PrometricMCQ.com 11/03/2015 10:45

Notify the physician

Administer 100% oxygen by face


mask

Place patient in left side lateral


decubitus (Correct Answer)

Obtain stat blood glucose


measurement

Report an issue

A 16 year- old boy fractured his right tibia


and fibula during a football match.
Eight weeks later, the fractures were
successfully healed and the cast was
removed.
Which range- of- motion exercises would
be most beneficial?

Active (Correct Answer)

Passive

Combined

Resistive

Report an issue

A 65 year- old woman who was diagnosed


with hypothyroidism at the age of
45 is brought to the clinic by ambulance.

file:///Users/mohammad/Desktop/PrometricMCQ.com.webarchive Page 284 of 377


PrometricMCQ.com 11/03/2015 10:45

On arrival, she had a severely


decreased level of consciousness. Her
breathing is shallow and irregular. The
skin is cool, dry and pale. There is
generalized non- pitting edema of all
extremities and face.
Blood pressure 70/40 mmhg
Heart rate 60/min
Respiratory rate 12/ min
Body temperature 35.5 C tympanic
What is the most appropriate method to re-
warm this patient?

Warmed intravenous isotonic fluids

Place the patient is a warm bath


(Correct Answer)

Apply heat packs to the head and


neck

Cover the patient with hospital


blankets

Report an issue

The physician ordered a blood glucose test


for the neonates the nurse knows
the best site to puncture is usually:

The lateral heel (Correct Answer)

Anterior sole

Finger tip

Anterior scalp

file:///Users/mohammad/Desktop/PrometricMCQ.com.webarchive Page 285 of 377


PrometricMCQ.com 11/03/2015 10:45

Report an issue

The nurse is teaching a patient who was


recently diagnosed with rheumatoid
arthritis affecting the hands.
Which of the following treatment should
the nurse discuss with the patient
at this time?

Transcutaneous electric nerve


stimulation (TENS)

Iontophoresis

Nonsteroidal anti-inflammatory
drugs (NSAIDs) (Correct Answer)

Synovectomy

Report an issue

A patient has come to the clinic for follow-


up one week after being discharged
from the hospital for treatment of a
hypertensive crisis. Blood pressure
stabilized at 124/78 mm Hg. The patient
reports feeling well, has no edema, no
longer has daily headaches. Blood
pressure is 156/90 mm Hg. During
evaluation the patient admits to having
stopped taking medication that had
been ordered because headaches are no
longer present. Unless the symptoms
return, the patient states he will not be
returning to the clinic.
What should the nurse do?

file:///Users/mohammad/Desktop/PrometricMCQ.com.webarchive Page 286 of 377


PrometricMCQ.com 11/03/2015 10:45

Resolve and discontinue the entire


care plan per patient request,
suggest psychology consult

Resolve B and D; continue A and C

Continue entire care plan as written

Add a new problem to the care plan,


non-compliance and interventions to
determine potential reasons.
(Correct Answer)

Report an issue

A 34 year- old quadriplegia patient resides


at home with his wife. In order to
prevent contractures of all extremities, the
community care nurse will instruct
the patient’s wife in performance of:

Active range of motion exercises

Passive range of motion exercises


(Correct Answer)

Active- assistive range of motion


exercises

Resistive range of motion exercises

Report an issue

file:///Users/mohammad/Desktop/PrometricMCQ.com.webarchive Page 287 of 377


PrometricMCQ.com 11/03/2015 10:45

A six years- old patient has presented to


the clinic with fever, malaise, and
anorexia. The patient was treated 2 weeks
ago for a streptococcal infection of
the throat.
The nurse should expect the physician to
order what test?

Electrocardiogram (Correct
Answer)

Jones test

Spinal tap

Heart biopsy

Report an issue

A 40 year- old woman presented with right


hip pain. Palpation of the pelvic
girdle is normal. An X- ray shows bone
deformities, with osteolytic lesions and
bone enlargement. The patient has not
suffered any trauma and has been
generally healthy.
Which serum laboratory analysis would be
most useful?

Prothrombin time

Alkaline phosphatase (Correct


Answer)

Acid phosphatase

Parathyroid hormone

file:///Users/mohammad/Desktop/PrometricMCQ.com.webarchive Page 288 of 377


PrometricMCQ.com 11/03/2015 10:45

Report an issue

A client complains of itching about 20


minutes after he began his blood
infusion, which of the following is the best
nursing action his caring nurse
must take:

Stop the infusion immediately and


call the physician (Correct Answer)

Call the physician immediately and


the stop the infusion

Culminate the client and continue


infusion

Continue the infusion while


monitoring the client closely

Report an issue

A 32 year- old woman with diabetes


mellitus type 1 underwent a
cholecystectomy and is now on day two of
recovery. The patient’s bowel
sounds have returned and she has
resumed a normal diet but has been
finishing less than half of each meal on the
tray. The nurse enters the room to
perform a routine assessment and finds
the patient confused and shaky
Blood pressure 110/60 mmhg
Heart rate 96/min
Respiratory rate 22/min
Temperature 37.0 C oral
What is the most appropriate initial
file:///Users/mohammad/Desktop/PrometricMCQ.com.webarchive Page 289 of 377
PrometricMCQ.com 11/03/2015 10:45

intervention?

Administer glucagon

Notify the physician (Correct


Answer)

Give an insulin injection

Offer a glass of juice

Report an issue

A Cancer patient wants to stop


chemotherapy and change to palliative
care,
this situation indicates which psychological
status:

Depression

Hopelessness (Correct Answer)

Anxiety

Restlessness

Report an issue

What do we call the type of treatment of


terminal illness patients?

Conservative
file:///Users/mohammad/Desktop/PrometricMCQ.com.webarchive Page 290 of 377
PrometricMCQ.com 11/03/2015 10:45

Curative

Palliative (Correct Answer)

Selective

Report an issue

Psychiatric patient appear violent for


himself and others was put in the room
alone during the period of exacerbation,
then patient calm down and informed
the nurse I am ok now let me with others,
but the nurse refused that as a
punishment way. At which underline Label
will nurse accused under the
court:

Abandonment

False imprisonment (Correct


Answer)

Negligence

Duty to act

Report an issue

Which of the following actions is the first


priority of care for a client
exhibiting signs and symptoms of coronary
artery disease?

file:///Users/mohammad/Desktop/PrometricMCQ.com.webarchive Page 291 of 377


PrometricMCQ.com 11/03/2015 10:45

Decrease anxiety

Enhance myocardial oxygenation


(Correct Answer)

Administer sublingual nitroglycerin

Educate the client about his


symptoms

Report an issue

What is the first intervention for a client


experiencing MI?

Administer morphine

Administer oxygen (Correct


Answer)

Administer sublingual nitroglycerin

Obtain an ECG

Report an issue

A patient complains of stable angina


presented with chest pain. The physician
ordered Nitroglycerine tablet sublingual.
The nurse knows that the right dose
to give the patient is:

0.1 mg

file:///Users/mohammad/Desktop/PrometricMCQ.com.webarchive Page 292 of 377


PrometricMCQ.com 11/03/2015 10:45

0.2 mg

0.4 mg (Correct Answer)

0.8 mg

Report an issue

Cancer patient during the shift and he


surrounded by visitor and laughing
loudly before the shift end patient call the
nurse and asked for pain
medication and said that his level of pain
8/10 (sever pain) patient appears not
on pain, what is the best intervention for
this situation:

Give the patient extra dose of his


pain medication

Ask the patient about the last time


he had his medication

Ignore the patient as you are going


to leave

Check the patient’s file for


history of drug abuse (Correct
Answer)

Report an issue

A patient with pain was prescribed


Ibuprofen 300 mg q 4 hours PRN and
morphine 5mg q 4 hours PRN. After 2

file:///Users/mohammad/Desktop/PrometricMCQ.com.webarchive Page 293 of 377


PrometricMCQ.com 11/03/2015 10:45

hours of receiving Ibuprofen the


patient still complaining of pain, what is the
best intervention:

Wait 2 hours and give a second


dose of ibuprofen

Give the patient his dose of


morphine (Correct Answer)

Wait 1 hour and give the patient his


morphine

Ask the patient to bear the pain

Report an issue

Which instruction should a nurse give the


parents of an infant undergoing
cleft lip repair?

Offer the pacifier as needed

Lay the infant on his abdomen for


sleep

Sit the infant up for each feeding


(Correct Answer)

Loosen arm restraints every hour

Report an issue

Patient with abdominal incision, seven


file:///Users/mohammad/Desktop/PrometricMCQ.com.webarchive Page 294 of 377
PrometricMCQ.com 11/03/2015 10:45

days postoperatively the patient has


pain at the site of incision which increased
by walking and the incision appear
red and there’s pus surrounding the
incision site. This may indicate which of
the following:

Normal finding

Inflammation of the incision


(Correct Answer)

Under nutrition

Good indicator

Report an issue

A preschooler is admitted to the hospital


the day before scheduled surgery.
This is the child’s first hospitalization.
Which action will best help reduce the
child’s anxiety about the upcoming
surgery:

n preoperative teaching immediately

Describe preoperative and


postoperative procedures in detail

Give the child dolls and medical


equipment to play out the
experience (Correct Answer)

Explain that the child will be put to


sleep during surgery and won’t
feel anything

file:///Users/mohammad/Desktop/PrometricMCQ.com.webarchive Page 295 of 377


PrometricMCQ.com 11/03/2015 10:45

Report an issue

Which teaching aid provided by the nurse


is developmentally appropriate for
a preschooler who is about to have a bone
marrow puncture:

Dolls or puppets (Correct Answer)

Pamphlets or booklets

Colored diagrams

Commercial videotapes

Report an issue

A nurse is caring for an infant with spina


bifida. Which assessment findings
suggest hydrocephalus?

Depressed fontanels and suture


lines

Deep-set eyes, which appear to look


upward only

Rapid increase in head size and


irritability (Correct Answer)

Motor and sensory dysfunction in


the foot and leg

file:///Users/mohammad/Desktop/PrometricMCQ.com.webarchive Page 296 of 377


PrometricMCQ.com 11/03/2015 10:45

Report an issue

Parents of a newborn with a unilateral cleft


lip are concerned about having
the defect repaired. The nurse explains that
a child with a cleft lip usually
undergoes surgical repair:

Immediately after birth

By 3 months of age (Correct


Answer)

After 12 months of age

Varies in every case

Report an issue

The hepatitis B vaccine series should begin


at what age:

Newborn (Correct Answer)

2 months

6 months

12 months

Report an issue

file:///Users/mohammad/Desktop/PrometricMCQ.com.webarchive Page 297 of 377


PrometricMCQ.com 11/03/2015 10:45

A school age client admitted to the hospital


because of decreased urine output
and per-orbital edema and diagnosed with
glomerulonephritis. Which of the
following interventions would receive the
highest priority:

Monitoring the vital signs every 4


hours

Monitoring intake and output every


12 hours

Monitoring the client weight daily


(Correct Answer)

Monitoring serum electrolyte daily

Report an issue

The nurse is caring for a client with renal


failure who is being treated with
peritoneal dialysis. Which assessment
before and after peritoneal dialysis is
most valuable in evaluating the outcome of
treatment:

Pulse rate

Body weight (Correct Answer)

Abdominal girth

Urine output

file:///Users/mohammad/Desktop/PrometricMCQ.com.webarchive Page 298 of 377


PrometricMCQ.com 11/03/2015 10:45

Report an issue

Which finding provides the best evidence


that peritoneal dialysis is achieving a
therapeutic effect:

Urine output increases

Appetite improves

Potassium level falls (Correct


Answer)

Red blood cell count is lower

Report an issue

When the nurse is advising the client about


the potential complications
associated with peritoneal dialysis, which
complication is most important to
include:

Pulmonary edema

Abdominal peritonitis (Correct


Answer)

Abdominal hernia

Ruptured aorta

Report an issue

file:///Users/mohammad/Desktop/PrometricMCQ.com.webarchive Page 299 of 377


PrometricMCQ.com 11/03/2015 10:45

A client receiving hemodialysis treatment


arrives at the hospital. He
complains of shortness of breath, and
pedal edema is noted. His last
hemodialysis treatment was yesterday.
Blood pressure 200/100
Heart rate 110/min
Respiratory rate 36/m
Oxygen saturation 89% on room air
Which of the following interventions should
be done first?

Administer oxygen (Correct


Answer)

Elevate the foot of the bed

Restrict the client’s fluids

Prepare the client for hemodialysis

Report an issue

Changes in personality and judgment are


often associated with a lesion in
which of the following:

Frontal lobe (Correct Answer)

Parietal lobe

Broca's area

Wernicke's area

file:///Users/mohammad/Desktop/PrometricMCQ.com.webarchive Page 300 of 377


PrometricMCQ.com 11/03/2015 10:45

Report an issue

Anterior lobe of the brain is responsible for


which of the following:

Personality (Correct Answer)

Movement

Speech

Memory

Report an issue

A history of smoking, abnormal permanent


enlargement of the alveoli, cough,
and dyspnea suggest:

Asthma

Emphysema (Correct Answer)

Chronic bronchitis

Obstructive sleep apnea

Report an issue

file:///Users/mohammad/Desktop/PrometricMCQ.com.webarchive Page 301 of 377


PrometricMCQ.com 11/03/2015 10:45

Which of the following is the end of the first


stage of labor:

Cervix dilated to 10 cm (Correct


Answer)

Crowning of the presenting part

Increased bloody show

Contractions lasting up to 60
seconds

Report an issue

During the latent phase of the first stage of


labor, how often should the nurse
plan to assess the fetal heart rate?

Every 5 to 15 minutes

Every 15 to 30 minutes

Every 30 to 60 minutes (Correct


Answer)

Every 60 to 90 minutes

Report an issue

The nurse explains that oxytocin (Pitocin) is


given after delivery of the baby
and placenta for which purpose:

file:///Users/mohammad/Desktop/PrometricMCQ.com.webarchive Page 302 of 377


PrometricMCQ.com 11/03/2015 10:45

To increase the blood pressure

To prevent the uterus from inverting

To decrease the likelihood of


hemorrhage (Correct Answer)

To prevent rupture of the uterus

Report an issue

A nurse in the delivery room is assisting


with the delivery of a newborn infant.
After the delivery of the newborn, the nurse
assists in delivering the placenta.
Which observation would indicate that the
placenta has separated from the
uterine wall and is ready for delivery:

The umbilical cord shortens in


length and changes in color

A soft and boggy uterus

Maternal complaints of severe


uterine cramping

A sudden gush of dark blood from


the introitus (Correct Answer)

Report an issue

Which vitamin deficiency increase the risk

file:///Users/mohammad/Desktop/PrometricMCQ.com.webarchive Page 303 of 377


PrometricMCQ.com 11/03/2015 10:45

of neural tube defects:

Folic acid (Correct Answer)

Vitamin C

Vitamin B12

Calcium

Report an issue

A client has been diagnosed with folic acid


deficiency. The nurse’s discharge
teaching will focus on foods high in folic
acid. Which of the following foods has
the highest folic acid level:

Citrus fruits

Raisins

Brewer’s yeast

Eggs (Correct Answer)

Report an issue

Which of the following is the recommended


daily dose of folic acid during
pregnancy:

200 mcg

file:///Users/mohammad/Desktop/PrometricMCQ.com.webarchive Page 304 of 377


PrometricMCQ.com 11/03/2015 10:45

400 mcg (Correct Answer)

800 mcg

1000 mcg

Report an issue

Lochia normally disappears after how


many days postpartum:

5 days

7-10 days (Correct Answer)

18-21 days

28-30 days

Report an issue

If the maintenance requirement of fluid for


a child is 900 ml/ day, each daily
feeding account 120 ml, what is the
number of feeding is required to achieve
this maintenance:

5 feedings

6 feedings

7 feedings

file:///Users/mohammad/Desktop/PrometricMCQ.com.webarchive Page 305 of 377


PrometricMCQ.com 11/03/2015 10:45

8 feedings (Correct Answer)

Report an issue

A 12 year- old patient had a cast removed


from the left leg after wearing if for
eight weeks. The patient wants to resume
sports as soon as possible. In order
to regain muscle strength lost while
wearing cast, the nurse will instruct the
patient in performance of:

Resistive range of motion exercises


to left leg

Passive range of motion exercises to


right leg

Active- assistive range of motion


exercises to the right leg

Active range of motion exercises to


both legs (Correct Answer)

Report an issue

A 31 years- old woman with diabetes type


1 presents to the clinic with fatigue,
blurred vision, and loss of appetite. Her
breath smells like fruit and she leaves
the room twice during the examination to
use the toilet. She has brought a
little bottle of water with her that she
finishes while at the clinic. She reports
that she has had a cold for the past three
days, but has not taken additional

file:///Users/mohammad/Desktop/PrometricMCQ.com.webarchive Page 306 of 377


PrometricMCQ.com 11/03/2015 10:45

insulin during the illness


Blood pressure 130/70 mmhg
Heart rate 90/min
Respiratory rate 20/min
Body temperature 38.0 C oral
What is the most appropriate nursing
diagnosis

Risk for impaired skin integrity


related to circulation

Deficient knowledge related to


illness management (Correct
Answer)

Risk for fluid volume excess related


to fluid intake

Imbalanced nutrition related to


decreased appetite

Report an issue

Adult patient suffers from hoarseness of


voice, and difficulty in speech may be
suffering from an injury of which of the
following cranial nerves:

IX

X (Correct Answer)

XI

XII

Report an issue

file:///Users/mohammad/Desktop/PrometricMCQ.com.webarchive Page 307 of 377


PrometricMCQ.com 11/03/2015 10:45

The parent of a child with chronic asthma


is hesitant to discipline because the
child often doesn’t feel well. The nurse
should encourage the parent to:

Set consistent behavior limits


(Correct Answer)

Be more lenient during times of


illness

Cherish the limited time the child


has to live

Avoid upsetting the child with limit-


setting

Report an issue

A 55 year- old woman presented with pain


in the joints of the hands. She feels
generally tired and has had a low grade
fever for the past week. On
examination, the joints of the fingers on
both hands are symmetrically swollen.
Blood pressure 120/70 mmhg
Heart rate 82/min
Respiratory rate 18/min
Temperature 38.0 C
Oxygen saturation 99% on room air
Which type of arthritis is most likely?

Rheumatoid (Correct Answer)

Septic

file:///Users/mohammad/Desktop/PrometricMCQ.com.webarchive Page 308 of 377


PrometricMCQ.com 11/03/2015 10:45

Psoriatic

Osteoarthritis

Report an issue

A patient presented to the office for a


physical examination. The patient is
found to be healthy and fit but occasionally
drinks alcohol and has
unprotected sex.
What is the BEST nursing diagnosis?

Health- seeking behavior

Knowledge deficit, high- risk


behaviors (Correct Answer)

Low self- esteem

Altered thought process

Report an issue

A seven year- old child presented to the


emergency room with a fracture of the
right arm.
What would be the long- term goal for this
patient?

Patient verbalizes decreased pain


level

file:///Users/mohammad/Desktop/PrometricMCQ.com.webarchive Page 309 of 377


PrometricMCQ.com 11/03/2015 10:45

Nurse notes neurovascular checks


are within normal limits

Nurse notes decreased swelling of


the right arm

Patient performs activities of daily


living without difficulty (Correct
Answer)

Report an issue

The nurse is caring for a woman whose


husband beats her regularly. Which is
the most important long-term goal for this
woman?

Provide a long-term support group

Help her feel like a survivor


(Correct Answer)

Point out the ways she behaved

Be able to blame the abuser

Report an issue

In what position should a dyspneic patient


be placed?

Prone

Recumbent
file:///Users/mohammad/Desktop/PrometricMCQ.com.webarchive Page 310 of 377
PrometricMCQ.com 11/03/2015 10:45

Semi-fowler’s (Correct
Answer)

Trendelenburg

Report an issue

A nurse is admitting a two year- old child


with an umbilical hernia.
Which of the following interventions does
NOT meet the child’s
developmental needs?

Allowing the child to make choices


when possible

Providing rooming in the unlimited


visitation

Attempting to continue rituals used


at home

Maintaining strict bedrest (Correct


Answer)

Report an issue

A six year- old boy is in the postoperative


care unit following a tonsillectomy.
The nurse observes that his gag reflex has
returned and removes the artificial
airway. The patient then begins to cry and
tells the nurse that his throat hurts
badly. What type of data is the patient
providing the nurse?
file:///Users/mohammad/Desktop/PrometricMCQ.com.webarchive Page 311 of 377
PrometricMCQ.com 11/03/2015 10:45

Objective

Inferential

Comparative

Subjective (Correct Answer)

Report an issue

A first time mother of a three week- old


breastfed baby brings the infant to the
clinic and complains that her child has
been forcefully vomiting after feeding.
He was born 40 week’s gestation.
Weighing 3.6 kilograms. He is constantly
hungry and irritable. Examination reveals a
swollen abdomen and a palpable
mass in the middle upper right quadrant.
What is the most likely health problem?

Intussusception

Pyloric stenosis (Correct Answer)

Gastroeosophageal reflux

Diaphragmatic hernia

Report an issue

file:///Users/mohammad/Desktop/PrometricMCQ.com.webarchive Page 312 of 377


PrometricMCQ.com 11/03/2015 10:45

A home care nurse reviews the second


follow- up laboratory test results of a
patient with iron deficiency anemia. The
nurse compares the recent laboratory
results to the results taken 3 months ago.
The patient has been eating food rich
in iron as prescribed by the nutritionist.
What can be inferred about the progress of
the treatment by comparing the
two laboratory values?
Laboratory values three months prior
Test Result Normal value
Albumin 35 34-56 g/L
Glucose 5.3 3.5-6.5 mmol/L
HCT 0.32 0.41-0.50
Hb 121 120-140 g/L
Blood urea nitrogen 6.1 2.8 to 8.9 mmol/L
Creatinine 60 58-145 μmol/L
Current laboratory values
Test Result Normal value
Albumin 33 34-56 g/L
Glucose 5.7 3.5-6.5 mmol/L
HCT 0.41 0.41-0.50
Hb 139 120-140 g/L
Blood urea nitrogen 5.7 2.8 to 8.9 mmol/L
Creatinine 60 58-145 μmol/L

Patient’s uptake of iron by diet


is sufficient, and no additional
intervention is necessary

Patient’s uptake of iron by diet


is sufficient, but additional
intervention is necessary (Correct
Answer)

Patient’s uptake of iron by diet


is insufficient, and additional
intervention is necessary

Patient’s uptake of iron by diet


is insufficient, and the physicianâ
€™s immediate action is necessary

Report an issue

file:///Users/mohammad/Desktop/PrometricMCQ.com.webarchive Page 313 of 377


PrometricMCQ.com 11/03/2015 10:45

Which assessment finding would the nurse


expect in an infant diagnosed with
pyloric stenosis?

Abdominal rigidity

Ribbon-like stools

Visible waves of peristalsis


(Correct Answer)

Rectal prolapse

Report an issue

A six year-old child with type 1 diabetes


has an uncontrolled blood glucose
level. The child has been given regular
insulin with minimal change in glucose
level in the first 30 minutes.
Which type of insulin has an onset of 15
minutes and a duration of no more
than 4 hours?

Aspart (Novolog)

Lispro (Humalog) (Correct Answer)

Semilete

Neutral protamine hagedom (NPH)

Report an issue

file:///Users/mohammad/Desktop/PrometricMCQ.com.webarchive Page 314 of 377


PrometricMCQ.com 11/03/2015 10:45

A patient with cerebrovascular accident,


left-sided hemiplegia, and aphasia
has nursing diagnosis of risk for aspiration
related to swallowing difficulties
with a short-term goal that the patient will
not aspirate. The patient has
undergone insertion of a percutaneous
gastric endoscopy and has all nutrition
and fluid administered through the tube.
The nurse should:

Continue the care plan as written

Discontinue the risk for aspiration


diagnosis

Revise the goals and interventions


for the diagnosis (Correct Answer)

Add a nursing diagnosis of


ineffective health maintenance

Report an issue

A nurse is evaluating a patient 5 days after


a right total hip replacement.
Which of the following goals is appropriate
for the patient?

Maintain abduction without


dislocation (Correct Answer)

Rest with legs elevated while sitting

Tie shoes and put on undergarments

file:///Users/mohammad/Desktop/PrometricMCQ.com.webarchive Page 315 of 377


PrometricMCQ.com 11/03/2015 10:45

without assistive devices

Perform scissor-like exercises daily

Report an issue

A three year- old child was admitted to the


postoperative care unit following a
heart transplant. The nurse administers
cyclosporine by intravenous infusion.
Fifteen minutes later the child has difficulty
breathing, his skin feels cold and
clammy and he appears restless. Which is
the most appropriate initial nursing
action?

Ensure airway patency

Administer oxygen therapy

Discontinue intravenous infusion


(Correct Answer)

Administer intramuscular
epinephrine

Report an issue

A 40 year-old woman is a gravida2, para 2


and is currently trying to conceive.
Her previous pregnancy resulted in the
birth of a baby with cleft lip and
palate. The patient is anxious and
concerned about future pregnancies and
the
nurse provides genetic counseling and
file:///Users/mohammad/Desktop/PrometricMCQ.com.webarchive Page 316 of 377
PrometricMCQ.com 11/03/2015 10:45

reassurance.
Which food would most effectively prevent
recurrence?

Green vegetables and citrus fruit


(Correct Answer)

Eggs, milk and dairy products

Wheat, corn, rice, oats and rye

Beef, chicken and yellow vegetables

Report an issue

A 50 year-old woman presented with poor


balance and coordination. She says
that she has developed pain on the outer
aspect of the thighs and the inner side
of the arch of the foot. The nurse wishes to
examine the integrity of the
affected lumbar spinal nerve root and
performs a deep tendon reflex
examination.
Which reflexes would most likely be
diminished?

Planter

Brachioradialis

Patellar (Correct Answer)

Achilles tendon

Report an issue

file:///Users/mohammad/Desktop/PrometricMCQ.com.webarchive Page 317 of 377


PrometricMCQ.com 11/03/2015 10:45

When administrating an intramuscular


injection to an infant, which of the
following sites is appropriate for the nurse
to use?

Rectus femoris (Correct Answer)

Deltoid

Dorsogluteal

Ventrogluteal

Report an issue

A 66 years-old woman is admitted to the


hospital with a history of
hypertension. She presented with
breathing difficulties that worsen with
activity and while sleeping. She is generally
weak and feels that her heart
misses beats and that it sometimes beats
loudly. An electrocardiogram shows
atrial fibrillation, right ventricular
hypertrophy and deviation toward the
right.
Which nursing intervention is most
appropriate for this patient?

Provide a bedside commode


(portable toilet) (Correct Answer)

Place in right side-lying positio

Encourage family and friends to visit

file:///Users/mohammad/Desktop/PrometricMCQ.com.webarchive Page 318 of 377


PrometricMCQ.com 11/03/2015 10:45

Encourage independent hygienic


activities

Report an issue

Oral iron supplements are prescribed for a


6-year-old child with iron
deficiency anemia. The nurse instructs the
mother to administer the iron with
which of the following food item to
enhance absorption of iron:

Milk

Water

Apple juice

Orange juice (Correct Answer)

Report an issue

9 months old infant was brought to the


hospital by his parents because he is
crying most of the time. The examination
revealed that he is suffering from
otitis media. Which of the following
instructions you would include in the care
plan you suggest to the parents:

Give antibiotic as prescribed


(Correct Answer)

Try to calm down your baby

file:///Users/mohammad/Desktop/PrometricMCQ.com.webarchive Page 319 of 377


PrometricMCQ.com 11/03/2015 10:45

Wash the ears regularly

Ignore the child’s pain

Report an issue

A child with pneumonia was prescribed


penicillin injection. Before giving the
injection to the child the nurse performed
skin allergy test which showed that
the child has penicillin allergy. Which of the
following is the next step the
nurse should take:

Stop the treatment and inform the


doctor (Correct Answer)

Administer penicillin intramuscular

Postpone the does until the child is


well

Ignore the test result and give the


injection

Report an issue

The following table represents the blood


pressure value for a patient in 3
successive days. Regarding the
information given in the tablet, this patient
is
considered:

file:///Users/mohammad/Desktop/PrometricMCQ.com.webarchive Page 320 of 377


PrometricMCQ.com 11/03/2015 10:45

Normal blood pressure

Pre-hypertension (Correct Answer)

Hypertensive

Hypotensive

Report an issue

A patient with pneumonia with excessive


mucous in the left lung, which of the
following is the best position to facilitate
drainage of the mucous from the
lower left lung:

On the right side with trendelenburg


position (Correct Answer)

On the left side with elevation of


HOB

On the right side with elevation of


the HOB

On the left side with trendelenburg


position

Report an issue

A patient complains of left eye redness and


itching, the doctor told you to put
atropine eye drops for the patient to
examine his eye. The nurse should instill
the eye drops into:

file:///Users/mohammad/Desktop/PrometricMCQ.com.webarchive Page 321 of 377


PrometricMCQ.com 11/03/2015 10:45

The left eye

The right eye

Both right and left eyes (Correct


Answer)

Neither of the eyes

Report an issue

The district nurse visits a 30 year-old


woman at home following the delivery of
her second child, a full term girl. Following
the delivery of her first child, she
had developed a breast infection and
stopped breastfeeding because of the
pain. She asks the nurse how she can best
prevent it with this infant.
What is the most appropriate response?

Provide feeding on demand

Apply vitamin E cream to the nipples

Request a prophylactic antibiotic

Apply heat to the breasts after


feeding (Correct Answer)

Report an issue

In planning home care for an

file:///Users/mohammad/Desktop/PrometricMCQ.com.webarchive Page 322 of 377


PrometricMCQ.com 11/03/2015 10:45

immunocompromised child, the nurse


instructs
the parents to use cream or emollients to
prevent or manage dry and cracked
skin. A parent will BEST demonstrate
understanding of the rationale for this
be stating:

Creams will prevent breaks in the


skin and decrease the chance of
infection (Correct Answer)

Pleasantly scented creams will mask


other less pleasant smells

Micronutrients in the creams will


help prevent malnutrition

Creams will help prevent


dehydration when my child does not
drink enough

Report an issue

The nurse is teaching a patient about


spironolactone (Aldactone).
Which of the following instructions should
review with the patient?

Increasing the intake of foods that


are high in potassium

Taking the medication right before


going to sleep

Avoid seasonings that are labeled as


salt substitutes (Correct Answer)

Scheduling the medication so that a

file:///Users/mohammad/Desktop/PrometricMCQ.com.webarchive Page 323 of 377


PrometricMCQ.com 11/03/2015 10:45

multivitamin is taken an hour later

Report an issue

A 49 year-old man with a diagnosis of


alcoholic cirrhosis and ascites is
discharged from the hospital with a priority
nursing diagnosis of altered
nutrition, less than body requirements.
Which dietary plan is most appropriate?

Low-protein, low sodium

Low sodium, high- protein (Correct


Answer)

High- protein, low potassium

Low potassium, high protein

Report an issue

A patient who is 4 days postoperatively


after a total hip replacement surgery,
is obese and has not been able to
ambulate since the surgery. The patient is
now diaphoretic, has chills, and complains
of pain in the thigh. The MOST
likely cause is:

Wound infection (Correct Answer)

Deep vein thrombosis (DVT)

file:///Users/mohammad/Desktop/PrometricMCQ.com.webarchive Page 324 of 377


PrometricMCQ.com 11/03/2015 10:45

Pulmonary edema

Dehydration

Report an issue

Which statement by the patient with


hyperlipidemia shows a basic
understanding of the disease and its
treatment?

Exercises has no effect on


cholesterol levels

Hyperlipidemia is usually
asymptomatic until significant target
organ damage is done (Correct
Answer)

HDL cholesterol level of greater than


60mg/dL increases the chance of
coronary artery disease

Cholestyramine (questran) should be


taken in the morning with other
medications

Report an issue

Which of the following describe the correct


sequence of nursing process:

Assessment, planning, diagnosis,

file:///Users/mohammad/Desktop/PrometricMCQ.com.webarchive Page 325 of 377


PrometricMCQ.com 11/03/2015 10:45

implementation and evaluation


(Correct Answer)

Planning, assessment, diagnosis,


implementation and evaluation

Planning, diagnosis, implementation,


assessment and evaluation

Assessment, implementation,
evaluation, planning and diagnosis

Report an issue

A 14 year-old boy presented to the


emergency department. He complains of
having progressively worsening stomach
pain for the past eight hours. On
assessment, the pain is localized in the
lower right quadrant. He rates it as a
level ten on the one-ten pain scale.
Abdominal palpation shows rebound
tenderness in the lower right quadrant, and
positive McBurney and psoas
signs.
Blood pressure 134-78 mmhg
Heart rate 88/min
Respiratory rate 24/min
Temperature 38.2 C
Test result normal values
WBC 12.4 4-10.5 x 109
Which intervention would be most
appropriate to alleviate this patient’s
pain?

Apply warm packs to affected area

Maintain semi-fowler’s with legs


up

Withhold solids and liquids

file:///Users/mohammad/Desktop/PrometricMCQ.com.webarchive Page 326 of 377


PrometricMCQ.com 11/03/2015 10:45

Administer analgesics (Correct


Answer)

Report an issue

Female patient with a diagnosis of atrial


fibrillation, the physician ordered
you to give her digoxin IV, but the patient
refused to be injected and told that
she feel she is going to vomit, what you
should do:

Change the IV digoxin to oral form

Force the patient to accept the


medication

Inform the doctor about the


situation (Correct Answer)

Don’t give any medications to


this patient

Report an issue

40 years old client complaining from


abdominal pain, laboratory result for
stool analysis showed that there’s occult
blood in the stool. Which of the
following procedure would help for
diagnosis of this patient:

Abdominal ultrasound

file:///Users/mohammad/Desktop/PrometricMCQ.com.webarchive Page 327 of 377


PrometricMCQ.com 11/03/2015 10:45

Abdominal x ray

Colonoscopy (Correct Answer)

MRI

Report an issue

A 7-year-old client is brought to the E.R.


He’s tachypneic and afebrile and has
a respiratory rate of 36 breaths/minute and
a nonproductive cough. He
recently had a cold. From his history, the
client may have which of the
following:

Acute asthma (Correct Answer)

Bronchial pneumonia

Chronic obstructive pulmonary


disease (COPD)

Emphysema

Report an issue

Which of the following is a priority goal for


the client with COPD:

Maintaining functional ability


(Correct Answer)

file:///Users/mohammad/Desktop/PrometricMCQ.com.webarchive Page 328 of 377


PrometricMCQ.com 11/03/2015 10:45

Minimizing chest pain

Increasing carbon dioxide levels in


the blood

Treating infectious agents

Report an issue

A 34-year-old woman with a history of


asthma is admitted to the emergency
department. The nurse notes that the client
is dyspneic, with a respiratory rate
of 35 breaths/minute, nasal flaring, and use
of accessory muscles. Auscultation
of the lung fields reveals greatly diminished
breath sounds. Based on these
findings, what action should the nurse take
to initiate care of the client:

Initiate oxygen therapy and reassess


the client in 10 minutes.

Draw blood for an ABG analysis and


send the client for a chest x-ray.

Encourage the client to relax and


breathe slowly through the mouth

Administer bronchodilators
(Correct Answer)

Report an issue

Which of the following is the best breathing

file:///Users/mohammad/Desktop/PrometricMCQ.com.webarchive Page 329 of 377


PrometricMCQ.com 11/03/2015 10:45

pattern you should teach a client


with COPD:

Pursed-lip breathing (Correct


Answer)

Deep breathing

Abdominal breathing

Slow lite breathing

Report an issue

A nurse instructs a female client to use the


pursed-lip method of breathing and
the client asks the nurse about the purpose
of this type of breathing. The nurse
responds, knowing that the primary
purpose of pursed-lip breathing is to:

Promote oxygen intake

Strengthen the diaphragm

Strengthen the intercostal muscles

Promote carbon dioxide


elimination (Correct Answer)

Report an issue

An infant with tetralogy of Fallot is

file:///Users/mohammad/Desktop/PrometricMCQ.com.webarchive Page 330 of 377


PrometricMCQ.com 11/03/2015 10:45

experiencing an attack involving cyanosis


and dyspnea. Which position should the
infant be placed in:

Fowler's

Knee-chest (Correct Answer)

Trendelenburg's

Prone

Report an issue

A client is unresponsive and has been


brought to the emergency department.
Initial laboratory results reveal:
Serum K+ 3.6 mmol/L
Glucose 26 mg/dl
Hemoglobin 12. 6 gm/dl
Carbon dioxide 26. 2 mmol/L. The nurse
will anticipate:

Dextrose by mouth

50% dextrose IV (Correct Answer)

1 unit of packed cells

10 mEq of KCl over 0.5 hour

Report an issue

When a client has a nephrostomy tube, the


file:///Users/mohammad/Desktop/PrometricMCQ.com.webarchive Page 331 of 377
PrometricMCQ.com 11/03/2015 10:45

priority nursing care is to:

Ensure drainage of urine (Correct


Answer)

“Milkâ€​the tube every 2 hours

Keep an accurate record of intake


and output

Instill 2 ml of normal saline solution


every shift

Report an issue

Which of the following is the most common


symptom of myocardial infarction
(MI):

Chest pain (Correct Answer)

Dyspnea

Edema

Palpitations

Report an issue

A client has surgery for a perforated


appendix with localized peritonitis. In
which position should the nurse place the
client:

file:///Users/mohammad/Desktop/PrometricMCQ.com.webarchive Page 332 of 377


PrometricMCQ.com 11/03/2015 10:45

Sims position

Trendelenburg

Semi-fowlers (Correct Answer)

Dorsal recumbent

Report an issue

Which of the following position should the


client with appendicitis assume to
relieve pain:

Prone

Sitting

Supine

Lying with legs drawn up (Correct


Answer)

Report an issue

A women breast feed her infant for one or


two hours and her infant cries most
of the time and she feels pain in her breast,
which of the following instructions
are appropriate for the nurse to give the
mother

file:///Users/mohammad/Desktop/PrometricMCQ.com.webarchive Page 333 of 377


PrometricMCQ.com 11/03/2015 10:45

Regulate breast feeding every 3


hours (Correct Answer)

That’s normal feeding problem

Shift to bottle feeding

Start weaning your baby

Report an issue

While caring for a child with a


ventriculoperitoneal shunt revision, the
nurse
finds the patient lying with the head and
feet flexed back. The nurse should
call for help and prepare for a (n):

Spinal tap (Correct Answer)

Shunt culture

Electrocardiogram

Ventricular tap

Report an issue

While assessing a child with pulmonary


stenosis, the nurse should give
PRIORITY to:

Deep tendon reflexes

file:///Users/mohammad/Desktop/PrometricMCQ.com.webarchive Page 334 of 377


PrometricMCQ.com 11/03/2015 10:45

Urinary output

Exercise tolerance (Correct


Answer)

Pattern of food intake

Report an issue

A 25 years-old male patient suffered a


spinal cord injury at the T-4 level and is
being cared for in hospital. The nurse
enters the patient’s room and finds the
patient sitting upright and looking anxious
and restless. He complains of
sudden headache and nausea. Sweat
forms on his forehead yet his feet are cool
to touch
Blood pressure 150/100 mmhg
Heart rate 55/min
Respiratory rate 28/min
Temperature 37.1C
What nursing intervention is initially most
appropriate?

Assess for a full bladder (Correct


Answer)

Lower the head of the bed 30


degrees

Loosen clothing and bed sheets

Apply heating pad to lower


extremities

Report an issue

file:///Users/mohammad/Desktop/PrometricMCQ.com.webarchive Page 335 of 377


PrometricMCQ.com 11/03/2015 10:45

An 80 year-old man presents to the


hospital with chronic fatigue, dyspepsia
and constipation. On examination he is
jaundiced, has red palms, dilated veins
around the umbilicus, the abdomen is very
distended and he has black, tarry
stool on a rectal exam. He is noted to be
lethargic and have a flat tone. A
paracentesis reveals clear colored fluid
with low protein content.
What is most likely diagnosis?

Gilbert’s syndrome

Thalassemia

Sickle cell crisis

Cirrhosis (Correct Answer)

Report an issue

Admission Report at 22:45 (10:45p.m.)


Diagnosis
14 years old admitted
18 hours ago
Remains febrile, maximum temperature
this shift 39.3C
Respiration rate 12-14 per minute
Heart rate 100-180 per minute
Blood pressure level within baseline
normal range
Next dose of intravenous anti-infective is
due at 08:00 (8:00 a.m.)
Bacterial
pneumonia
Refer to the accompanying figure.
The nurse should plan to monitor which of
the following at least every 4

file:///Users/mohammad/Desktop/PrometricMCQ.com.webarchive Page 336 of 377


PrometricMCQ.com 11/03/2015 10:45

hours?

Vital signs (Correct Answer)

Urinary output

Food intake

Level of activity

Report an issue

Which instruction take priority in reducing


anxiety related to surgery?

Surgical procedure and


postoperative exercises

Risk of infection after surgery

Advanced directive and what it mea

Pre-operative laboratory result and


what to expect on it (Correct
Answer)

Report an issue

When caring for a patient with an ostomy,


the nurse knows that extra
protection for peristomal skin is MOST
important for those patients with a
(an):

file:///Users/mohammad/Desktop/PrometricMCQ.com.webarchive Page 337 of 377


PrometricMCQ.com 11/03/2015 10:45

Ileostomy (Correct Answer)

Ascending colostomy

Transverse colostomy

Sigmoid colostomy

Report an issue

Which of the following nursing diagnosis


takes PRIORITY for a patient
admitted to the critical care unit with
diabetic ketoacidosis?

Deficient fluid volume secondary to


hyperglycemia

Risk of infection secondary to


weakened immune system

Deficient knowledge of cause and


prevention (Correct Answer)

Imbalanced nutrition related to


hyperglycemic state

Report an issue

A 69 year-old man is admitted to the


intensive care unit following cardiac
surgery. Two hours after admission, the
nurse performs a routine assessment
and notes the patient’s chest tube drainage

file:///Users/mohammad/Desktop/PrometricMCQ.com.webarchive Page 338 of 377


PrometricMCQ.com 11/03/2015 10:45

is 200 milliliters and a dark red


color. He has had 60 milliliters output from
the indwelling urinary catheter
Blood pressure 138/68 mmhg
Heart rate 76/min
Respiratory rate 16/min
Body temperature 37.0C oral
Oxygen saturation 94% 6L/min nasal
cannula
Which finding should be reported to the
doctor?

Dark red chest tube drainage


(Correct Answer)

Urinary output

Oxygen saturation

Chest tube output volume

Report an issue

To minimize a toddler from scratching and


picking at healing skin graft, the
nurse should utilize:

Mild sedatives

Hand mittens (Correct Answer)

Punishment for picking

Distractions

Report an issue

file:///Users/mohammad/Desktop/PrometricMCQ.com.webarchive Page 339 of 377


PrometricMCQ.com 11/03/2015 10:45

The following pain medications are ordered


for a patient who had a right leg
amputation: Oxycodone 5 mg every 4
hours as needed and morphine 5 mg
every 4 hours as needed. The nurse
administered oxycodone 2 hours ago, but
the patient reports pain rated 8 on a scale
of 0 (no pain) to 10 (severe pain) as
the dressing change begins
After evaluating the effectiveness of the
pain medication, what action should
the nurse take?
Blood pressure 169/98 mmhg
Heart rate 112/min
Respiratory rate 22/min
Temperature 36.7C

Administer additional oxycodone 5


mg

Administer morphine 5 mg
(Correct Answer)

Change the dressing quickly

Ask the patient to wait 2 hours

Report an issue

A 55 year-old man presented to the clinic


with complaints of numbness and
tingling in his feet for the past 3 months.
He appears thin and his skin is pale.
Examination confirms the loss of vibration
sense. He also has altered
proprioception. He had a subtotal
gastrectomy two years ago (see lab results)
Test result normal values
MCH 2.72 1.45-2.01 fmol/cell

file:///Users/mohammad/Desktop/PrometricMCQ.com.webarchive Page 340 of 377


PrometricMCQ.com 11/03/2015 10:45

HCT 0.31 0.41-0.50


MCV 105 81-97 fl
Reticulocyte count 1.5 0.5%-2.5%
Which supplement would most likely
improve his condition?

Iron

B12 (Correct Answer)

Folate

Calcium

Report an issue

A 35 year-old man is hospitalized following


a blunt chest injury. He has chest
pain, breathing difficulty and asymmetrical
lung expansion. Chest
auscultation reveals decreased lung
sounds on the left side. The jugular veins
are distended and there is tracheal
deviation to the right.
Blood pressure 92/54 mmhg
Heart rate 120/min
Respiratory rate 24/min
Temperature 36.2C
SaO2 88% on oxygen
Which initial intervention is most
appropriate?

Send patient for X-ray examination


(Correct Answer)

Administer bronchodilators

Administer intravenous fluid bolus

Prepare for needle decompression

file:///Users/mohammad/Desktop/PrometricMCQ.com.webarchive Page 341 of 377


PrometricMCQ.com 11/03/2015 10:45

Report an issue

A patient is being followed in the clinic for


hypertension, adult onset diabetes,
and obesity. The patient is apathetic about
learning about nutritional
guidelines to reach the goals of weight loss
and consumption of a healthy diet.
The patient admitted to eating “whatever is
put in front of me”
Which of the following actions would the
nurse take?

Collaborate with the patient to set


goals

Add a nursing diagnosis of non-


compliance

Refer for psychiatric screening for


depression

Discuss nutritional interventions with


the spouse (Correct Answer)

Report an issue

The nurse is measuring the chest tube


drainage of a patient who had open
heart surgery 4 hours ago.
Which of the following is the MAXIMUM
hourly amount of chest tube
drainage is expected in this timeframe?

file:///Users/mohammad/Desktop/PrometricMCQ.com.webarchive Page 342 of 377


PrometricMCQ.com 11/03/2015 10:45

100 ml

200 ml (Correct Answer)

300 ml

400 ml

Report an issue

A nurse is caring for a three year-old child


with a fractured arm.
Which of the following interventions is the
MOST appropriate for pain
management?

Administer analgesics when


necessary (Correct Answer)

Assess pain once a shift

Anticipate pain and intervene early

Encourage the use of self-quieting


techniques

Report an issue

After a hearing restoration operation, a


patient has no signs of complications
and soon recovers.
Which of the following is an expected
outcome 5 days after the hearing
restoration surgery?

file:///Users/mohammad/Desktop/PrometricMCQ.com.webarchive Page 343 of 377


PrometricMCQ.com 11/03/2015 10:45

Regain full hearing (Correct


Answer)

Minimal facial nerve paralysis

Minimal urinary incontinence

Ambulates without difficulty

Report an issue

An adolescent patient with a history of


migraine headaches for the past
year has been followed for pain
management. The headaches have
decreased in severity and now occur only
occasionally
What are the BEST three long term goals?

A, D, F

B, D, F

C, E, F (Correct Answer)

D, E, G

Report an issue

A community health nurse visits a patient


who had a cerebrovascular
accident. The patient is at risk for deficient
fluid volume due to voluntary
reduction of fluid intake to avoid the use of
the bathroom. The nurse educates

file:///Users/mohammad/Desktop/PrometricMCQ.com.webarchive Page 344 of 377


PrometricMCQ.com 11/03/2015 10:45

the patient on the importance of drinking


fluids and maintaining hydration/
Which of the following indicates the
efficacy of the nursing intervention?

Amber color urine

Respiration of 35

Tachycardia

Moist mucous membrane (Correct


Answer)

Report an issue

A patient with diabetes mellitus and


multiple sclerosis has been prescribed
baclofen (Lioresal). The nurse knows this
medication may result in?

Increased insulin needs

Renal failure

Optic neuritis

Muscle tremors (Correct Answer)

Report an issue

Nursing diagnosis
A Impaired physical mobility
B Activity intolerance

file:///Users/mohammad/Desktop/PrometricMCQ.com.webarchive Page 345 of 377


PrometricMCQ.com 11/03/2015 10:45

C Alternation of comfort
D Risk of alternation in skin integrity
E Knowledge deficit
F Decreased cardiac output
A patient with Alzheimer’s disease and
severe cardiomyopathy presents to the
hospital with a fractured left hip. The
patient is on bed rest until a cardiologist
clears the patient to have surgery.
Which nursing diagnosis have the highest
priority?

E and F

D and C

C and F (Correct Answer)

D and F

Report an issue

A 19 year-old woman telephones the nurse


and complains of difficulty of
breathing. The symptoms developed while
exercising that morning. She has a
productive cough with thick mucus
secretions and wheezing. The nurse hears
that the breathing problems do not
interfere with talking on the phone. The
woman plans to attend the clinic but must
wait for two hours before she can
be driven there.
Which treatment should be recommended
first by the physician?

Purse-lip breathing

Deep breathing and coughing

Decrease physical activity (Correct


file:///Users/mohammad/Desktop/PrometricMCQ.com.webarchive Page 346 of 377
PrometricMCQ.com 11/03/2015 10:45

Answer)

B2 agonist administration

Report an issue

A patient is recovering from surgery using


spinal anesthesia. The patient
developed a spinal headache.
Which of the following nursing actions
would be MOST appropriate?

Elevate the head of the bed 30


degrees

Keep the patient well hydrated


(Correct Answer)

Limit intake of salty foods

Lower the temperature of the room

Report an issue

A patient is preparing for a total knee


replacement. During the preoperative
interview process the patient reports an
allergic reaction to penicillin.
Which of the following is considered a side
effect and not a true allergy to
medication?

Shortness of breath

file:///Users/mohammad/Desktop/PrometricMCQ.com.webarchive Page 347 of 377


PrometricMCQ.com 11/03/2015 10:45

Tingling lips and tongue

Rash

Upset stomach (Correct Answer)

Report an issue

Prior to providing care for a hospitalized


infant, the nurse who focuses on
preventive measures must:

Introduce self to parents

Perform hand hygiene (Correct


Answer)

Have a witness present

Assess the child’s


developmental level

Report an issue

As identified by Dr. Elizabeth Kubler-Ross,


which stage of dying is
characterized by the transition from, “NO,
not me” to “Yes, me, but…”

Anger

Depression

Acceptance

file:///Users/mohammad/Desktop/PrometricMCQ.com.webarchive Page 348 of 377


PrometricMCQ.com 11/03/2015 10:45

Bargaining (Correct Answer)

Report an issue

A 50 year-old male presents to the medical


office 3 weeks after cardiac surgery
with complaints of a feeling of weakness,
difficulty breathing, and joint pains.
Upon examination the nurse finds a fever
and a friction rub on auscultation of
the chest. The nurse recognizes that the
MOST likely surgical complication is:

Neuropsychological dysfunction

Postpericardiotomy syndrome
(Correct Answer)

Cardiac tamponade

Phrenic nerve damage

Report an issue

A patient with heart failure has the


following vital signs, which of these vital
signs should be reported to the physician
prior to administrating the next dose
of digoxin?
Blood pressure 136/84 mmhg
Heart rate 48/min
Respiratory rate 20/min
Temperature 37.1C

file:///Users/mohammad/Desktop/PrometricMCQ.com.webarchive Page 349 of 377


PrometricMCQ.com 11/03/2015 10:45

Blood pressure

Pulse (Correct Answer)

Temperature

Respiratory rate

Report an issue

The floating ribs that are not attached with


sternum are:

Ribs 7&8

Ribs 9&10

Ribs 11&12 (Correct Answer)

Ribs 9&10&11&12

Report an issue

A pediatric nurse is providing medication


instructions to the mother of a 13
year-old boy who is starting anti-
depressant therapy. The mother appears
unfocused, agitated and confused and
asks the nurse to repeat the instructions
several times. She tells the nurse that she
is concerned that she might
administer the medication incorrectly. The
nurse is concerned that the mother
is not following the instructions.
Which intervention would be most

file:///Users/mohammad/Desktop/PrometricMCQ.com.webarchive Page 350 of 377


PrometricMCQ.com 11/03/2015 10:45

appropriate?

Reassure her that doubts are normal

Reinforce the importance of correct


dosage

Refocus the teaching with printed


material (Correct Answer)

Take a small break and the continue

Report an issue

A child is treated for superficial (first-


degree) thermal burns to the thigh. The
child is in great discomfort and does not
eat.
Which of the following diagnosis should
receive PRIORITY?

Altered nutrition (Correct Answer)

Impaired skin integrity

Risk of infection

Acute pain

Report an issue

A 3-week-old infant is hospitalized with


jaundice. When considering the fluid
needs relative to body size of the infant as

file:///Users/mohammad/Desktop/PrometricMCQ.com.webarchive Page 351 of 377


PrometricMCQ.com 11/03/2015 10:45

compared to the fluid needs of an


adult. The nurse knows this infant requires:

Less fluids

More fluids (Correct Answer)

Same amount of fluids

Much less fluids

Report an issue

Which of the following statement is most


accurate?

Girls have more ADHD than boys

Boys are more frequently diagnosed


with chronic illnesses (Correct
Answer)

Boys are less frequently diagnosed


with chronic illnesses

Girls have more other chronic illness


than ADHD

file:///Users/mohammad/Desktop/PrometricMCQ.com.webarchive Page 352 of 377


PrometricMCQ.com 11/03/2015 10:45

Report an issue

What is the percentage of boy with


attention deficit hyperactivity disorder?

12% (Correct Answer)

16%

8%

4%

Report an issue

The nurse is observing a detailed


neurological assessment on a client with a
suspected brain tumor. When performing
the Romberg test, the client sways
when the eyes are both open and closed.
What does this indicate?

The problem is probably in the

file:///Users/mohammad/Desktop/PrometricMCQ.com.webarchive Page 353 of 377


PrometricMCQ.com 11/03/2015 10:45

cerebellum (Correct Answer)

It is a position sense abnormality

This is not an abnormal test result

The client has lost proprioception

Report an issue

An 18 year-old woman who broke her right


ankle is seen in the physician’s
office one week after the cast was
removed.
Which of the following is the short term
goal for this patient?

Walk 100 feet with crutches


(Correct Answer)

Walk completely independent

Relieve the pain

Strict bedrest

Report an issue

A client with multiple sclerosis has been


prescribed the drug baclofen
(Lioresal). What is the action of this drug?

Reduces spasticity (Correct


Answer)

file:///Users/mohammad/Desktop/PrometricMCQ.com.webarchive Page 354 of 377


PrometricMCQ.com 11/03/2015 10:45

Skeletal muscle relaxation

Immune suppression

Prevents viral infections

Report an issue

What is the average incubation period of


Hepatitis A?

30 days (Correct Answer)

60 days

50 days

14 days

Report an issue

After the health care team meets to


discuss the client’s nursing needs, the
nursing diagnosis “Disturbed body image”
is added to the care plan. The best
rationale for adding this nursing diagnosis
to the care plan in the case of a
female is that females with Cushing’s
syndrome typically experience which
physiologic effect?

Masculine characteristics (Correct


Answer)

file:///Users/mohammad/Desktop/PrometricMCQ.com.webarchive Page 355 of 377


PrometricMCQ.com 11/03/2015 10:45

Heavy menstrual flow

Extreme weight loss

Large, pendulous breasts

Report an issue

A 64-year-old client with uterine cancer is


scheduled to undergo an abdominal
hysterectomy under general anesthesia.
Before the client returns from the
postanesthesia care unit, the registered
nurse asks the licensed practical nurse
to help revise the care plan for the client
who has undergone a hysterectomy.
Which nursing diagnosis is most
appropriate for the nurse to add to the
client’s care plan at this time?

Risk for ineffective airway


clearance (Correct Answer)

Risk for imbalanced nutrition

Ineffective coping

Impaired verbal communication

Report an issue

The nurse initiates a teaching plan for the


client with Parkinson’s disease.
Which instruction should be the nurse’s
priority in this situation?

file:///Users/mohammad/Desktop/PrometricMCQ.com.webarchive Page 356 of 377


PrometricMCQ.com 11/03/2015 10:45

Steps to enhance the client’s


immune system

Importance of maintaining a
balanced diet

Need to remove all safety hazards


(Correct Answer)

Importance of social interactions

Report an issue

The charge nurse enters the nursing


diagnosis “Risk for ineffective airway
clearance related to an inability to swallow”
on the client’s care plan. Which
nursing intervention is most appropriate for
managing the identified
problem?

Keeping the client supine

Removing all head pillows

Performing oral suctioning


(Correct Answer)

Providing frequent oral hygiene

Report an issue

The client has returned from surgery with a

file:///Users/mohammad/Desktop/PrometricMCQ.com.webarchive Page 357 of 377


PrometricMCQ.com 11/03/2015 10:45

leg cast, and the nurse is assisting


the client back to bed. Which of the
following would the nurse identify as the
highest priority when documenting the
postoperative circulation status of the
recently casted extremity?

Adequate neurovascular
functioning (Correct Answer)

Minimal pain on movement

Vital signs within normal limits

No drainage noted on the case

Report an issue

After X-rays are taken of the head, neck,


and spine, the client is diagnosed
with a head injury and admitted for
inpatient care. When assessing the client
with a head injury, which of the following
should receive priority attention?

Lung sounds

Clarity of speech

Mobility of fingers

Pupillary responses (Correct


Answer)

Report an issue

file:///Users/mohammad/Desktop/PrometricMCQ.com.webarchive Page 358 of 377


PrometricMCQ.com 11/03/2015 10:45

The client experiencing a severe allergic


reaction becomes pulseless. The nurse
shakes the client, shout the client’s name
but gets no response, and activates
the emergency medical response system.
Which nursing action becomes the
next priority?

Administer a single blow to the


sternum

Give two quick breaths that make


the chest visibly rise

Begin chest compressions at a rate


of 100 per minute (Correct
Answer)

Administer an epinephrine
(Adrenalin) injection

Report an issue

A 50-year-old client is scheduled for a


heart transplant tomorrow. The night
nurse is asked to review the surgical
procedure with the client. Because of the
client’s anxiety, the client has difficulty
comprehending the nurse’s
information. During the postoperative
period, what is the best rationale for
the nurse frequently assessing the client’s
fluid status?

Urine retention is common after a


heart transplant

file:///Users/mohammad/Desktop/PrometricMCQ.com.webarchive Page 359 of 377


PrometricMCQ.com 11/03/2015 10:45

Urine output is an indication of


perfusion to the kidneys (Correct
Answer)

Hydration determines when the


client needs to be transfused

Hydration indicates when fluids


should be increased

Report an issue

A nurse is assigned to care for a patient


with a diagnosis of thrombotic stroke.
The nurse knows that this type of stroke is
MOST LIKELY caused by:

Blockage of large vessels as a result


of atherosclerosis

Emboli produced from valvular heart


disease (Correct Answer)

Decreased cerebral blood flow due


to circulatory failure

A temporary disruption in
oxygenation of the brain

Report an issue

A nurse obtains a urine dipstick analysis


sample from a 35 year-old woman
who reports having burning sensation with
urination and a sense of urgency

file:///Users/mohammad/Desktop/PrometricMCQ.com.webarchive Page 360 of 377


PrometricMCQ.com 11/03/2015 10:45

and frequency. She had been diagnosed


with the condition six months
previously and was prescribed a course of
antibiotics.
Urinalysis Results Normal values
Colour Dark yellow Straw-coloured
Odour Abnormal Almost nothing
Appearance Turbid Clear
Leukocyte esterase Positive Negative
Nitrites Positive Negative
Which type of pharmacological treatment
is most likely?

Anti-viral

Anti-fungal

Anti-bacterial (Correct Answer)

Anti-parasitic

Report an issue

A 30 year-old woman has been prescribe


albuterol PRN and prophylactic
inhaled corticosteroids to be taken once
per day. She has been taking the
prophylactic as prescribed but has needed
to use albuterol more often than
usually. She has a chronic cough and often
has air hunger.
Which intervention is initially most
appropriate?

Refer for a chest X-ray

Administer magnesium sulfate

Assess peak flow measure

file:///Users/mohammad/Desktop/PrometricMCQ.com.webarchive Page 361 of 377


PrometricMCQ.com 11/03/2015 10:45

Perform arterial blood gas


sampling (Correct Answer)

Report an issue

Which of the following drugs is not used in


the treatment of pulmonary
embolism?

Heparin

Warfarin

Digoxin (Correct Answer)

Streptokinase

Report an issue

A home care patient with chronic


obstructive pulmonary disease (COPD)
reports an upset stomach. The patient is
taking theophylline (Theo-Dur) and
triamcinolone acetonide (Azmacort). The
nurse should instruct the patient to
take:

Theo-Dur on an empty stomach

Theo-Dur and Azmacort at same


time

Theo-Dur and Azmacort 12 hours

file:///Users/mohammad/Desktop/PrometricMCQ.com.webarchive Page 362 of 377


PrometricMCQ.com 11/03/2015 10:45

apart

Theo-Dur with milk or crackers


(Correct Answer)

Report an issue

A 65 year-old woman presented to her care


provider with complaints of bright
red blood in the stool, a loss of appetite, a
feeling of fullness and fatigue. She
had lost 5 kilograms in the past three
weeks without dieting. A faecal occult
blood test is positive and the patient is
scheduled for additional screening test.
Which screening test is most likely for this
patient?

Barium enema

Colonoscopy

Endoscopy (Correct Answer)

Computed tomography scan

Report an issue

A baby girl was born prematurely at 33


weeks gestation due to placenta
abruption. She is now two months old and
has a potential hearing deficit. In
the neonatal intensive care unit (NICU) she
had passed the initial screening
tests for hearing but has significant risk
factors that suggest further

file:///Users/mohammad/Desktop/PrometricMCQ.com.webarchive Page 363 of 377


PrometricMCQ.com 11/03/2015 10:45

evaluation.
Which autonomic reflex should be further
evaluated?

Moro (Correct Answer)

Babinski

Pupillary

Tonic neck

Report an issue

A 45 year-old woman presented with a


generalized rash that is not itchy. She
reports that she has had the problem for
the past 15 years. Examination
reveals a well-outlined, reddish plaque over
the right gluteal fold. The plaque
has scales over it and is cracked in some
areas.
Which intervention is initially appropriate?

Apply topical cream to the affected


area (Correct Answer)

Expose area to sunlight for twenty


minutes daily

Maintain immunosuppressant
therapy regimen

Increase dietary intake of vitamin A

Report an issue

file:///Users/mohammad/Desktop/PrometricMCQ.com.webarchive Page 364 of 377


PrometricMCQ.com 11/03/2015 10:45

The patient is receiving mechanical


ventilation set at fraction of inspired
oxygen (FiO2) 100%.
The nurse should understand that which of
the following can improve this
patient’s oxygenation?

Adding positive end expiratory


pressure (PEEP) (Correct Answer)

Place the patient in trendelenburg


position

Increasing the FiO2

Suctioning the patient hourly

Report an issue

A nurse is caring for a 3-week-old infant


who just admitted to the hospital.
Which of the following nursing
interventions does NOT support this
infant’s
basic emotional and social needs?

Provide for continual contact


between parents and infant

Actively involve parents in caring for


the infant

Keep the infant’s environment


quite, dim and free of sensory
stimulation

Foster infant-sibling relationships as

file:///Users/mohammad/Desktop/PrometricMCQ.com.webarchive Page 365 of 377


PrometricMCQ.com 11/03/2015 10:45

appropriate (Correct Answer)

Report an issue

A community health nurse is teaching a


health class about infectious diseases
process.
The nurse instructs the class that rabies
would be considered which of the
following type of infection?

Viral (Correct Answer)

Protozoan

Fungal

Bacterial

Report an issue

A couple who wants to conceive but has


been unsuccessful during the last 2
years has undergone many diagnostic
procedures. When discussing the
situation with the nurse, one partner states,
“We know several friends in our
age group and all of them have their own
child already, why can’t we have
one?” Which of the following would be the
most pertinent nursing diagnosis
for this couple?

Fear related to the unknown


file:///Users/mohammad/Desktop/PrometricMCQ.com.webarchive Page 366 of 377
PrometricMCQ.com 11/03/2015 10:45

Pain related to numerous


procedures

Ineffective family coping related to


infertility

Self-esteem disturbance related to


infertility (Correct Answer)

Report an issue

A 64 year old male client with a long


history of cardiovascular problem
including hypertension and angina is to be
scheduled for cardiac
catheterization. During pre-cardiac
catheterization teaching, the nurse should
inform the client that the primary purpose
of the procedure is:

To determine the existence of CHD

To visualize the disease process in


the coronary arteries (Correct
Answer)

To obtain the heart chambers


pressure

To measure oxygen content of


different heart chambers

Report an issue

file:///Users/mohammad/Desktop/PrometricMCQ.com.webarchive Page 367 of 377


PrometricMCQ.com 11/03/2015 10:45

A gravida 3 para 2 is admitted to the labor


unit. Vaginal exam reveals that the
client's cervix is 4cm dilated. The patient
complains of pain which she stated
to be 7/10.at this time and until a full cervix
dilatation is achieved, what is the
priority nursing goal for such patient at this
time is:

Pain management (Correct


Answer)

Prevent fetal distress

Preparing the patient for anesthesia

Keeping this patient NPO

Report an issue

A client with iron deficiency anemia is


scheduled for discharge. Which
instruction about prescribed ferrous
gluconate therapy should the nurse
include in the teaching plan?

“Take the medication with an


antacid.â€​

“Take the medication with a


glass of milk.â€​

“Take the medication with


cereal.â€​

“Take the medication on an


empty stomach.â€​ (Correct
Answer)

file:///Users/mohammad/Desktop/PrometricMCQ.com.webarchive Page 368 of 377


PrometricMCQ.com 11/03/2015 10:45

Report an issue

After 4 hours of active labor, the nurse


notes that the contractions of a
primigravida client are not strong enough
to dilate the cervix. Which of the
following would the nurse anticipate
doing?

Obtaining an order to begin IV


oxytocin infusion (Correct Answer)

Administering a light sedative to


allow the patient to rest for several
hour

Preparing for a cesarean section for


failure to progress

Increasing the encouragement to the


patient when pushing begins

Report an issue

A 24 year-old woman was prescribed


loratidine (Claritin) 10 mg tablet q 12
hours a.c. for allergy.
The nurse reviews the medication order
and explains to the patient the
relation of this drug to meals is as
following:

The drug is to be taken after meals

file:///Users/mohammad/Desktop/PrometricMCQ.com.webarchive Page 369 of 377


PrometricMCQ.com 11/03/2015 10:45

The drug is to be taken during meals

The drug is to be taken before


meals (Correct Answer)

The drug is to be taken away from


meals

Report an issue

An elderly client is experiencing an


alteration in his equilibrium and
coordinated muscle movements. The nurse
realizes that these functions are
controlled by which area of the nervous
system?

Brain stem

Cerebrum

Diencephalon

Cerebellum (Correct Answer)

Report an issue

A 32-year-old female is admitted for a


hemorrhoidectomy. During the nursing
assessment, all of the following factors are
elicited. Which one is most likely to
have contributed to the development of
hemorrhoids?

file:///Users/mohammad/Desktop/PrometricMCQ.com.webarchive Page 370 of 377


PrometricMCQ.com 11/03/2015 10:45

The client states that she usually


cleans herself from back to front
after a bowel movement

The client says her mother and


grandmother had hemorrhoids

The client has had four


pregnancies (Correct Answer)

The client eats bran every day

Report an issue

A client who has hepatitis A asks, “How


could I have gotten this disease?”
what is the nurse’s best response?

You may have eaten contaminated


food (Correct Answer)

You could have contracted the


disease by using intravenous drugs

You must have received and


infected blood

You could have contracted the


disease by engaging in unprotected
sex

Report an issue

A client undergoes right mastectomy for

file:///Users/mohammad/Desktop/PrometricMCQ.com.webarchive Page 371 of 377


PrometricMCQ.com 11/03/2015 10:45

carcinoma. When teaching the client


post-mastectomy exercises, it is important
for the nurse to:

Exercise both arms simultaneously


(Correct Answer)

Exercise the right arm only

Have the client wear a sling between


exercise periods

Wait until the incision has healed

Report an issue

The following picture represents a newborn


reflex known as?

Rooting reflex (Correct Answer)

Moro reflex

Grasping reflex

Startle reflex

Report an issue

file:///Users/mohammad/Desktop/PrometricMCQ.com.webarchive Page 372 of 377


PrometricMCQ.com 11/03/2015 10:45

The nurse is caring for a client who has


had a right modified radical
mastectomy this morning. Which exercise
should the nurse encourage the
client to perform this evening?

Hair combing exercises with the


right arm

Wall climbing exercises with the


right arm

Movement of the fingers and wrists


of the right arm (Correct Answer)

Exercises of the left arm only

Report an issue

A 38-year-old client who has mitral


stenosis is hospitalized for a valve
replacement. Which condition is the client
most likely to report having had
earlier in life?

Meningitis

Syphilis

Rheumatic fever (Correct Answer)

Rubella

Report an issue

file:///Users/mohammad/Desktop/PrometricMCQ.com.webarchive Page 373 of 377


PrometricMCQ.com 11/03/2015 10:45

An 82-year-old woman who has


Alzheimer’s disease is admitted to the
acute
care unit. She frequently gets out of bed
and wanders in the hall, unable to
find her way back to her room. She even
gets in the beds of other clients. What
nursing action is most appropriate for this
client?

Restrain her so she will not wander


in the halls

Ask her roommate to call the nurse


whenever she leaves the room

Punish her when she gets in a bed


other than her own

Put her favorite picture on the door


to her room (Correct Answer)

Report an issue

A 35-year-old man is admitted with severe


renal colic. The nurse should
monitor this man for possible
complications. Which of the following is a
complication of renal colic?

Anemia

Polyuria

Hypertension

file:///Users/mohammad/Desktop/PrometricMCQ.com.webarchive Page 374 of 377


PrometricMCQ.com 11/03/2015 10:45

Oliguria (Correct Answer)

Report an issue

A nurse discusses high-risk complications


with a group of women at a prenatal
clinic. Which client would the nurse identify
as being at highest risk for
developing complications during
pregnancy?

A 25-year-old gravida I client

A client with the placenta implanted


on the fundus of the uterus

A client who has nausea and


vomiting during the first trimester

A 30-year-old client with DM


(Correct Answer)

Report an issue

A 56 year-old man was brought to the


emergency room by his relatives, on
examination he appears sick and has
severe weakness. One of his relatives told
the nurse that the man eats nearly nothing
and have been crying most of the
time for 3 months now since he lost his
son in an accident. The nurse knows
that this patient is suffering from:

file:///Users/mohammad/Desktop/PrometricMCQ.com.webarchive Page 375 of 377


PrometricMCQ.com 11/03/2015 10:45

Mania

Insomnia

Depression (Correct Answer)

Schizophrenia

Report an issue

Which of the following drugs is used for the


treatment of digitalis toxicity?

Protamine sulfate

Streptokinase

Theophylline

Digoxin immune Fab (Correct


Answer)

Report an issue

A female client is admitted with a diagnosis


of acute renal failure. She is
awake, alert, oriented, and complaining of
severe back pain, nausea and
vomiting and abdominal cramps.
Blood pressure 100/70 mm Hg
Pulse 110
Respiratory rate 30
Temperature 38°C oral
Sodium 120 mEq/L

file:///Users/mohammad/Desktop/PrometricMCQ.com.webarchive Page 376 of 377


PrometricMCQ.com 11/03/2015 10:45

Potassium 5.2mEq/L
Urinary output for the first 8 hours is 50 ml
The client is displaying signs of which
electrolyte imbalance?

Hyponatremia (Correct Answer)

Hyperkalemia

Hyperphosphatemia

Hypercalcemia

Report an issue

file:///Users/mohammad/Desktop/PrometricMCQ.com.webarchive Page 377 of 377

You might also like